Você está na página 1de 108

MATERIAL DE APOIO FSICA EXAME NACIONAL DO ENSINO MDIO

ENEM 2009

SANDRO FERNANDES

Prof: Sandro Fernandes

ndice
Aula I Aula II Aula III Aula IV Aula V Aula VI (Conhecimentos Bsicos e o Movimento)...................Pg 3. (O Equilbrio e a descoberta das Leis Fsicas )............Pg 20. (A Mecnica e o Funcionamento do Universo)...........Pg 35. (O Calor e os Fenmenos Trmicos)..........................Pg 48. (Energia, Trabalho e Potncia)..................................Pg 63. (Fenmenos Eltricos e Magnticos).........................Pg 79.

Aula VII (Oscilaes, Ondas, pticas e Radiao).....................Pg 94. Gabarito ...................................................................................Pg 108.

Esse material tem como objetivo facilitar os estudos dos candidatos quer iro prestar o ENEM 2009. As aulas esto separadas de acordo com o contedo programado na Matriz de Referncia. Leia os textos de apoio e s ento passe para os exerccios. Os textos e exerccios tambm sero de valor para os candidatos ao Exame de Qualificao da UERJ, contudo, neste caso algumas aulas no devem ser estudadas j que o contedo cobrado na primeira fase do vestibular da UERJ mais condensado, privilegiando principalmente a mecnica, a termologia e a eletrodinmica.

Bons estudos!

Prof: Sandro Fernandes

De acordo com a Matriz de Referncias para o Enem 2009. Professor: Sandro Fernandes Tema: Conhecimentos Bsicos e o Movimento

Aula I de Fsica
(I) Como Aristteles e Galileu Analisavam O Movimento?
Para Aristteles (384 322 a.C) havia dois tipos de movimento: O natural Cada um dos quatro elementos possui um lugar bem definido no universo. O movimento natural de um corpo consiste em uma busca pelo seu lugar natural. O movimento de queda de uma pedra ou da gua, por exemplo, um movimento natural, pois visa retornar aos seus lugares naturais. O forado Deve estar associado presena constante de uma fora. Para ele o meio tambm desempenha um papel fundamental no movimento, oferecendo resistncia e sustentao ao movimento. Apesar de nunca ter analisado matematicamente suas idias, podemos concluir que para ele: A velocidade diretamente proporcional fora aplicada no corpo. Quanto maior a fora maior a velocidade. Ao cessar a fora cessa o movimento. A velocidade inversamente proporcional resistncia oferecida pelo meio. De acordo com suas idias, um corpo abandonado longe de seu lugar natural retorna a ele tanto mais rpido quanto o meio permitir. Vale frisar que a idia de um vcuo hipottico implicaria em uma velocidade infinita o que era (e continua sendo) uma idia absurda. J Galileu descreve o movimento atravs do princpio que hoje chamamos de inrcia de um corpo. A inrcia (apatia, inerte) uma propriedade intrnseca da matria. Com outras palavras, a matria tem a propriedade de resistir s aceleraes. A massa uma medida dessa inrcia. No devemos confundir a massa inercial com a massa gravitacional dada pela lei da gravitao universal de Newton, que indica a capacidade de atrao entre duas massas quaisquer separadas por uma dada distncia. Ao contrrio do que muitos pensam, quando medimos a massa de um corpo no estamos medindo a quantidade de matria que h no corpo. Quantidade de matria se mede em mol. A principal consequncia disso que no h a necessidade de uma fora para manter um movimento, ele se mantm por si s. Foras servem para alterar esse movimento.

Prof: Sandro Fernandes

Alm de inaugurar a dinmica com essa lei que mais tarde foi utilizada por Newton para descrever sua mecnica, Galileu tambm descreveu o movimento pendular, a proporcionalidade entre o deslocamento e o quadrado do tempo. No trecho abaixo podemos verificar um dilogo encontrado em um de seus livros e que mostra com grande riqueza de detalhes a sua anlise do movimento de um pndulo. Logo na primeira jornada dos Discorsi intorno a due nuevo scienze, temos o seguinte dilogo entre Salviati (isto , Galileu) e seu discpulo Sagredo. Salvati: ... Ora, nada disso acontece, mas o tempo mais breve, e conseqentemente o movimento mais veloz, aquele que se faz pelo arco, do qual a linha reta a corda. Quanto proporo entre os tempos de oscilao de mveis suspenso por fios de diferentes comprimentos, esses tempos esto entre si na mesma proporo que as razes quadradas dos comprimentos desses fios, o que quer dizer que os comprimentos esto entre si como os quadrados dos tempos...; do que se segue que os comprimentos dos fios esto entre si na proporo inversa dos quadrados os nmeros de oscilaes realizadas no mesmo tempo. Sagredo: Se entendi bem, eu poderia, portanto, conhecer rapidamente o comprimento de uma corda pendente de qualquer altura, ainda que o ponto a que esta atada fosse invisvel e somente se visse sua extremidade inferior. Com efeito, se amarro parte inferior da corda em questo um peso bastante grande, ao qual comunico um movimento de vaivm, e se um amigo conta o nmero de suas oscilaes enquanto ao mesmo tempo conto tambm as oscilaes de outro mvel, atado a uma corda com o comprimento exato de um cvado, a partir dos nmeros de oscilaes desses pndulos, efetuadas ao mesmo tempo, encontro o comprimento da corda: suponhamos, por exemplo, que no tempo em que um amigo tenha contado vinte oscilaes da corda comprida, eu conto duzentos e quarenta da minha: ... direi que a corda comprida contm 57.600 unidades das quais a minha contm 400; ... direi que aquela corda tem 144 cvados de comprimento. Salviati: V. As. No teria errado nem mesmo de um palmo, especialmente se tomasse um grande nmero de oscilaes. [1] Esse texto mostra que Galileu foi um dos pioneiros no estudo da teoria da semelhana fsica e dos modelos. O pndulo constitudo pela corda comprida com um corpo pesado amarrado sua extremidade inferior o prottipo, e o pequeno pndulo, seu modelo reduzido. Galileu percebeu, no caso das oscilaes de pndulos, que a escala do tempo igual raiz quadrada da escala geomtrica, permitindo prever o comportamento do prottipo a partir de observaes realizadas sobre o modelo: por exemplo, conhecida a escala do tempo, graas comparao dos perodos de oscilao do modelo e do prottipo, possvel deduzir o comprimento do pndulo-prottipo partindo apenas do conhecimento do comprimento do pndulo-modelo. No exemplo dado por Galileu, determinar o comprimento do pndulo-prottipo seria difcil, pois a extremidade superior da corda est fixada em uma altura muito grande. A utilizao do modelo em escala reduzida permite a
Prof: Sandro Fernandes

determinao direta desse comprimento. exatamente essa a filosofia do emprego de modelos reduzidos nas pesquisas experimentais. No exemplo dado por Galileu a escala do tempo determinada da seguinte forma: igual ao inverso da relao entre os nmeros de oscilaes, isto , ao inverso de 240/20, e, portanto 1:12. A escala geomtrica, igual ao quadrado da escala do tempo, ser 1:144, e o comprimento do pnduloprottipo, igual a 144 vezes o do pndulo-modelo. Neste caso a condio de semelhana fsica corresponde lei de Galileu segundo a qual o perodo de oscilao T de um pndulo proporcional raiz quadrada do seu comprimento.

(II)

Sistema de Unidades

A necessidade de medir muito antiga e remonta origem das civilizaes. Por longo tempo cada pas, cada regio, teve o seu prprio sistema de medidas, baseado em unidades arbitrrias e imprecisas, como por exemplo, aquelas baseadas no corpo humano: palmo, p, polegada, braa, cvado. Isso criava muitos problemas para o comrcio, porque as pessoas de uma regio no estavam familiarizadas com o sistema de medida das outras regies. Imagine a dificuldade em comprar ou vender produtos cujas quantidades eram expressas em unidades de medida diferentes e que no tinham correspondncia entre si. Em 1789, numa tentativa de resolver o problema, o Governo Republicano Francs pediu Academia de Cincias da Frana que criasse um sistema de medidas baseado numa "constante natural". Assim foi criado o Sistema Mtrico Decimal. Posteriormente, muitos outros pases adotaram o sistema, inclusive o Brasil, aderindo "Conveno do Metro". O Sistema Mtrico Decimal adotou, inicialmente, trs unidades bsicas de medida: o metro, o litro e o quilograma. Entretanto, o desenvolvimento cientfico e tecnolgico passou a exigir medies cada vez mais precisas e diversificadas. Por isso, em 1960, o sistema mtrico decimal foi substitudo pelo Sistema Internacional de Unidades - SI, mais complexo e sofisticado, adotado tambm pelo Brasil em 1962 e ratificado pela Resoluo n 12 de 1988 do Conselho Nacional de Metrologia, Normalizao e Qualidade Industrial Conmetro, tornando-se de uso obrigatrio em todo o Territrio Nacional.
FONTE: www.inmetro.gov.br

(III)

A Fora da Cincia est na sua Universalidade


Dada a complexidade do mundo em nossa volta, no nada surpreendente que os cientistas usem

simplificaes aparentemente drsticas no estudo de fenmenos naturais. Por exemplo, se quisermos estudar a rbita da Lua em torno da Terra, irrelevante incluirmos em nossa descrio que a Terra tem montanhas, oceanos e atmosfera, ou que a Lua tem crateras de todos os tamanhos. Basta sabermos a massa da Terra e a da Lua e a distncia entre elas. O balano de uma folha ao vento, o vaivm de uma criana num balano, um sino soando: todos esses "sistemas" podem ser modelados, com maior ou menor preciso, pelo movimento de um pndulo sujeito a uma fora externa. No caso da folha, a fora externa vem do vento, no caso da criana, dos empurres de seu pai e, no caso do sino, do padre puxando a corda.
Prof: Sandro Fernandes

Numa primeira aproximao, o modelo matemtico que descreve o movimento desses sistemas essencialmente o mesmo. Pela descrio matemtica dos fenmenos, os fsicos revelam a belssima unidade que existe na natureza. Modelos imitam a natureza, recriando suas sutilezas de forma compreensvel. Descrever o comportamento de sistemas complexos por frmulas simples um ingrediente fundamental no trabalho cientfico e um de seus maiores desafios. H um equilbrio delicado entre simplificar demais _ignorando dados fundamentais sobre um sistema_ e incluir detalhes irrelevantes que compliquem desnecessariamente seu estudo. Para testarmos a eficincia de um modelo, comparamos suas previses com medidas obtidas por cuidadosas observaes. No exemplo da folha balanando ao vento, podemos medir o tempo que a folha demora para voltar a um determinado ponto. Se o resultado medido no for semelhante previso do modelo, este tem de ser modificado. Isso verdade tanto para o balanar de uma folha quanto para qualquer modelo matemtico de descrio de algum fenmeno, de escalas subatmicas at o Universo como um todo. E aqui a intuio do cientista fundamental. Como encontrar as modificaes corretas? Para mim, a construo de modelos uma arte: a de modelar a natureza. Pelo seu processo criativo, o cientista viabiliza sua viso do mundo. Para mim, assim como a obra de um artista, a obra de um cientista um reflexo de sua personalidade. Claro, o veculo de expresso completamente diferente, pois as linguagens so diferentes. Mas o momento que existe entre o surgimento de uma idia e sua expresso, seja por uma equao ou por uma aquarela, essencialmente idntico. Ao recriar o mundo matematicamente, o cientista reinventa a realidade a sua volta, representando-a por smbolos universais. Mesmo que o processo criativo cientfico seja to subjetivo quanto o processo criativo artstico, o produto final do trabalho do cientista acessvel a qualquer outro que domine o vocabulrio tcnico da cincia. (E, espero, tambm ao pblico no especializado por um esforo dos cientistas de transmitir suas idias de modo acessvel.) Em princpio, no deve haver subjetividade na interpretao de uma obra cientfica. Os modelos criados por cientistas so universais. Por meio da universalidade de sua linguagem, esses modelos so gradativamente corrigidos e aprimorados (o progresso cientfico raramente caminha em linha reta), chegando eventualmente a uma formulao aceita pela comunidade cientfica. nessa universalidade que reside a fora da cincia. As equaes que descrevem um fenmeno so idnticas para todos os cientistas, independentemente de qualquer diferena religiosa, racial ou poltica. A natureza no se presta a nossos tolos jogos de poder. A Cincia, em sua verso mais pura, uma das formas mais humanas de conhecimento.
Fonte: http://marcelogleiser.blogspot.com

Prof: Sandro Fernandes

(IV)

Os Problemas sobre o clculo da idade do Universo

Em 1929, o astrnomo americano Edwin Hubble concluiu, a partir de suas observaes, que o Universo est em expanso, com as galxias se afastando umas das outras com velocidades proporcionais a suas distncias. Hubble raciocinou que, como as galxias esto se afastando cada vez mais agora, em algum instante no passado elas estavam praticamente se superpondo. Medindo as velocidades de vrias galxias e as distncias entre elas, ele concluiu que esse instante ocorreu h cerca de 2 bilhes de anos. O problema com essa estimativa que j se sabia que a Terra tinha mais de 2 bilhes de anos. Como ela pode ser mais velha que o Universo? A questo roubou o sono de vrios cosmlogos at 1952, quando Walter Baade demonstrou, com medidas mais precisas, que o Universo teria pelo menos 5 bilhes de anos. A idade da Terra hoje estimada em torno de 4,5 bilhes de anos. Mas a questo da idade do Universo est longe de ser resolvida. Existem trs mtodos usados para se estimar a idade do Universo. O primeiro deles o usado por Hubble e Baade, que estima distncias entre galxias remotas e suas velocidades, extraindo delas a idade do Universo. Em Astronomia, distncias so estimadas a partir de uma lei que diz que a luminosidade de uma fonte cai com o quadrado da distncia. Se temos duas fontes iguais em lugares distintos, sabendo-se a distncia at o lugar mais prximo, podemos estimar a distncia at o lugar mais distante. Assim, Hubble estimou a distncia at a galxia Andrmeda, que est a aproximadamente 2 milhes de anos-luz do Sol. Esses "indicadores de distncia" so fundamentais para se obter medidas precisas de distncia. Em suas observaes, Hubble usou um tipo de estrela conhecida como varivel Cefeida, cuja luminosidade varia periodicamente. Mas encontrar variveis Cefeida ou outros indicadores de distncia em galxias muito distantes no nada fcil. E a que comea o problema dos astrnomos modernos. Diferentes indicadores de distncia resultam em estimativas diferentes de distncia e, portanto, em estimativas diferentes da idade do Universo. Valores atuais variam entre 8 e 25 bilhes de anos! O segundo mtodo utilizado para se estimar a idade do Universo vem do estudo de aglomerados estelares, conjuntos de milhares de estrelas atradas entre si pela gravidade. A idia que nesses aglomerados podem ser encontradas algumas das estrelas mais velhas que existem. Como ns conhecemos razoavelmente bem como uma estrela se desenvolve queimando seu hidrognio como combustvel, podemos estimar sua idade a partir dos diferentes estgios durante sua evoluo. A idade do Universo tem de ser maior do que a idade de suas estrelas mais velhas, ecoando o problema de Hubble com a idade da Terra. Estimativas da idade desses aglomerados estelares variam entre 10 e 14 bilhes de anos. Finalmente, pode-se usar a "nucleocosmocronologia", que se baseia em medidas da abundncia e da produo de istopos radioativos e em estudos da evoluo qumica de nossa galxia para se estimar a poca de formao dos elementos qumicos encontrados no sistema solar. As estimativas indicam uma idade para a Via Lctea de pelo menos 9,6 bilhes de anos, com erros que tendem a aumentar esse valor em mais de 1 bilho ou 2 bilhes de anos. O que podemos concluir agora? Que o Universo tem de 10 a 20 bilhes de anos; que problemas com as vrias medidas de distncia, evoluo estelar e abundncia isotpica sero, em princpio, resolvidos na prxima dcada. Que boatos jornalsticos recentes dizendo que o modelo do Big Bang est errado devido a problemas com a idade do Universo no tm sentido. E que a Cincia est longe de progredir em linha reta ou de forma previsvel.
Prof: Sandro Fernandes Fonte: http://marcelogleiser.blogspot.com

Algumas Aplicaes...
1) A tabela a seguir nos mostra a evoluo dos tempos na corrida dos 100m rasos ao longo da histria das olimpadas:

Fonte: http://esporte.uol.com.br/olimpiadas/modalidades/atletismo/evoluao.jhtm Observando a tabela, pode-se constatar que, ao recorde mundial, associa-se uma velocidade escalar mdia, em m/s, de: a) 10,16 b) 10,12 c) 10,08 d) 10,04 e) 10,02

2) Um caminho percorre trs vezes o mesmo trajeto. Na primeira, sua velocidade mdia de 15 m/s e o tempo de viagem t1. Na segunda, sua velocidade mdia de 20 m/s e o tempo de viagem t2. Se, na terceira, o tempo de viagem for igual a (t1 + t2)/2, qual ser a velocidade mdia do caminho nessa vez? a) 20,00 m/s. b) 17,50 m/s. c) 17,14 m/s. d) 15,00 m/s. e) 15,34 m/s

3) As figuras a seguir representam as posies sucessivas, em intervalos de tempo iguais, e fixos, dos objetos I, II, III e IV em movimento.

O objeto que descreveu um movimento retilneo uniforme foi a) I b) II c) III d) III e IV e) IV

Prof: Sandro Fernandes

4) Quando a gua da chuva corre pelo rio Tite, na cidade de So Paulo, ela inicia um percurso de 700 km at desembocar no rio Paran. Supondo que a velocidade da gua, em mdia, de 4 km/h, o tempo que dever ser gasto no percurso para a gua chegar ao rio Paran, aproximadamente, : a) 5 dias b) 6 dias c) 7 dias d) 8 dias e) 9 dias

5) O IDH um indicador social que considera a qualidade de vida da populao, observando tambm o impacto dos poluentes no meio ambiente nas cidades brasileiras. O grfico a seguir sintetiza os estudos referentes concentrao de monxido de carbono na Grande So Paulo. Nele pode-se verificar que o ar que se respira na cidade de So Paulo tambm est mais limpo, devido a algumas medidas adotadas para a reduo da poluio ambiental, tais como: - instalao de filtros nos escapamentos dos veculos automotores, - melhoria na qualidade dos combustveis, - ampliao da rede de transportes metropolitanos sobre trilhos, - adoo do sistema de rodzio de veculos automotores. A ampliao da rede de trem metropolitano (metr) na cidade de So Paulo visa reduzir o caos do congestionamento urbano, melhorar o transporte coletivo da populao e contribuir com a melhoria da qualidade do ar. Considere uma composio do trem em movimento entre duas estaes seguidas, partindo do repouso na Estao Tiradentes e parando na Estao Luz. O esboo grfico velocidade tempo que melhor representa o movimento :

6) O grfico representa a posio (X) de uma partcula, em funo do tempo (t).

Prof: Sandro Fernandes

Sobre essa partcula, INCORRETO afirmar que sua a) velocidade mxima em t=1s. b) posio nula no instante t=3,5s. c) acelerao constante no intervalo de 0 a 1s. d) velocidade muda de sentido na posio x=4m. e) sua acelerao negativa. 7) O grfico a seguir mostra como varia a velocidade de um mvel, em funo do tempo, durante parte do seu movimento.

O movimento representado pelo grfico pode ser o de uma a) esfera que desce um plano inclinado e continua rolando por um plano horizontal. b) fruta caindo de uma rvore. c) composio de metr, que se aproxima de uma estao e pra. d) bala no interior de um cano de arma, logo aps o disparo. e) um carro de frmula I em uma arrancada. 8) Em uma bicicleta que se movimenta com velocidade constante, considere um ponto A na periferia da catraca e um ponto B na periferia da roda. Analise as afirmaes: I. A velocidade escalar de A igual de B. II. A velocidade angular de A igual de B. III. O perodo de A igual ao de B. Est correto SOMENTE o que se afirma em: a) I b) II c) III d) I e III e) II e III

9) Leia a tira abaixo.

Prof: Sandro Fernandes

10

Calvin, o garotinho assustado da tira, muito pequeno para entender que pontos situados a diferentes distncias do centro de um disco em rotao tm a) mesma freqncia, mesma velocidade angular e mesma velocidade linear. b) mesma freqncia, mesma velocidade angular e diferentes velocidades lineares. c) mesma freqncia, diferentes velocidades angulares e diferentes velocidades lineares. d) diferentes freqncias, mesma velocidade angular e diferentes velocidades lineares. e) diferentes freqncias, diferentes velocidades angulares e mesma velocidade linear. 10) No site www.agespacial.gov.br, da Agncia Espacial Brasileira, aparece a seguinte informao: "O Centro de Lanamento de Alcntara (CLA) vem sendo construdo desde a dcada de 80 e est atualmente preparado para lanar foguetes de sondagem e veculos lanadores de satlites de pequeno porte. Localizado na costa do nordeste brasileiro, prximo ao Equador, a posio geogrfica do CLA aumenta as condies de segurana e permite menores custos de lanamento." Um dos fatores determinantes dessa reduo de custos se deve inrcia do movimento de rotao da Terra. Graas a essa inrcia, o veculo lanador consome menos energia para fazer com que o satlite adquira a sua velocidade orbital. Isso ocorre porque, nas proximidades do Equador, onde se encontra o CLA, a) a velocidade tangencial da superfcie da Terra maior do que em outras latitudes. b) a velocidade tangencial da superfcie da Terra menor do que em outras latitudes. c) a velocidade tangencial da superfcie da Terra igual velocidade orbital do satlite. d) a acelerao da gravidade na superfcie da Terra menor do que em outras latitudes. e) a acelerao da gravidade na superfcie da Terra maior do que em outras latitudes.

11) Um nibus percorre em 30 minutos as ruas de um bairro, de A at B, como mostra a figura:

Considerando a distncia entre duas ruas paralelas consecutivas igual a 100 m, analise as afirmaes: I. A velocidade vetorial mdia nesse percurso tem mdulo 1 km/h. II. O nibus percorre 1500 m entre os pontos A e B. III. O mdulo do vetor deslocamento 500 m. IV. A velocidade vetorial mdia do nibus entre A e B tem mdulo 3 km/h.

Prof: Sandro Fernandes

11

Esto corretas: a) I e III. b) I e IV. c) III e IV. d) I e II. e) II e III

12) Num certo instante, esto representadas a acelerao e a velocidade vetoriais de uma partcula. Os mdulos dessas grandezas esto tambm indicados na figura

Dados: sen 60 = 0,87 cos 60 = 0,50 No instante considerado, o mdulo da acelerao escalar, em m/s, e o raio de curvatura, em metros, so, respectivamente, a) 3,5 e 25 b) 2,0 e 2,8 c) 4,0 e 36 d) 2,0 e 29

13) A figura 1 representa uma sucesso de fotografias de uma atleta durante a realizao de um salto ornamental numa piscina. As linhas tracejadas nas figuras 1 e 2 representam a trajetria do centro de gravidade dessa atleta para este mesmo salto. Nos pontos I, II, III e IV da figura 2, esto representados os vetores velocidade, V, e acelerao, a, do centro de gravidade da atleta.

Os pontos em que os vetores velocidade, V, e acelerao, a, esto representados corretamente so a) II e III. b) I e III. c) II e IV. d) I e IV. e) I e IV

14) Nos esquemas esto representadas a velocidade V e a acelerao a do ponto material P. Assinale a alternativa em que o mdulo da velocidade desse ponto material permanece constante.

Prof: Sandro Fernandes

12

15) No jogo final do Campeonato Paulista de Futebol 2004, Taa 450 Anos, entre os times So Caetano - Paulista Jundia, o goleiro Slvio Lus chuta a bola no tiro de meta para o alto e centro do campo. A trajetria descrita pela bola, desprezando a resistncia do ar, : a) semicircunferncia c) semi-elipse e) hiprbole 16) Uma caminhonete move-se, com acelerao constante, ao longo de uma estrada plana e reta, como representado na figura: A seta indica o sentido da velocidade e o da acelerao dessa caminhonete. Ao passar pelo ponto P, indicado na figura, um passageiro, na carroceria do veculo, lana uma bola para cima, verticalmente em relao a ele. Despreze a resistncia do ar. Considere que, nas alternativas a seguir, a caminhonete est representada em dois instantes consecutivos. Assinale a alternativa em que est MAIS BEM representada a trajetria da bola vista por uma pessoa, parada, no acostamento da estrada. b) parbola d) segmento de reta

17) Trs pedras so atiradas horizontalmente, do alto de um edifcio, tendo suas trajetrias representadas a seguir.

Prof: Sandro Fernandes

13

Admitindo-se a resistncia do ar desprezvel, correto afirmar que, durante a queda, as pedras possuem: a) aceleraes diferentes. b) tempos de queda diferentes. c) componentes horizontais das velocidades constantes. d) componentes verticais das velocidades diferentes, a uma mesma altura. e) carga eltrica diferentes. 18) Os quatro blocos, representados na figura com suas respectivas massas, so abandonados em um plano inclinado que no apresenta atrito e termina voltado para a direo horizontal.

Os blocos, ao deixarem a plataforma, descrevem trajetrias parablicas em queda livre e alcanam o solo, formando, da esquerda para a direita, a seqncia: a) m; 5m; 2m; 3m c) 3m; 2m; 5m; m e) 2m; m; 5m; 3m 19) O homem sempre desafiou ares, buscando realizar um de seus mais antigos desejos: voar. Descobrir um aparelho capaz de lev-lo s alturas representou uma verdadeira obsesso. Um longo caminho foi percorrido at a engenhosidade de Santos Dumont materializar esse sonho. Justamente por voar, o avio caa, j que tudo que sobe, desce. PARAN, "Fsica - Mecnica" - vol. 1 [adapt.] A partir das idias do texto e tambm de seus conhecimentos, assinale a alternativa com o grfico que representa a posio, em funo do tempo, de uma pedra lanada para cima, que, aps 4s, atinge a altura mxima. Despreze a resistncia do ar e considere g=10m/s2. b) m; 2m; 3m; 5m d) 3m; 5m; m; 2m

Prof: Sandro Fernandes

14

20) Suponha que Cebolinha, para vencer a distncia que o separa da outra margem e livrar-se da ira da Mnica, tenha conseguido que sua velocidade de lanamento, de valor 10 m/s, fizesse com a horizontal um ngulo , cujo sen = 0,6 e cos = 0,8. Desprezando-se a resistncia do ar, o intervalo de tempo decorrido entre o instante em que Cebolinha salta e o instante em que atinge o alcance mximo do outro lado

a) 2,0 s

b) 1,8 s

c) 1,6 s

d) 1,2 s

e) 1,6 s

21) Jobim confirma queda do avio da Air France O ministro da Defesa, Nelson Jobim, confirmou nesta tera-feira (2) que os destroos encontrados durante a madrugada no Oceano Atlntico so do Airbus da Air France. De manh, a Aeronutica informou que no poderia confirmar a origem do material. Airbus da Air France desapareceu aps decolar do Rio de Janeiro no domingo (31) em direo a Paris e desapareceu. O vo AF 447 levava 228 pessoas. Jobim disse que no possvel saber se h sobreviventes. Segundo o ministro, o avio Hrcules da Fora Area Brasileira identificou diversos materiais em uma faixa de 5 km. "Fios, metais, enfim, elementos que compem a aeronave", esclareceu. O local fica dentro da rea em torno do arquiplago de So Pedro e So Paulo. www.globo.com(02/06/09, 19h44min)

Prof: Sandro Fernandes

15

O avio Hrcules sobrevoou a regio onde os destroos foram encontrados com velocidade horizontal constante, largando 4 esferas sinalizadoras, em intervalos de tempos iguais, para marcar a rea da possvel queda, j que a noite se aproximava e as buscas deveriam continuar. No caso em questo, a atmosfera na regio estava muito estvel e tranqila de maneira que a RESISTNCIA DO AR PUDESSE SER DESPREZADA. A figura que melhor poderia representar as posies aproximadas do avio e das esferas sinalizadoras, em um mesmo instante, :

22) Com suas descobertas astronmicas, Galileu derrubou uma concepo que dominava a cosmologia desde os tempos de Aristteles, no sculo 4 a.C. O antigo filsofo grego dividira o cosmo em duas regies diferentes. A Terra e suas imediaes seriam formadas por uma mistura varivel de quatro "elementos": terra, gua, ar e fogo. Da estarem sujeitas a mudanas constantes. A partir da rbita da Lua, porm, outro tipo de matria, a nobre "quintessncia", tornava os corpos celestes perfeitos, eternos e imutveis. Antes de Galileu, essa falsa idia foi contestada por filsofos como Nicolau de Cusa (1401-1464) e Giordano Bruno (1548-1600) e astrnomos como Ticho Brahe (1546-1601) e Johannes Kepler (1571-1630). Faltava-lhes, porm, uma prova irrefutvel, que pudesse contrapor enorme autoridade de Aristteles. Foi publicada recentemente em uma revista Britnica uma carta aberta populao na qual um grupo de cosmlogos critica a postura dos defensores do modelo cosmolgico do Big Bang. Os cientistas argumentam que hoje em dia, na cosmologia, no se tolera a dvida e a discordncia. Eles tambm criticam que essa postura totalitria faz com que as observaes astrofsicas sejam interpretadas de modo distorcido. Assim, quando surgem dados observacionais discordantes daquele modelo, em vez o colocarem em cheque, eles so ignorados ou ridicularizados pelos defensores do referido modelo. Com base nessas informaes, conclui-se que esse grupo de cosmlogos est chamando a ateno para o fato de que:

Prof: Sandro Fernandes

16

a) a cincia lida com a realidade ltima, por isso os modelos no podem estar errados e correspondem a essa realidade. b) a cincia lida com modelos, os quais podem estar errados na interpretao da realidade, mesmo quando so aceitos por muitos cientistas. c) a pesquisa cientfica no comete erros ao interpretar a realidade, mesmo quando os cientistas esto em desacordo entre si sobre qual modelo verdadeiro. d) a pesquisa cientfica feita por cientistas imparciais e objetivos, os quais querem encontrar testes observacionais para mostrar que os modelos esto errados. e) a cincia em momento algum lida com modelos, e sim com a realidade ltima, por isso os modelos no so aceitos por esse grupo de cientistas. 23) Este ano alm de comemorarmos 400 anos das primeiras observaes astronmicas feitas por Galileu em 1609 (Ano Internacional da Astronomia), tambm comemoramos em julho, 40 anos da primeira ida do homem lua. O astronauta Neil Armstrong foi o primeiro homem a pisar na superfcie da Lua, em 1969. Na ocasio, realizou uma experincia que consistia em largar, ao mesmo tempo e a partir do repouso, um martelo e uma pena, deixando-os cair sobre a superfcie lunar, e observou que o(s): a) martelo caiu e a pena subiu. b) martelo caiu mais rpido do que a pena. c) dois corpos ficaram flutuando em repouso. d) dois corpos tocaram o solo lunar ao mesmo tempo. e) dois corpos comearam a subir, afastando-se da superfcie lunar. 24) 2009. Ano internacional da astronomia. Ano em que comemoramos 400 das primeiras observaes astronmicas de Galileu. Vero de 1609: um texto curioso chega s mos do matemtico e fsico italiano Galileu Galilei (1564-1642). Era a descrio de um instrumento, construdo na Holanda, que permitia enxergar coisas distantes como se estivessem prximas. Tomando por modelo essa luneta holandesa, Galileu fabrica, ele mesmo, um aparelho semelhante, capaz de aumentar nove vezes o tamanho aparente dos objetos. Tinha, na poca, 45 anos, e, embora desfrutasse de certo prestgio, como professor de matemtica da Universidade de Pdua, no havia publicado ainda nenhum trabalho de peso. Sempre lutando com dificuldades financeiras, precisava dar aulas particulares para complementar o salrio. Mas estava destinado a voar alto. Nos meses seguintes, no parou de aperfeioar o telescpio e apontou-o para o cu. As descobertas que realizou revolucionaram a cosmologia e elevaram sua fama altura das estrelas.

Prof: Sandro Fernandes

17

(...) Depois de longas investigaes, convenci-me por fim de que o Sol uma estrela fixa rodeada de planetas que giram em volta dela e de que ela o centro e a chama. Que, alm dos planetas principais, h outros de segunda ordem que circulam primeiro como satlites em redor dos planetas principais e com estes em redor do Sol. (...) No duvido de que os matemticos sejam da minha opinio, se quiserem dar-se ao trabalho de tomar conhecimento, no superficialmente, mas duma maneira aprofundada, das demonstraes que darei nesta obra. Se alguns homens ligeiros e ignorantes quiserem cometer contra mim o abuso de invocar alguns passos da Escritura (sagrada), a que toram o sentido, desprezarei os seus ataques: as verdades matemticas no devem ser julgadas seno por matemticos. (COPRNICO, N. De Revolutionibus orbium caelestium) Aqueles que se entregam prtica sem cincia so como o navegador que embarca em um navio sem leme nem bssola. Sempre a prtica deve fundamentar-se em boa teoria. Antes de fazer de um caso uma regra geral, experimente-o duas ou trs vezes e verifique se as experincias produzem os mesmos efeitos. Nenhuma investigao humana pode se considerar verdadeira cincia se no passa por demonstraes matemticas. (VINCI, Leonardo da. Carnets.)

O aspecto a ser ressaltado em ambos os textos para exemplificar o racionalismo moderno a) a f como guia das descobertas. b) o senso crtico para se chegar a Deus. c) a limitao da cincia pelos princpios bblicos. d) a importncia da experincia e da observao. e) o princpio da autoridade e da tradio. 25) As grandezas fsicas A e B so medidas, respectivamente, em newtons (N) e em segundos (s). Uma terceira grandeza C, definida pelo produto de A por B, tem dimenso de: a) acelerao. d) momento de fora. b) fora. e) impulso de uma fora. c) trabalho de uma fora.

26) Nas transformaes adiabticas, podemos relacionar a presso p de um gs com o seu volume V atravs da expresso p.VY =K onde y e K so constantes. Para que K tenha dimenso de trabalho, y: a) deve ter dimenso de fora. c) deve ter dimenso de temperatura. e) deve ser adimensional. 27) Uma tcnica muito empregada para medir o valor da acelerao da gravidade local aquela que utiliza um pndulo simples. Para se obter a maior preciso no valor de g deve-se: a) usar uma massa maior. c) medir um nmero maior de perodos. e) fazer vrias medidas com massas diferentes. b) usar um comprimento menor para o fio. d) aumentar a amplitude das oscilaes. b) deve ter dimenso de massa. d) deve ter dimenso de deslocamento.

Prof: Sandro Fernandes

18

28) O velocmetro indica a velocidade instantnea de um veculo. Num certo instante, a indicao do aparelho est representada a seguir.

A MELHOR leitura da velocidade, em km/h a) 80 b) 84 c) 87 d) 90 e) 92

29) Um sistema de radar programado para registrar automaticamente a velocidade de todos os veculos trafegando por uma avenida, onde passam em mdia 300 veculos por hora, sendo 55km/h a mxima velocidade permitida. Um levantamento estatstico dos registros do radar permitiu a elaborao da distribuio percentual de veculos de acordo com sua velocidade aproximada.

A velocidade mdia dos veculos que trafegam nessa avenida de: a) 35 km/h b) 44 km/h c) 55 km/h d) 76 km/h e) 85 km/h

30) Com base neste conhecimento, Galileu, antes mesmo de realizar seu famoso experimento da torre de Pisa, afirmou que uma pedra leve e outra pesada, quando abandonadas livremente de uma mesma altura, deveriam levar o mesmo tempo para chegar ao solo. Tal afirmao um exemplo de:

a) lei
Prof: Sandro Fernandes

b) teoria

c) modelo

d) hiptese

e) frmula

19

De acordo com a Matriz de Referncias para o Enem 2009. Professor: Sandro Fernandes Tema: O Equilbrio e a descoberta das Leis Fsicas

Aula II de Fsica
A Evoluo das Idias
Os fsicos esto interessados nas regularidades que se revelam na observao das coisas e dos fenmenos. Suas teorias s conseguem descrever a enorme complexidade do mundo fsico porque existem certas correlaes entre fenmenos, regularidades, certas propores que convencionamos chamar leis naturais. O trabalho e o esforo dos fsicos consistem em descobrir essas leis e as condies iniciais que permitem encontrar as solues e, atravs das prprias leis, estabelecer predies. A pesquisa do conhecimento atravs da contemplao da variedade das coisas conduziu j na Grcia clssica, noo de necessidade, de proporo entre os elementos, idia da existncia de elementos constitutivos da matria.

A Escola de Mileto
Tales foi um dos primeiros a enunciar a idia da existncia de um elemento fundamental, de uma substncia primordial. Segundo ele, todas as coisas seriam feitas de gua. Como a gua contm tomos de hidrognio, essa concepo no est em contradio com as idias modernas de astrofsica: da observao de material csmico, deduz-se que os elementos predominantes no estgio inicial do Universo eram o hidrognio e o hlio, em uma proporo de abundncia de hidrognio dez vezes superior do hlio. J Anaximandro, outro filsofo da escola de Mileto, afirmava que a substncia primordial de todas as coisas no a gua, nem, efetivamente, nenhum outro corpo material conhecido. Para ele, o elemento fundamental de todas as coisas infinito e eterno e est subjacente em todos os mundos. Essa substncia se transforma em objetos materiais que ns percebemos. Segundo Anaximandro, no mundo material existe uma proporo definida de ar, de fogo, de gua e de terra. A competio entre esses elementos concebidos como deuses, ou seja, a proporo de tais elementos, regulamentada por uma fatalidade, por uma certa necessidade necessidade de proporo entre esses elementos que constituiria, segundo certos filsofos, a origem da noo da lei da natureza. Para Anaxmenes, terceiro pensador da Escola de Mileto, a substncia primordial o ar. A alma do homem feita de ar, o fogo o ar rarefeito; ao condensar, o ar se transforma em gua que, por sua vez, se condensa em terra, em pedras. Segundo essas especulaes, por assim dizer, pioneira da qumica, as foras de coeso seriam uma espcie de respirao: visto que nossa alma, feita de ar, nos mantm unidos e estveis, tambm o ar e a respirao universal asseguram a coeso, a estabilidade do mundo o ar seria substitudo no sculo XIX pelo ter, que transmitiria as aes fsicas.

Prof: Sandro Fernandes

20

Pitgoras
Atribui-se a Pitgoras a origem da palavra teoria: palavra que queria dizer estado de contemplao com afinidade e paixo. Segundo Pitgoras, a contemplao com afinidade e paixo uma atividade intelectual que d origem ao conhecimento matemtico. Devemos a ele a afirmao de que todas as coisas so nmeros, afirmao essa que, depois de Galileu e Newton, incorporou-se fsica e pode ser encontrada nos trabalhos de Maxwell e Lorentz, de Einstein, de Schrdinger e Dirac, assim como no trabalho dos fsicos contemporneos sobre as teorias dos campos. Eis um resumo de um apanhado geral da filosofia dos Pitagricos feito por Alexander Polyhistor no sculo I a.C. e reproduzido por Digenes de Laerta: "O primeiro princpio de todas as coisas o Um. Do Um proveio um Dois indefinido, enquanto Matria para o Um que causa. Do Um e do indefinido Dois provieram os nmeros; dos nmeros, os pontos; dos pontos, as linhas; das linhas, as figuras planas; das figuras planas, as figuras slidas; das figuras slidas, os corpos sensveis. Os elementos deste ltimo so quatro: fogo, gua, terra, ar; esses elementos mudam e se transformam e deles resulta um Cosmo, animado, inteligente, esfrico, que compreende a terra que , ela prpria, esfrica e habitada por todos os lados" (citao de Cornford).

Herclito e Parmnides
Belas divagaes filosficas tambm nos foram legadas por Herclito (sculo V a.C.). Ele considerava o fogo como substncia primordial, visto que ele tem as propriedades da menos corporal e mais sutil matria. Tal a chama do fogo, tudo nasce da morte de algo; diramos hoje: ftons so emitidos (nascem) na aniquilao (morte) eltron-psitron; pares partcula-antipartcula nascem da morte de um fton. Assim, afirmava Herclito, os seres mortais so imortais, os imortais so mortais, um vive a morte do outro e morre a vida de um outro. Com Parmnides de Ela, um pitagrico dissidente, foi introduzida a noo do Um, de um ser substancial eterno e imutvel. Ele rejeitou o postulado de Pitgoras segundo o qual do Um original provm dois e, em seguida, vrios. Eis algumas de suas premissas: 1) O que , , e no pode no ser; o que no , no , e no pode ser. 2) O que , pode ser pensado ou conhecido, expresso ou realmente nomeado; o que no , no o pode. Naturalmente, o ponto fraco do sistema de Parmnides que seus postulados rejeitam o mundo, a variedade das coisas resultante do Um. Essa variedade, assim como nascer, tornar-se, mudana, movimento, , segundo ele, irreal. De sua filosofia restou, entretanto o conceito de substncia fundamental permanente, de uma realidade indestrutvel. Os sucessores de Parmnides deviam restabelecer a questo da realidade das coisas, da pluralidade, do mundo que nos dado por nossas percepes e que, para Parmnides, seria apenas uma iluso, visto que no poderia ser subtrado da unidade. Empdocles admitiu que o Um sempre vrios, visto que seria constitudo de quatro partes, uma mistura de quatro elementos diferentes que podem deslocar-se os quatro elementos de Anaximandro, o fogo, o ar, a gua, e a terra. Esses elementos so eternos, imutveis,
Prof: Sandro Fernandes

21

movem-se uns atravs dos outros assim como para Parmnides, o vazio tambm no existe para Empdocles. Para Anaxgoras, se os elementos no podem ser criados ou perecer, o aparecimento de uma coisa o resultado de uma nova combinao dos quatro elementos, seu desaparecimento resulta de uma dissoluo de uma dada combinao. Assim, Empdocles e Anaxgoras substituram o monismo absoluto de Parmnides por uma pluralidade de elementos permanentes que podem ter movimento e, dessa forma, ocasionar mudanas.

A cosmogonia de Plato
A cosmogonia de Plato est exposta em seu dilogo Timeu: o que permanente, imutvel, adquirido pela inteligncia; o que est em transformao adquirido pelo que ele chama de opinio. Visto que o mundo apreendido por nossas sensaes, ele no pode ser eterno, deve ter sido criado por Deus. Os quatro elementos fogo, ar, gua, terra so representados por nmeros que mantm uma certa proporo entre si. O tempo e o cu foram criados juntos. Mas os verdadeiros elementos primordiais no so os quatro elementos citados acima; so, antes, duas espcies de tringulo retngulo, sendo um a metade de um quadrado e o outro a metade de um tringulo equiltero; essas so as mais belas formas. Devido sua beleza, Deus os utilizou para constituir a matria. Cada tomo de um dos quatro elementos um slido regular (conexo) construdo a partir desses tringulos: os tomos da terra so cubos, os do fogo so tetraedros, os do ar, octaedros, os da gua, icosaedros. O quinto, o dodecaedro, no pode ser construdo pelos dois tringulos de Plato mas sim a partir de pentgonos regulares. Segundo Plato, Deus o utilizou no esquema do Universo que seria, apesar dessa afirmao, esfrico. No Teeteto, Plato critica a concepo segundo a qual o conhecimento a mesma coisa que a percepo. Apenas o pensamento pode nos fazer conhecer o que existe, ou seja, as idias; o conhecimento consiste, portanto, em reflexes, e no, de forma alguma, em impresses e percepes. Em Plato, como em Pitgoras, encontramos, ento, as origens da concepo segundo a qual a matemtica descreve o mundo, uma concepo que ser incorporada na fsica com Galileu.

A fsica de Aristteles
Como sabemos, a fsica e a cosmogonia de Aristteles no contriburam para a cincia moderna. Mas tm uma importncia histrica indubitvel, porque dominaram as especulaes sobre o mundo at Galileu, at o sculo XVII. Segundo Aristteles, existem duas espcies de movimento: o dos corpos terrestres e o dos corpos celestes. O cu consiste em dez esferas concntricas, tendo a esfera da lua o menor raio. No interior dessa esfera, tudo o que est sob a Lua est sujeito corrupo e desintegrao. Fora da esfera da Lua, tudo indestrutvel. O movimento dos corpos terrestres se produz como o dos animais, com uma finalidade. Os corpos celestes, ao contrrio, so caracterizados pela regularidade de seus movimentos, produzidos pela vontade de um Deus. Alm das esferas de Mercrio, de Vnus, do Sol, de Marte, de Jpiter e de Saturno, existe a esfera das estrelas fixas, o Primum Mobile. Alm do Primum Mobile, no h movimento, tempo ou lugares. Deus,
Prof: Sandro Fernandes

22

o Motor Primordial, ele prprio imvel, produz a rotao do Primum Mobile que transmite seu movimento para a esfera das estrelas fixas e esse movimento transmitido at a esfera da Lua: essa a concepo do mundo cristo da Idade Mdia, herdada de Aristteles e apresentada no Paraso de Dante. Quanto fsica de Aristteles, era um corpo terico logicamente coerente e construdo para descrever os movimentos de nossa experincia de todos os dias: um corpo pesado cai para baixo; a chama se move para cima. Segundo Aristteles, acima de tudo, cada corpo tem um lugar determinado no mundo e ope resistncia a qualquer esforo que tende a retir-lo daquele lugar. Da, a idia de movimento como resultado de uma violncia e, uma vez cessada a violncia, os corpos em movimento voltam ao repouso. Em termos modernos, podemos dizer que a dinmica de Aristteles define a fora como sendo a impulso. A equao de movimento de Aristteles a seguinte: Segundo Aristteles, o vazio no existe. No vazio, assim como no espao geomtrico, no existem lugares nem direes privilegiadas. Conseqentemente, as figuras geomtricas no podem descrever os corpos materiais: a fsica no pode ser descrita pela matemtica. Seria at mesmo perigoso, segundo Aristteles, misturar fsica e geometria, aplicar o raciocnio matemtico ao estudo da realidade fsica.

A crtica de Aristteles
Os crticos e os adversrios da dinmica de Aristteles chamavam a ateno para o fato de que o movimento continua, assim que cessou a fora, ao motriz que lhe deu origem. Dentre eles, citemos Jean Phi-lopon, Jean Buridan e Nicole Oresme, da Escola dos Nominalistas de Paris (sculo XIV), Leonardo da Vinci, Benedetti e Galileu (sculos XVI e XVII). Essa crtica produziu a teoria do impetus: ao invs de considerar o ar ao mesmo tempo como resistncia e motor dos movimentos, por que no admitir que alguma coisa transmitida quilo que se move pela ao motriz, alguma coisa que foi, ento, denominada virtus motiva, virtus impressa, impetus, impetus impressus, que faz com que o movimento continue? Durante mil anos, essa noo de impetus permaneceu ambgua e confusa.

A revoluo cristalizada em Galileu


Uma revoluo na concepo fsica do mundo, a formulao de uma nova linguagem e de uma nova filosofia foram necessrias para a ecloso da fsica moderna. A concepo aristotlica e medieval do Cosmo finito, constitudo de um certo nmero de esferas hierarquicamente ordenada teve de ser substituda pela idia de um Cosmo aberto, um Universo infinito. Se no mundo de Aristteles havia lugar para leis aplicveis ao Cu e leis descritivas apenas das coisas da Terra, no novo sistema do mundo existiria apenas um nico tipo de leis, as leis fsicas universais, vlidas em toda a parte.

Prof: Sandro Fernandes

23

O novo sistema do mundo, que adquiriu forma mais precisa a partir de Galileu, estabeleceu, ento, a identificao do espao fsico com o espao infinito da geometria euclidiana, onde e possvel pensar um corpo isolado do resto do Universo, ingrediente do princpio da inrcia. O movimento e o repouso so, ento, considerados como estados em um mesmo nvel existencial, ontolgico. Em linguagem moderna, pode-se expressar a equivalncia ontolgica dos estados de repouso e de movimento retilneo e uniforme dizendo-se que a mecnica clssica admite o grupo de Galileu: j que o repouso no precisa de nenhuma causa para se manter, o mesmo acontece com um movimento retiineo e uniforme que se deduz do estado de repouso pela aplicao de uma transformao desse grupo. Em 1543, Coprnico retirou a Terra de seu repouso abaixo do Paraso e lanou-a ao espao. Entre 1609 e 1619, Kepler formulou as leis de movimento dos corpos celestes, destruindo, portanto, a hierarquia das esferas do Cosmo fechado de Aristteles. E Galileu, observando o Cu com os primeiros telescpios, descobriu novos corpos celestes no previstos no modelo aristotlico preestabelecido por Deus. Descobrindo o princpio da inrcia, assim como a lei da queda livre dos corpos, Galileu abriu o caminho para a grande sntese de Newton e, como Pitgoras e Plato, declarou que o livro da natureza est escrito em linguagem matemtica.

O sistema do mundo newtoniano


A fsica moderna adquiriu, ento, sua primeira forma com o sistema de Newton, em seus Princpios Matemticos da Filosofia Natural. Sua equao do movimento, que estabelece que a fora o produto da massa do corpo por sua acelerao, esteve na base da fsica at a descoberta da mecnica quntica em 1925. Sua lei de gravitao universal foi a intuio de um gnio que completou a tarefa de Galileu assimilando os movimentos dos corpos terrestres aos movimentos dos corpos celestes submetidos a uma mesma fora, a fora de gravitao. O fato de que essa fora fosse transmitida instantaneamente uma ao distncia era certamente um mistrio que inquietou o prprio Newton. Os sucessos da mecnica newtoniana, os trabalhos de pesquisa de homens como Maupertuis, D'Aiembertt, Euler, Lagrange, Laplace, fizeram esquecer a dificuldade de interpretao da fora de gravitao. Segundo Ernest Mach, a atrao gravitacional perdeu seu carter de incompreenso extraordinria para ter apenas uma incompreenso ordinria. No sculo XVIII, graas filosofia de Locke e s cartas filosficas de Voltaire, o newtoniano se tornou o dogma do sistema fsico do mundo. O sistema de Newton incorporou as idias atmicas. Pois, como se pode notar, no mencionei at aqui as geniais intuies dos atomistas gregos do sculo IV a.C., de Leucipo e de Demcrito, influenciados pelo monismo de Parmnides e de Zeno. Talvez, com a preocupao de fazer uma sntese entre os sistemas de Parmnides e de Empdocles, eles postularam que todas as coisas so compostas por tomos que se movem incessantemente no vazio, no espao vazio; que os tomos so indivisveis, que sempre estiveram em movimento e que estaro sempre em movimento. Os atomistas admitiam o determinismo: nada pode acontecer por acaso. O sistema filosfico de Leucipo e de Demcrito foi retomado por Gassendi no comeo do sculo XVII; ele um dos inspiradores da fsica moderna.
Prof: Sandro Fernandes

24

Est claro que essa concepo se associava harmoniosamente ao sistema do mundo de Galileu e Newton, sendo as leis do movimento de Newton responsveis pelo movimento dos tomos. (Devemos destacar o fsico ingls Robert Boyle, que tentou opor o atomismo ao sistema de Galileu-Newton: em vez de ser escrito em linguagem matemtica, o livro da Natureza seria um romance imaginado em termos corpusculares.) Pela primeira vez, um antigo dualismo conceituai, que consistiria na oposio entre as noes de um e de vrios adquire uma forma explcita e precisa do objeto material e de seu movimento e que agora se exprime no dualismo matria-fora.

Inicialmente algumas situaes para se analisar...

1) Imagine uma superfcie horizontal ilimitada. Voc lana horizontalmente um corpo e ele se move ao longo dela. O que ocorre normalmente com o corpo? Por qu? Caso a superfcie seja cada vez mais lisa e se desprezarmos os efeitos da resistncia do ar qual seria a tendncia do corpo? 2) Um corpo preso extremidade de um fio posto a girar pela outra extremidade, num plano horizontal. O que ocorre com o corpo caso o fio arrebente? 3) Pode existir movimento sem que haja fora? Explique. 4) Voc est sentado numa poltrona de um veculo que se desloca com movimento retilneo uniforme. De repente voc lana verticalmente para cima uma bola. Onde ela dever cair? Explique. 5) A bola da figura solta em A (topo de uma rampa). Como se comporta a velocidade da bola no trecho inclinado e no trecho horizontal? Por qu?

6) Considere uma rampa inclinada fixa ligada por um trecho horizontal outra rampa de inclinao varivel. Uma bolinha solta em A (topo da rampa) e percorre o trecho ABC.

C A

Fixa

Varivel B 25

Prof: Sandro Fernandes

Considerando todas as superfcies lisas e desprezando a resistncia do ar, compare as distncias percorridas pela bolinha ao subir a rampa da direita. Explique. Caso a inclinao da rampa seja gradativamente diminuda, como fica afetada a distncia percorrida pela bolinha ao longo dela? Se a rampa ficar alinhada na horizontal com o trecho horizontal e se considerssemos ilimitada, o que ocorre com a bolinha? Explique. 7) Imagine uma pedra sendo jogada num local aonde no existe gravidade. Como ser o movimento da pedra? Explique. 8) Recentemente, o astronauta brasileiro, Cel. Marcos Csar Pontes, esteve em rbita e passou alguns dias na Estao Espacial Internacional (EEI) a 402 km de altitude, onde experimentou um ambiente de microgravidade. O ambiente de microgravidade a condio de quase ausncia de efeitos gravitacionais que encontrada na rbita da Terra. A falta de impacto gravitacional do ambiente espacial provoca perda de massa muscular nos astronautas, uma vez que a resistncia a ser vencida, para mover-se, sempre bem menor do que na Terra. Em relao a este assunto, analise as proposies a seguir. I. No ambiente de microgravidade da EEI, no h acelerao, desta forma no existem foras atuando sobre ela. II. A Terra atrai a EEI com uma fora de mesma direo, mesmo sentido e mesma intensidade da fora com a qual a EEI atrai a Terra. III. Em relao Terra, o Cel. Marcos Csar Pontes, mesmo sem apresentar movimento, pode estar sob a ao de foras. IV. Na Terra, a resistncia, para mover-se e vencer nossa inrcia, maior, face os acentuados efeitos gravitacionais e atritos. Com base na anlise feita: a) II e III so verdadeiras c) I, II e IV so falsas e) IV verdadeira 9) Leia atentamente as afirmativas a seguir e marque a opo CORRETA. I. Se a acelerao de uma partcula for nula, a partcula no pode estar em movimento. II. Se a acelerao de uma partcula tiver mdulo constante, a direo de seu movimento pode variar. III. Se a acelerao de uma partcula for diferente de zero, a partcula pode ter velocidade nula. a) Todas as afirmativas so corretas. b) Apenas a afirmativas I e II so corretas. c) Apenas as afirmativas I e III so corretas. d) Apenas as afirmativas II e III so corretas. e) Todas as afirmativas so falsas.
Prof: Sandro Fernandes

b) I, III e IV so falsas d) apenas I falsa

26

10) Um homem est puxando uma caixa sobre uma superfcie, com velocidade constante, conforme indicado na figura 1. Escolha, dentre as opes a seguir, os vetores que poderiam representar as resultantes das foras que a superfcie exerce na caixa e no homem.

11) "A palavra 'pesado', em latim, 'gravis'. Vem da o termo "mulher grvida". por isso que a fora peso chamada gravitacional". "Aprendendo Fsica 1" - Chiquetto e outros - pg. 136-243. Ed. Scipione. Um jogador chuta a bola: uma das foras aplicada na bola e a outra no p, conforme figura 1. Sattite: uma fora aplicada no satlite e a outra, na Terra, conforme figura 2.

Observe agora a situao a seguir, que envolve a mesma lei fsica presente nos exemplos anteriores. Sobre uma mesa horizontal, repousa um livro de Fsica de 1,2kg de massa. Sobre ele, est um livro de Geografia, tambm em equilbrio, de massa igual a 0,8kg. Considere a acelerao da gravidade na Terra igual a 10m/s2 e, na Lua, aproximadamente um sexto desse valor. Em relao situao apresentada, correto afirmar que a) o mdulo da fora exercida, na Terra, pelo livro de Fsica sobre o de Geografia vale 12N. b) o mdulo da fora exercida, na Terra, pelo livro de Fsica sobre a mesa vale 4N. c) o mdulo da fora exercida, na Terra, pelo livro de Fsica sobre a mesa vale 20N. d) o mdulo da fora exercida, na Lua, pelo livro de Fsica sobre o de Geografia zero. e) o mdulo da fora exercida pelo livro de Fsica sobre o livro de Geografia ser menor na Lua, j que suas massa diminuem.

Prof: Sandro Fernandes

27

12) A brasileira Maria Esther Bueno foi a primeira tenista a se tornar campe de duplas nos quatro torneios mais importantes do mundo (o da Austrlia, o de Wimbledon, o de Roland Garros e o dos Estados Unidos), numa mesma temporada. (http://www.tennisfame.org/enshrinees/maria_bueno.html) Imagine que a tenista consiga golpear a bolinha com sua raquete de modo a faz-la passar sobre a rede e atingir a quadra de sua adversria. Considere as seguintes foras: P - fora vertical para baixo devido gravidade Fr - fora devido raquetada Fa- fora devido presena da atmosfera Assinale a opo que melhor representa as foras, dentre as trs acima, que atuam sobre a bolinha, aps a raquetada.

13) Uma caminhonete sobe uma rampa inclinada com velocidade constante, levando um caixote em sua carroceria, conforme ilustrado na figura a seguir. Sabendo-se que P o peso do caixote, N a fora normal do piso da caminhonete sobre o caixote e f(a) a fora de atrito entre a superfcie inferior do caixote e o piso da caminhonete, o diagrama de corpo livre que melhor representa as foras que atuam sobre o caixote :

Prof: Sandro Fernandes

28

14) comum as embalagens de mercadorias apresentarem a expresso "Peso lquido". O termo lquido sugere que o valor indicado na embalagem corresponde apenas ao seu contedo. Em um pote de mel pode-se ler a frase: "Peso lquido 500g". Nesse sentido, analise quanto coerncia com os sistemas de unidades adotados na Fsica, se as afirmativas a seguir so falsas ou verdadeiras, na medida em que a frase indicada na embalagem: I) est errada, porque o peso uma fora e s pode ser expresso em newtons (N). II) estaria certa, se o peso lquido fosse expresso em gf (grama-fora). III) est certa, porque g o campo gravitacional e P = mg. IV) est errada, porque o peso no pode ser expresso em gramas. Considerando as afirmativas, a combinao correta : a) I e II verdadeiras / III e IV falsas b) I e III falsas / II e IV verdadeiras c) I e IV falsas / II e III verdadeiras d) I, II e III falsas / IV verdadeira e) I, III e IV verdadeiras / II falsa 15) Apesar de Giordano Bruno ter sido levado fogueira em 1600 por sustentar que o espao infinito, Newton (1642-1727) admite essa possibilidade, implicitamente, em algumas de suas leis, cujos enunciados so: I - Na ausncia de resultante de foras, um corpo em repouso continua em repouso e um corpo em movimento mantm-se em movimento retilneo com velocidade constante. II - A acelerao que um corpo adquire diretamente proporcional resultante das foras que atuam nele e tem a mesma direo e o mesmo sentido desta resultante. III - Quando um corpo exerce uma fora sobre outro corpo, este reage sobre o primeiro com uma fora de mesmo mdulo, mesma direo e sentido oposto. IV - Dois corpos quaisquer se atraem com uma fora proporcional ao produto de suas massas e inversamente proporcional ao quadrado da distncia entre eles. As leis que, implicitamente, pressupem a existncia do espao infinito so: a) I e III b) I e IV c) II e III d) II e IV e) I e II

16) Uma pilha de seis blocos iguais, de mesma massa m, repousa sobre o piso de um elevador, como mostra a figura. O elevador est subindo em movimento uniformemente retardado com uma acelerao de mdulo a. O mdulo da fora que o bloco 3 exerce sobre o bloco 2 dado por

Prof: Sandro Fernandes

29

a) 3m (g + a).

b) 3m (g - a).

c) 2m (g + a).

d) 2m (g - a). e) 3m(a - g)

17) "O mais audacioso passo da aeronutica (e astronutica) brasileira desde Santos-Dumont."

Coincidentemente, em 2006, comemoramos os 100 anos do histrico vo de Alberto Santos Dumont (1873-1932) com o 14-Bis. Em 23 de outubro de 1906, ele voou cerca de 60 m a uma altura de 2 a 3 metros, no Campo de Bagatelle em Paris. Por este feito, Santos Dumont considerado por parte da comunidade cientfica e da aeronutica, e principalmente em seu pas de origem, o Brasil, como o "Pai da Aviao". Cem anos depois, outro brasileiro entra para a histria. Marcos Csar Pontes, em 2006, tornou-se o primeiro astronauta brasileiro a participar de uma misso na Estao Espacial Internacional (EEI), denominada "misso centenrio". Com base nestas informaes, INCORRETO afirmar: a) O princpio bsico, tanto para a propulso de foguetes quanto para o vo de um avio a jato, a terceira lei de Newton. b) Comentou-se muito na imprensa que a gravidade no espao zero. Isso uma contradio, pois ela que mantm a EEI "presa" Terra. c) A magnitude do empuxo do foguete no lanamento depende da variao temporal da sua quantidade de movimento. d) Um astronauta verificaria que, na EEI (g = 8,6 m/s2), a parte submersa de um mesmo cubo de gelo em um copo seria maior que na Terra. 18) Um sistema de massas, que se encontra sob a ao da gravidade terrestre, formado por duas esferas homogneas, X e Y, cujos centros esto afastados 0,8 m um do outro. A esfera X tem massa de 5 kg, e a esfera Y tem massa de 3 kg. A que distncia do centro da esfera X se localiza o centro de gravidade do sistema? a) A 0,2 m. b) A 0,3 m. c) A 0,4 m. d) A 0,5 m. e) A 0,6 m.

Prof: Sandro Fernandes

30

19) Os grficos a seguir representam as velocidades, em funo do tempo, de dois objetos esfricos homogneos idnticos, que colidem frontalmente. Se p a quantidade de movimento do sistema formado pelos dois objetos e E a energia cintica deste mesmo sistema, podemos afirmar que na coliso:

a) p se conservou e E no se conservou. b) p se conservou e E se conservou. c) p no se conservou e E se conservou. d) p no se conservou e E no se conservou. e) (p +E) se conservou. 20) Um projtil de 450g disparado horizontalmente com velocidade 20 1/2 m/s, contra um corpo de massa 0,45kg suspenso por um fio de 2m de comprimento. Em um choque perfeitamente elstico e frontal, o corpo sobe at uma altura h. Qual o ngulo mximo formado pelo fio com a vertical?

a) 30

b) 45

c) 60

d) 75

e) 90

21) Trs blocos cbicos iguais esto empilhados, conforme sugere a figura. Nestas condies, a mxima distncia x, para que ainda se tenha equilbrio, :

a) a/2
Prof: Sandro Fernandes

b) (3/4)a

c) (7/8)a

d) (11/12)a

e) a

31

22) Pretendendo-se arrancar um prego com um martelo, conforme mostra a figura, qual das foras indicadas (todas elas tm o mesmo mdulo), ser mais eficiente, na posio considerada?

23) Quando algum objeto cai dentro da gua contida no vaso sanitrio, imediatamente, o sifo se encarrega de reestabelecer o nvel da gua, permitindo que parte dela transborde para o esgoto.

Considerando uma situao de equilbrio entre a gua do vaso sanitrio e um objeto slido que nela foi depositado suavemente, analise: I. Flutuando parcialmente ou permanecendo completamente mergulhado, qualquer slido dentro da gua do vaso sanitrio est sujeito a uma fora resultante vertical voltada para cima. II. Independentemente de o corpo flutuar ou no, a fora de empuxo tem intensidade igual do peso do lquido derramado para o esgoto. III. Um objeto que afunde completamente tem seu peso maior que o empuxo que recebe e densidade maior que a densidade da gua. IV. Quando um objeto afunda totalmente na gua, pode-se concluir que o peso do lquido que escorre pelo sifo igual ao peso do objeto. Est correto o contido em a) I e II, apenas. d) III e IV, apenas. b) I e IV, apenas. e) I, II, III e IV. c) II e III, apenas.

Prof: Sandro Fernandes

32

24) Um cubo slido e macio, preso na extremidade de um fio fino e inextensvel, est totalmente mergulhado em um lqido, como mostra a figura a seguir.

Sabendo-se que a densidade do cubo maior que a densidade do lqido, considere as afirmativas a seguir. I - O empuxo que o lqido exerce sobre o cubo igual ao peso do lqido deslocado pelo cubo. II - O peso do cubo igual ao empuxo que o lqido exerce sobre ele. III - O empuxo sobre o cubo igual diferena entre as foras de presso, exercida pelo lqido, nas faces inferior e superior do cubo. Sobre essas afirmativas, pode-se afirmar que a) somente I correta. c) I e III so corretas. e) Todas esto erradas. 25) Um brao mecnico de um trator usado para fazer valetas tem um sistema hidrulico que se compe, basicamente, de dois cilindros conectados por uma mangueira resistente a altas presses, todos preenchidos com leo. Se, no equilbrio, P a presso num cilindro, a presso no outro, que tem rea 10 vezes maior, a) 10 P b) 5 P c) P d) P/5 e) P/10 b) I e II so corretas. d) todas so corretas.

26) A figura representa em plano vertical um trecho dos trilhos de uma montanha russa na qual um carrinho est prestes a realizar uma curva. Despreze atritos, considere a massa total dos ocupantes e do carrinho igual a 500 kg e a mxima velocidade com que o carrinho consegue realizar a curva sem perder contato com os trilhos igual a 36 km/h. O raio da curva, considerada circular, , em metros, igual a

a) 3,6
Prof: Sandro Fernandes

b) 18

c) 1,0

d) 6,0

e) 10

33

27) A figura a seguir representa um pndulo cnico ideal que consiste em uma pequena esfera suspensa a um ponto fixo por meio de um cordo de massa desprezvel.

Para um observador inercial, o perodo de rotao da esfera, em sua rbita circular, constante. Para o mesmo observador, a resultante das foras exercidas sobre a esfera aponta a) verticalmente para cima. b) verticalmente para baixo. c) tangencialmente no sentido do movimento. d) para o ponto fixo. e) para o centro da rbita. 28) Um caminho, com massa total de 10.000kg est percorrendo uma curva circular plana e horizontal a 72km/h (ou seja, 20m/s) quando encontra uma mancha de leo na pista e perde completamente a aderncia. O caminho encosta ento no muro lateral que acompanha a curva que o mantm em trajetria circular de raio igual a 90m. O coeficiente de atrito entre o caminho e o muro vale 0,3. Podemos afirmar que, ao encostar no muro, o caminho comea a perder velocidade razo de, em m/s2, aproximadamente, a) 0,07 b) 1,3 c) 3,0 d) 10 e) 67

29) Referindo-se estrutura fsica, uma das causas importantes da degradao do solo na agricultura a sua compactao por efeito das mquinas e da chuva. Um trator tem rodas de grande dimetro e largura para que exera contra o solo, pequeno(a) a) presso. b) fora. c) peso. d) energia. e) atrito.

30) Uma pistola dispara um projtil contra um saco de areia que se encontra em repouso, suspenso a uma estrutura que o deixa plenamente livre para se mover. O projtil fica alojado na areia. Logo aps o impacto, o sistema formado pelo saco de areia e o projtil move-se na mesma direo do disparo com velocidade de mdulo igual a 0,25 m/s. Sabe-se que a relao entre as massas do projtil e do saco de areia de 1/999. Qual o mdulo da velocidade com que o projtil atingiu o alvo? a) 25 m/s. b) 100 m/s. c) 250 m/s. d) 999 m/s. e) 1000 m/s.

Prof: Sandro Fernandes

34

De acordo com a Matriz de Referncias para o Enem 2009. Professor: Sandro Fernandes Tema: A Mecnica e o Funcionamento do Universo

Aula III de Fsica


(I) Dos Mitos ao Big Bang.
Mitos - Os mitos cosmognicos, foram utilizados por todos os povos, que buscaram encontrar respostas para os questionamentos de quem somos? De onde viemos? Para onde vamos? Para isso, tentou-se explicar a origem do mundo ou do universo, mas para que chegassem ao resultado, criaram mitos, aos quais conseguiram dar respostas a esses questionamentos. Os mitos conseguiram explicar no somente a origem do universo como tambm dos animais, plantas e outros seres animados e inanimados. Viso Pr Socrtica de origem do universo - Os pr-socrticos so filsofos que viveram na Grcia Antiga e nas suas colnias. Assim so chamados, pois so os que vieram antes de Scrates, considerado um divisor de guas na filosofia. Muito pouco de suas obras est disponvel, restando apenas fragmentos e/ou doxografia. Frente ao pequeno nmero de informaes sobre esses filsofos, qualquer tentativa de descrever seu pensamento ser apenas uma tentativa. So chamados de filsofos da natureza, pois investigaram questes pertinentes a esta, como de que feito o mundo, e ainda romperam com a viso mtica e religiosa da natureza que prevalecia na poca, adotando uma forma cientfica de pensar. Alguns se propuseram a explicar as transformaes da natureza apresentavam ainda uma grande preocupao cosmolgica. Origem do universo segundo Timeu - Segundo o pensamento de Timeu sobre a origem do universo, "o criador" ou "arteso do universo" lhe deu um movimento circular, em torno do prprio centro, por ser este o movimento mais perfeito. A idia de que a forma esfrica e o movimento circular so os mais perfeitos e os nicos adequados para a constituio do universo, teve enorme influncia durante sculos. Tal qual os Pitagricos, Timeu admitia que tudo teria sido criado por leis matemticas, inclusive colocando a msica entre as matemticas e ainda admite um valor, a esse conhecimento, muito grande para a compreenso da origem do universo
"j que a alma do mundo teria sido criada da combinao entre a substncia do indivisvel que, sempre a mesma, e do divisvel que nasce nos corpos, comps a terceira" (...) Depois de aprestar uma unidade a estes trs elementos, dividiu-a em tantas partes quantas era conveniente haver, cada uma constante de uma liga do Mesmo, do Outro e da Existncia".

Origem do Universo Segundo os Relatos Bblicos - No podemos negar a influencia da Bblia sob a viso de origem do universo dos pensadores ocidentais. O livro bblico do gnesis, fala sobre a origem do universo, que podemos dividir em duas trades que esto relacionadas entre si. Na primeira trade so criados: no primeiro dia a luz; no segundo dia o cu e guas; e no terceiro dia a terra (a parte seca) e os vegetais. Na segunda trade so criados: no quarto dia os luzeiros (o Sol, a Lua e as estrelas); no quinto dia as aves e os peixes; e no sexto dia os animais e os seres humanos. Observe que na primeira trade se relaciona com a segunda para se completar: Luz luzeiros; cu e guas aves e peixes; e terra e vegetais animais e seres humanos.
Prof: Sandro Fernandes

35

Origem do Universo Segundo Descartes - A Cosmo viso de Descartes, ou seja, sua viso integrada do mundo inanimado, do reino animal e do homem, deveria ser publicada na obra chamada de Tratado do mundo e da luz, porm pouco antes de public-la, soube que Galileu havia sido condenado por admitir o movimento da Terra, idia, alis, que era compartilhada por Descartes. Na teoria de Descartes sobre a origem do universo Deus tem um papel fundamental em sua criao. Sua contribuio relevante e ocorreria na criao da matria inicial e em seu movimento. Todo o restante ocorreria como conseqncia das leis naturais, que tambm teriam sido criadas por Ele. Inicialmente Descartes imagina o universo como um espao totalmente preenchido por uma matria homognea, slida, como um imenso bloco de cristal. Essa matria foi agitada por Deus, de modo desordenado, em todas as direes, fragmentando-a em pequenos blocos. Origem do Universo Segundo o Big Bang - O TERMO Big Bang foi utilizado pela primeira vez, pelo astrnomo ingls Fred Hoyle, em uma srie de conferncias sobre astronomia na BBC de Londres, no ano de 1915. O termo se traduzido de forma literal dar a idia de uma grande exploso, essa teoria a respeito da origem do universo possui grande aceitao nos meios cientficos. O Big Bang uma teoria cosmolgica segundo a qual toda a matria do universo, em seu estado inicial, se apresentava bastante condensada. Segundo esse modelo a matria sofreu uma violenta exploso, dando origem a tudo o que existe hoje no espao e no tempo. O processo poderia ter se dado devido a algumas condies: Primeiro pela reduo da temperatura pela expanso do espao e, conseqentemente, diminuio da densidade o que foram condies necessrias para o surgimento dos materiais mais leves da natureza. (Hidrognio, Hlio, ftons, quarks e radiao).
http://www.sbf1.sbfisica.org.br/eventos/snef

(II)

Newton a Lei da Gravitao e o efeito das Mars

Newton, aps chegar expresso da fora gravitacional, a usou para realizar estudos e interpretar uma variedade de fenmenos que ocorrem na natureza como, por exemplo, as mars. Muitos dos fenmenos que ele estudou j eram conhecidos, s no havia uma explicao cientfica para eles. O sucesso que Newton obteve na explicao desses fenmenos constituiu um grande triunfo para a teoria da Gravitao Universal. A forma matemtica da Lei da Gravitao Universal mostra como calcular o mdulo da fora de atrao gravitacional entre duas massas.

G.

M1.M 2 d2

A mar um dos fenmenos naturais mais conhecidos. Esse fenmeno ocorre em razo do movimento peridico de subida e descida do nvel da gua, produzindo dessa maneira as chamadas mars altas e mars baixas. Foi Isaac Newton que, a partir da expresso da fora gravitacional, deu a explicao para esse fenmeno natural. Segundo as explicaes do fsico e matemtico Newton, as mars so causadas pela atrao do Sol e da Lua e da Lua sobre as guas do mar.

Prof: Sandro Fernandes

36

As Foras que atuam sobre as mars ocorrem porque a Terra um corpo extenso e que o campo gravitacional que produzido pelo Sol ou pela Lua no homogneo em todos os pontos, pois tem alguns pontos da Terra que esto mais prximos e outros mais distantes destes corpos celestes. Esses campos gravitacionais provocam aceleraes que atuam na superfcie terrestre com diferentes intensidades. Dessa forma as massas de gua que esto mais prximas da Lua ou do Sol sofrem acelerao com intensidades maiores que as massas de gua que esto mais afastadas desses astros. essa diferena de pontos mais prximos e mais afastados do Sol e da Lua que do origem s mars

(III)

Kepler e a Harmonia dos Mundos

Afirmar hoje que a rbita dos planetas descreve uma elipse ou que seu movimento no uniforme no nenhuma novidade - apenas enunciar duas leis bsicas da astronomia moderna. No era bem assim na poca em que Johannes Kepler (1571-1630) fez suas descobertas. Numa Europa convulsionada pelos conflitos da Contra-Reforma, o melhor era no proclamar nada em voz alta, mesmo que dcadas antes Coprnico j tivesse previsto que a Terra girava em torno do Sol. Primeira Lei de Kepler ou Lei das rbitas Os planetas descrevem rbitas elpticas em torno do Sol, que ocupa um dos focos da elipse descrita.

Segunda Lei de Kepler ou Lei das reas O segmento imaginrio que une o centro do Sol e o centro do planeta varre reas proporcionais aos intervalos de tempo dos percursos.

Prof: Sandro Fernandes

37

Terceira Lei de Kepler ou Lei dos Perodos O quadrado do perodo de revoluo de cada planeta proporcional ao cubo do raio mdio da respectiva rbita.

T2 R3

Algumas aplicaes...
1) Um eclipse ocorre quando um astro ocultado, total ou parcialmente, por um outro astro que se interpe entre ele e um observador. O eclipse entre o Sol, a Lua e a Terra pode ser solar ou lunar, conforme a posio relativa entre eles. Na figura a seguir representamos as fases da Lua (posies: A, B, C, D), a Terra e um feixe de luz solar.

Considerando a Terra como referencial, analise as afirmativas a seguir. I - A fase da Lua cheia quando ela se encontra na posio A. II - Quando ocorre o eclipse do Sol, a Lua encontra-se na posio C. III - Durante um eclipse lunar, a Lua encontra-se na posio A. Sobre essas afirmativas, pode-se afirmar que a) todas so corretas. c) apenas II e III so corretas. e) apenas a I correta. 2) Analise as afirmativas sobre a gravitao universal. I - Os planetas descrevem rbitas elpticas ao redor do sol, que ocupa um dos focos da elipse. II - O peso de um corpo diminui quando ele afastado da superfcie da Terra. III - A velocidade de translao de um planeta aumenta quando ele se afasta do sol. Sobre essas afirmativas correto afirmar que
Prof: Sandro Fernandes

b) apenas I e II so corretas. d) todas so incorretas.

38

a) todas so verdadeiras. c) apenas I e II so verdadeiras. e) apenas a I verdadeira.

b) todas so falsas. d) apenas I e III so verdadeiras.

3) Observe o gabarito com a resoluo de uma cruzadinha temtica em uma revista de passatempo. HORIZONTAIS 1. Fora presente na trajetria circular. 2. Astrnomo alemo adepto ao heliocentrismo. 3. Ponto mais prximo ao Sol no movimento de translao da Terra. VERTICAIS 1. rbita que um planeta descreve em torno do Sol. 2. Atrao do Sol sobre os planetas. 3. Lugar geomtrico ocupado pelo Sol na trajetria planetria.

Um leitor, indignado com o "furo" na elaborao e reviso da cruzadinha, em uma carta aos editores, destacou, baseando-se nas leis da Mecnica Clssica, a ocorrncia de erro a) na vertical 2, apenas. b) na horizontal 1, apenas. c) nas verticais 1 e 2, apenas. d) nas horizontais 1 e 3, apenas. e) na horizontal 3 e na vertical 3, apenas. 4) Em 1973, o Pink Floyd, uma famosa banda do cenrio musical, publicou seu disco "The Dark Side of the Moon", cujo ttulo pode ser traduzido como "O Lado Escuro da Lua". Este ttulo est relacionado ao fato de a Lua mostrar apenas uma de suas faces para ns, os seres humanos. Este fato ocorre porque a) os perodos de translao da Lua e da Terra em torno do Sol so iguais. b) o perodo de rotao da Lua em torno do prprio eixo igual ao perodo de rotao da Terra em torno de seu eixo. c) o perodo de rotao da Lua em torno do prprio eixo igual ao seu perodo de translao em torno da Terra. d) o perodo de translao da Lua em torno da Terra igual ao perodo de rotao desta em relao ao seu prprio eixo. e) a luz do Sol no incide sobre o "lado escuro" da Lua.
Prof: Sandro Fernandes

39

5) A sonda Galileo terminou sua tarefa de capturar imagens do planeta Jpiter quando, em 29 de setembro deste ano, foi lanada em direo ao planeta depois de orbit-lo por um intervalo de tempo correspondente a 8 anos terrestres. Considerando que Jpiter est cerca de 5 vezes mais afastado do Sol do que a Terra, correto afirmar que, nesse intervalo de tempo, Jpiter completou, em torno do Sol, a) cerca de 1,6 volta. c) aproximadamente 8 voltas. e) aproximadamente 3/4 de volta. 6) Um planeta gira, em rbita elptica, em torno do Sol. Considere as afirmaes: b) menos de meia volta. d) aproximadamente 11 voltas.

I - Na posio A, a quantidade de movimento linear do planeta tem mdulo mximo. II - Na posio C, a energia potencial do sistema (Sol+planeta) mxima. III - Na posio B, a energia total do sistema (Sol+planeta) tem um valor intermedirio, situado entre os correspondentes valores em A e C. Assinale a alternativa correta. a) I e III so verdadeiras. c) II e III so verdadeiras. e) Apenas I verdadeira. 7) As telecomunicaes atuais dependem progressivamente do uso de satlites geo-estacionrios. A respeito desses satlites, correto dizer que a) seus planos orbitais podem ser quaisquer. b) todos se encontram mesma altura em relao ao nvel do mar. c) a altura em relao ao nvel do mar depende da massa do satlite. d) os que servem os pases do hemisfrio norte esto verticalmente acima do Plo Norte. e) se mantm no espao devido energia solar. 8) A tabela abaixo resume alguns dados importantes sobre os satlites de Jpiter Ao observar os satlites de Jpiter pela primeira vez, Galileu Galilei fez diversas anotaes e tirou importantes concluses sobre a estrutura de nosso universo. A figura abaixo da tabela reproduz uma anotao de Galileu referente a Jpiter e seus satlites. b) I e II so verdadeiras. d) Apenas II verdadeira.

Prof: Sandro Fernandes

40

De acordo com essa representao e com os dados da tabela, os pontos indicados por 1, 2, 3 e 4 correspondem, respectivamente, a: a) Io, Europa, Ganimedes e Calisto. c) Europa, Calisto, Ganimedes e lo. e) Calisto, lo, Europa e Ganimedes. 9) Durante cerca de oito dias, um astronauta brasileiro dividiu com astronautas estrangeiros uma misso a bordo da Estao Espacial Internacional (EEI). Inmeras fotografias da parte interna da Estao mostraram objetos e os astronautas "flutuando" no seu interior. Este fenmeno ocorre porque I. a acelerao da gravidade sobre eles zero. II. os objetos e os astronautas tm a mesma acelerao da Estao. III. no h fora resultante sobre eles. Pela anlise das afirmativas conclui-se que somente est / esto correta(s) a) a I. b) a II. c) a III. d) a I e a III. e) a II e a III. b) Ganimedes, lo, Europa e Calisto. d) Calisto, Ganimedes, lo e Europa.

10) O diagrama da figura 1 representa duas pequenas esferas, separadas entre si por certa distncia. As setas representam as foras gravitacionais que as esferas exercem entre si. A figura 2 mostra cinco diagramas, representado possibilidades de alterao daquelas foras, quando a distncia entre as esferas modificada.

Segundo a Lei da Gravitao Universal, qual dos diagramas da figura 2 coerente com o diagrama da figura 1? a) I. b) II. c) III. d) IV. e) V.

Prof: Sandro Fernandes

41

11) Depois de anos de interrupo, ocorreu neste ano (2005) a retomada de lanamentos do nibus espacial pela NASA, desta vez com sucesso. Nas imagens divulgadas do dia-a-dia no nibus espacial girando ao redor da Terra, pudemos ver os astronautas realizando suas atividades, tanto fora da nave como no seu interior. Considerando que as rbitas da nave e dos astronautas sejam circulares, analise as afirmaes seguintes. I. No h trabalho realizado pela fora gravitacional para manter um astronauta em rbita ao redor da Terra. II. A acelerao de um astronauta girando ao redor da Terra deve-se exclusivamente ao da fora gravitacional. III. A velocidade vetorial do astronauta ao redor da Terra constante. Esto corretas as afirmaes: a) II, somente. d) II e III, somente. b) III, somente. e) I, II e III. c) I e II, somente.

12) O subir e descer das mars regulado por vrios fatores, sendo o principal deles a atrao gravitacional entre Terra e Lua. Se desprezssemos os demais fatores, teramos sempre o intervalo de 12,4 horas entre duas mars altas consecutivas, e tambm sempre a mesma altura mxima de mar, por exemplo, 1,5 metros. Nessa situao, o grfico da funo que relacionaria tempo (t) e altura de mar (A) seria semelhante a este:

O fato do intervalo de tempo entre duas mars altas sucessivas ser de 12,4 horas e no de 12 horas exatas explica-se pelo fato de que a) o perodo de rotao da Terra em torno de seu eixo no de 24 horas, e sim de 24,8 horas. b) a Lua gira em torno da Terra completando uma volta em, aproximadamente, 28 dias. c) a gua do mar tem uma inrcia muito grande que atrasa seu movimento. d) a rbita da Terra em torno do Sol elptica. e) o eixo de rotao da Terra inclinado.

Prof: Sandro Fernandes

42

13) Todas as diferentes foras que se observam na natureza podem ser explicadas em termos de quatro interaes bsicas das partculas elementares: 1. a fora gravitacional 2. a fora eletromagntica 3. a fora nuclear forte 4. a fora nuclear fraca As foras observadas na vida diria entre os corpos macroscpicos se devem ou fora gravitacional ou fora eletromagntica. Ambas comportam-se segundo a lei do inverso do quadrado da distncia entre os corpos que interagem. (Adaptado de Paul Tipler. "Fsica". v. 1. Rio de Janeiro: LTC. p.83) Um pequeno m atrai um prego colocado a uma distncia x com uma fora F cujo mdulo inversamente proporcional ao quadrado de x. Isso significa que, quando se duplicar a distncia x, o valor da fora magntica F passar a ser a) quatro vezes menor. c) a mesma. e) quatro vezes maior. 14) Sabe-se que o peso de um corpo na superfcie da Terra (considerada como esfrica e de raio R) o resultado da interao entre as massas da Terra e do corpo. Para que a fora de interao entre a Terra e o corpo seja metade do seu peso, a distncia d, do corpo ao centro da Terra dever ser de a) 4 R. b) 2 R. c) R/2. d) R.21/2. e) R.31/2. b) duas vezes menor. d) duas vezes maior.

15) Imagine que, no final deste sculo XXI, os habitantes da Lua vivam em um grande complexo pressurizado, em condies equivalentes s da Terra, tendo como nica diferena a acelerao da gravidade, que menor na Lua. Considere as situaes imaginadas bem como as possveis descries de seus resultados, se realizadas dentro desse complexo, na Lua: I. Ao saltar, atinge-se uma altura maior do que quando o salto realizado na Terra. II. Se uma bola est boiando em uma piscina, essa bola manter maior volume fora da gua do que quando a experincia realizada na Terra. III. Em pista horizontal, um carro, com velocidade V 0, consegue parar completamente em uma distncia maior do que quando o carro freado na Terra. Assim, pode-se afirmar que esto corretos apenas os resultados propostos em a) I b) I e II c) I e III d) II e III e) I, II e III

16) Considere duas massas puntiformes sob ao da fora gravitacional mtua. Assinale a alternativa que contm a melhor representao grfica da variao do mdulo da fora gravitacional sobre uma das massas, em funo da distncia entre ambas.

Prof: Sandro Fernandes

43

17) Uma sonda espacial aproxima-se de um corpo celeste desconhecido, em repouso em relao a um referencial inercial mantendo uma velocidade de 90km/h. Considere que, a partir do ponto P, a sonda est sujeita ao campo gravitacional do planeta e entra em rbita circular, conforme a figura adiante. Caso o mdulo da velocidade da sonda seja menor do que 90km/h, a figura que mostra o que dever acontecer com a trajetria da sonda ao entrar no campo gravitacional : Despreze possveis efeitos atmosfricos e suponha que o campo gravitacional do corpo celeste atuar a partir do ponto P.

18) Sabe-se que a atrao gravitacional da lua sobre a camada de gua a principal responsvel pelo aparecimento de mars ocenicas na Terra. A figura mostra a Terra, supostamente esfrica, homogeneamente recoberta por uma camada de gua.

Nessas condies, considere as seguintes afirmativas: I. As massas de gua prximas das regies A e B experimentam mars altas simultaneamente. II. As massas de gua prximas das regies A e B experimentam mars opostas, isto , quando A tem mar alta, B tem mar baixa e vice-versa. III. Durante o intervalo de tempo de um dia ocorrem duas mars altas e duas mars baixas.

Prof: Sandro Fernandes

44

Ento, est(o) correta(s), apenas a) a afirmativa I d) as afirmativas I e II b) a afirmativa II e) as afirmativas I e III c) a afirmativa III

19) O Pequeno Prncipe, do livro de mesmo nome, de Antoine de Saint-Exupry, vive em um asteride pouco maior que esse personagem, que tem a altura de uma criana terrestre. Em certo ponto desse asteride, existe uma rosa, como ilustrado nesta figura:

Aps observar essa figura, Jlia formula as seguintes hipteses: I) O Pequeno Prncipe no pode ficar de p ao lado da rosa, porque o mdulo da fora gravitacional menor que o mdulo do peso do personagem. II) Se a massa desse asteride for igual da Terra, uma pedra solta pelo Pequeno Prncipe chegar ao solo antes de uma que solta na Terra, da mesma altura. Analisando-se essas hipteses, pode-se concluir que a) apenas a I est correta. c) as duas esto corretas. b) apenas a II est correta. d) nenhuma das duas est correta.

20) Selecione a alternativa que preenche corretamente as lacunas nas afirmaes a seguir, na ordem em que elas aparecem. - ........ descreveu movimentos acelerados sobre um plano inclinado e estudou os efeitos da gravidade terrestre local sobre tais movimentos. - ......... usando dados coletados por Tycho Brahe, elaborou enunciados concisos para descrever os movimentos dos planetas em suas rbitas em torno do Sol. - ........ props uma teoria que explica o movimento dos corpos celestes, segundo a qual a gravidade terrestre atinge a Lua, assim como a gravidade solar se estende Terra e aos demais planetas. a) Newton - Kepler Galileu c) Galileu - Newton Kepler e) Kepler - Galileu Newton b) Galileu - Kepler - Newton d) Kepler - Newton - Galileu

Prof: Sandro Fernandes

45

21) Um satlite de massa m, usado para comunicaes, encontra-se estacionrio a uma altura h de um ponto da superfcie do planeta Terra, de massa MT, cujo raio RT. Com base nesses dados, assinale falsa (F) ou verdadeira (V) em cada uma das alternativas, considerando G a constante de gravitao universal.

A seqncia correta a) V - V - F - F. d) F - V - V - V. b) V - V - V - F. e) F - F - V - F. c) F - V - F - V.

22) Segundo a lei da gravitao universal de Newton, a fora gravitacional entre dois corpos diretamente proporcional ao produto de suas massas e inversamente proporcional ao quadrado da distncia entre seus centros de gravidade. Mesmo que no seja obrigatoriamente conhecido pelos artistas, possvel identificar o conceito bsico dessa lei na seguinte citao: a) "Trate a natureza em termos do cilindro, da esfera e do cone, todos em perspectiva" (Paul Czanne) b) "Hoje, a beleza o nico meio que nos manifesta puramente a fora universal que todas as coisas contm." (Piet Mondrian) c) Na natureza jamais vemos coisa alguma isolada, mas tudo sempre em conexo com algo que lhe est diante, ao lado, abaixo ou acima." (Goethe) d) "Ocorre na natureza alguma coisa semelhante ao que acontece na msica de Wagner, que embora tocada por uma grande orquestra, intimista." (Van Gogh) e) A vida feita de certezas e possibilidades. (Sandro Fernandes)

23) O raio do horizonte de eventos de um buraco negro corresponde esfera dentro da qual nada, nem mesmo a luz, escapa da atrao gravitacional por ele exercida. Por coincidncia, esse raio pode ser calculado no-relativisticamente como o raio para o qual a velocidade de escape igual velocidade da luz. Qual deve ser o raio do horizonte de eventos de um buraco negro com uma massa igual massa da Terra? a) 9 m. b) 9 mm. c) 30 cm. d) 90 cm. e) 3 km.

Prof: Sandro Fernandes

46

24) Uma pessoa relacionou abaixo os seguintes fenmenos naturais observados no nosso planeta: I - movimento das mars II - chuva III - terremoto IV - relmpago O(s) fenmeno(s) afetado(s) diretamente pela posio da Lua em relao Terra (so): a) apenas III. b) apenas I. c) apenas I e III. d) apenas II e III. e) apenas I e IV. 25) O perodo de oscilao de um pndulo simples pode ser calculado por T=2(L/g)1/2, onde L o comprimento do pndulo e g a acelerao da gravidade (ou campo gravitacional) do local onde o pndulo se encontra. Um relgio de pndulo marca, na Terra, a hora exata. correto afirmar que, se este relgio for levado para a Lua, a) atrasar, pois o campo gravitacional lunar diferente do terrestre. b) no haver alterao no perodo de seu pndulo, pois o tempo na Lua passa da mesma maneira que na Terra. c) seu comportamento imprevisvel, sem o conhecimento de sua massa. d) adiantar, pois o campo gravitacional lunar diferente do terrestre. e) no haver alterao no seu perodo, pois o campo gravitacional lunar igual ao campo gravitacional terrestre.

Prof: Sandro Fernandes

47

De acordo com a Matriz de Referncias para o Enem 2009. Professor: Sandro Fernandes Tema: O Calor e os Fenmenos Trmicos

Aula IV de Fsica
(I) Do calrico a termodinmica moderna.
Tudo comeou no sculo XII. Avanou at o sculo XVI. Foi uma devastao total nas florestas inglesas. A lenha era usada para aquecer as casas no inverno e tambm para as indstrias que comeavam a se instalar. No sculo XVII j no havia mais lenha suficiente. Os ingleses recorreram ao carvo de pedra. E aqui comea um importante captulo da histria Fsica. As minas de carvo de ento eram superficiais, mas, com o esgotamento dessas jazidas, comearam a abrir buracos e galerias cada vez mais profundos. Essas minas freqentemente ficavam inundadas de gua (devido aos lenis subterrneos) e era necessrio bombear essa gua para fora para extrair o carvo. A mquina a vapor foi inventada para este fim especfico. Bombear a gua para esvaziar as minas. O que o calor? Quanto calor necessrio fornecer a um corpo para aumentar sua temperatura? Como se pode realizar trabalho, produzir energia, a partir do calor? Estas perguntas so respondidas pela Termodinmica, que uma parte da Fsica que se desenvolveu justamente da tentativa de compreender e de aperfeioar a mquina a vapor. Da Grcia antiga h relatos sobre como produzir movimento por meio do calor. H citaes sobre como as portas de um templo se abriam "sozinhas" quando era aceso um fogo no altar (o ar dentro do altar era aquecido e se expandia, assim causava o movimento das portas). Voc pode construir uma "engenhoca" dessas para abrir portas "misteriosamente". Eis o plano de construo: Todavia, no havia mquinas que funcionassem continuamente base de calor. Havia moinhos e indstrias incipientes que eram movidas por rodas d'gua, mas isto s era possvel em locais onde havia quedas d'gua. A primeira mquina a vapor foi construda em 1698, mas era pouco eficiente e s se tornou de importncia econmica e social depois de aperfeioada durante cerca de setenta anos. Foi nesse perodo tambm que as idias fundamentais do estudo do calor se tornaram mais claras. Ainda no havia uma distino entre temperatura e quantidade de calor. Depois ficou evidente que eram necessrias estas duas grandezas para que se pudessem descreverem coerentemente os fenmenos observados. Ficou patente que: para aumentar a temperatura de um corpo deve-se fornecer calor a ele, como, por exemplo, uma panela com gua no fogo. Quanto mais calor a panela receber do fogo, mais alta ser sua temperatura.

Prof: Sandro Fernandes

48

Constatou-se que, para atingir certa temperatura, por exemplo, para fazer a gua ferver, a quantidade de calor necessria vai depender da quantidade de gua: para fazer ferver dois litros, o tempo (a quantidade de calor) ser aproximadamente o dobro do que para um litro. Isto mostra que quantidade de calor e temperatura so coisas distintas. Alm do mais, se em vez de gua houver leo ou outra substncia na panela, o calor que precisa ser fornecido para atingir a mesma temperatura novamente diferente: as diferentes substncias, mesmo em volumes (ou pesos) iguais, precisam de diferentes quantidades de calor para atingir a mesma temperatura final. O que ento o calor? Acreditava-se, no sculo XVIII, que era uma substncia invisvel que podia penetrar em qualquer corpo, chamada calrico. As explicaes de ento era algo assim: O calrico era atrado pelos tomos das substncias, mas era auto-repelente. Formava-se uma nuvem de calrico em torno de cada tomo e estas nuvens se repeliam entre si, evitando assim que os tomos se aproximassem demais. A temperatura dependia da densidade de calrico na superfcie do corpo. Para aumentar a temperatura, fornecia-se calrico ao corpo. Isto aumentava no s o calrico na superfcie, mas tambm a repulso entre os tomos, fazendo com que o corpo aumentasse de volume. Explicava-se assim o fato de os corpos se dilatarem quando a temperatura aumenta. A teoria do calrico explicava quase todos os fenmenos trmicos e foi aceita durante todo o sculo XVIII. Ainda hoje a parcela da populao no cientfica (e que pensa sobre estes fenmenos) julga o calor como se fosse um fluido, e mesmo entre os cientistas muitas palavras e muitas idias vm dessa teoria. No entanto, ela foi abandonada, em meados do sculo XIX, porque ficou claro que o calor uma forma de energia que passa de um corpo a outro e que devida s energias de movimento de todos os tomos do corpo. Se o corpo slido, o movimento de seus tomos de vibrao em torno de uma posio de equilbrio: quanto maior a temperatura, mais intensas estas vibraes. Se for um gs ou um lquido, seus tomos (ou molculas) se deslocam com velocidades tanto maiores, quanto maior for a temperatura. H muitas transformaes possveis de outras formas de energia em calor e vice-versa. Por exemplo, quando dois corpos slidos so esfregados um sobre outro, produz-se o calor: era assim que os ndios faziam fogo. Uma observao famosa deste tipo foi feita pelo conde Rumford, um aventureiro e cientista americano, na Bavria, em fins do sculo XVIII. Ele estava fabricando canhes para o potentado local, e para isto um tarugo de bronze precisava ser furado por uma broca de ao, para se fazer a alma do canho (o furo do cano). Ele observou que o atrito da broca com o canho produzia calor continuamente, mesmo quando a broca j tinha perdido o seu corte. Parecia haver um reservatrio infinitamente grande de calrico que fornecia o fluido broca e ao canho enquanto os dois estivessem se movimentando com atrito. Alis, quem j utilizou uma furadeira eltrica, sabe que sempre a broca esquenta e precisa ser resfriada com gua para no se destemperar a altas temperaturas. Quando a broca est cega (sem corte), ela esquenta muito mais. A energia que gasta no motor que faz a broca girar (Rumford usava uma parelha de cavalos em vez de motor) aparece como calor que aquece a broca e a pea que est sendo furada.
Prof: Sandro Fernandes

49

Quando um ferro de passar roupa ligado, a corrente eltrica que passa dentro o faz ficar quente: energia eltrica sendo transformada diretamente em calor. Reaes qumicas tambm liberam calor: quando comemos alimentos, a digesto (que consiste numa srie de reaes qumicas) produz calor que, entre outras coisas, mantm a nossa temperatura prxima de 37C. Se no houvesse fornecimento contnuo de calor ao nosso corpo, ele logo se resfriaria at ficar mesma temperatura do ambiente, por exemplo, 20C: o que acontece na morte. Um cadver adquire a mesma temperatura dos objetos em volta dele, como se fosse uma pedra ou um pedao de madeira. Os corpos vivos s mantm uma temperatura mais alta do que o ambiente graas energia que recebem pela alimentao. O calor pode dar lugar a outras formas de energia: a mquina a vapor produz trabalho ou energia mecnica a partir do calor. Um termopar um dispositivo que produz energia eltrica quando aquecido; utilizado (nos termostatos) para controlar a temperatura e mant-la constante. Por exemplo, pode se fazer com que seja ligado a um aquecedor sempre que a temperatura de um local aquecido caia abaixo de um valor preestabelecido. O calor uma forma de energia. Assim como existe energia mecnica, eltrica, qumica, nuclear, existe tambm energia trmica e calor. Entretanto desde o incio da termodinmica ficou claro que calor uma forma especial de energia, diferente das outras. energia em forma catica, desordenada. possvel transformar energia mecnica totalmente em energia trmica (e, posteriormente, em calor); tambm possvel transformar energia eltrica totalmente em calor. Mas no possvel transformar totalmente calor em energia mecnica ou em energia eltrica. Para se produzir trabalho ou eletricidade a partir do calor, necessrio sempre ter-se um excesso de calor e jogar-se grande parte dele fora. Isto foi percebido, j em 1824, por um jovem cientista francs, Sadi Carnot, que procurava um jeito de tornar a mquina a vapor mais eficiente: gastar menos carvo ou lenha e realizar mais trabalho. Na caldeira, o fogo aquece a gua transformando-a em vapor a alta temperatura e, conseqentemente, a alta presso. Esse vapor se expande no cilindro, forando o pisto a recuar nesse movimento que h realizao de trabalho. A expanso esfria um pouco o vapor, mas ele continua quente; para continuar o processo e fechar o ciclo, fazendo a gua voltar caldeira, necessrio esfriar o vapor ainda mais e liquefaz-lo, o que ocorre no condensador. O que Carnot percebeu que o condensador indispensvel em um processo cclico, e que ele representa uma ineficincia intrnseca, irremovvel, do processo, pois nele parte do calor que a caldeira forneceu, e que no foi transformada em trabalho no pisto, transferida para fora da mquina, por exemplo, para a gua do rio que resfria o condensador. No possvel construir-se uma mquina, seja a vapor ou de outro tipo, que transforme totalmente em trabalho (energia mecnica) certa quantidade de calor fornecida por uma chama ou por outra forma: parte do calor sempre sobra e precisa ser retirada. Esta impossibilidade uma lei da natureza, que se chama Segundo Princpio da termodinmica (o Primeiro Princpio afirma que o calor uma forma de energia). Os movimentos estudados na Mecnica so todos reversveis, isto , podem decorrer de trs para frente: a Terra poderia girar de leste para oeste em vez de como ela efetivamente gira; uma bola que rola rampa abaixo, acelerada, poderia rolar para cima, desacelerada, se assim fosse lanada. Quando vemos um filme de cinema projetado de trs para frente, muitas vezes demoramos para perceb-lo se s aparecerem
Prof: Sandro Fernandes

50

movimentos puramente mecnicos, reversveis. S quando aparece um processo irreversvel, onde h produo de calor, a coisa fica bvia: um homem que salta do trampolim e cai numa piscina no pode voltar para cima, pois, quando cai na gua, sua energia mecnica (de movimento) desaparece para dar lugar a pequenos movimentos desordenados da gua, e a calor, e este processo irreversvel. Os processos em que intervm o calor so irreversveis. Uma xcara de caf quente colocada sobre a mesa perde calor at que sua temperatura se iguale do ar circundante. Nunca ocorre o inverso uma xcara de caf frio sobre a mesa no se aquece espontaneamente, retirando calor do ar em volta. A impossibilidade deste processo tambm leva ao Segundo Princpio. A energia trmica, que pode ser transferida a outro corpo sob a forma de calor, consiste na soma das energias de movimento desordenado de bilhes de tomos. J a energia de movimento da Terra quando gira ou de uma bola que rola soma de energias de movimento ordenado de muitos e muitos tomos. Nos dois casos, so os movimentos dos tomos que detm (armazenam) a energia, mas num caso todos se movem em conjunto, coordenados, quando o corpo todo se move, e noutro caso cada tomo se move em uma direo diferente, com velocidade diferente e mudando rapidamente. O quente de uma xcara de caf consiste no movimento desordenado, mas rpido de todos os seus tomos, uns vibrando sem sair do lugar, outros indo para cima, outros para baixo, para o lado, e assim por diante. Quando o caf esfria, estes movimentos continuam, mais devagar, entretanto, com menos energia. A figura mostra como seria o caminho de uma partcula em um movimento desordenado desses. interessante notar que este movimento foi descoberto por um botnico chamado Brown, quando observava gros de plen de flores, com um microscpio. Depois se percebeu que tais movimentos ocorrem para quaisquer partculas no ar ou em um lquido. Nos slidos, os movimentos das partculas, devido energia trmica, so diferentes da figura, pois so deslocamentos em torno de uma posio fixa, como uma corda de violo que vibra para um lado e para outro de sua posio de equilbrio.
http://www.feiradeciencias.com.br

Prof: Sandro Fernandes

51

(II) Perfil vertical da variao de temperatura na nossa atmosfera e seus efeitos.


Por convenincia de estudo a atmosfera usualmente subdividida em camadas concntricas, de acordo com o perfil vertical mdio de temperatura (Fig.1).

A camada inferior, onde a temperatura decresce com a altitude, a troposfera, que se estende a uma altitude mdia de 12 km. Nesta camada a taxa de variao vertical da temperatura tem valor mdio de 6,5C/km. Esta taxa na realidade bastante varivel. De fato, algumas vezes a temperatura cresce em finas camadas, caracterizando uma inverso de temperatura. A troposfera o principal domnio de estudo dos meteorologistas, pois nesta camada que ocorrem essencialmente todos os fenmenos que em conjunto caracterizam o tempo. Na troposfera as propriedades atmosfricas so facilmente transferidas por turbulncia de grande escala e mistura. O seu limite superior conhecido como tropopausa. A camada seguinte, a estratosfera, se estende at 50 km. Inicialmente, por uns 20 km, a temperatura permanece quase constante e depois cresce at o topo da estratosfera, a estratopausa. Temperaturas mais altas ocorrem na estratosfera porque nesta camada que o oznio est concentrado. Conforme mencionamos, o oznio absorve radiao ultravioleta do sol. Conseqentemente, a estratosfera aquecida. Na mesosfera a temperatura novamente decresce com a altura, at a mesopausa, que est em torno de 80 km, onde atinge -90C. Acima da mesopausa, e sem limite superior definido, est a termosfera, onde a temperatura inicialmente isotrmica e depois cresce rapidamente com a altitude, como resultado da absoro de ondas muito curtas da radiao solar por tomos de oxignio e nitrognio. Embora as temperaturas atinjam valores muito altos, estas temperaturas no so exatamente comparveis quelas experimentadas prximo a superfcie da Terra. Temperaturas so definidas em termos da velocidade mdia
Prof: Sandro Fernandes

52

das molculas. Como as molculas dos gases da termosfera se movem com velocidades muito altas, a temperatura obviamente alta. Contudo, a densidade to pequena que muito poucas destas molculas velozes colidiriam com um corpo estranho; portanto, s uma quantidade insignificante de energia seria transferida. Portanto, a temperatura de um satlite em rbita seria determinada principalmente pela quantidade de radiao solar que ele absorve e no pela temperatura do ar circundante. Entre as altitudes de 80 a 900 km (na termosfera) h uma camada com concentrao relativamente alta de ons, a ionosfera. Nesta camada a radiao solar de alta energia de ondas curtas (raios X e radiao ultravioleta) tira eltrons de molculas e tomos de nitrognio e oxignio, deixando eltrons livres e ons positivos. A maior densidade de ons ocorre prximo a 300 km. A concentrao de ons pequena abaixo de 80 km porque nestas regies muito da radiao de ondas curtas necessria para ionizao j foi esgotada. Acima de 400 km a concentrao pequena por causa da extremamente pequena densidade do ar, possibilitando a produo de poucos ons. A estrutura da ionosfera consiste de 3 camadas de densidade varivel de ons: as camadas D, E e F, com altitude e densidade de ons crescente. Como a produo de ons requer a radiao solar direta, a concentrao de ons diminui do dia para a noite, particularmente nas camadas D e E, onde os eltrons se recombinam com ons positivos durante a noite. A taxa de recombinao depende da densidade do ar, isto , quanto mais denso o ar maior a probabilidade de coliso e recombinao das partculas. Assim, a camada D desaparece noite, a camada E se enfraquece consideravelmente, mas a camada F continua presente noite, embora enfraquecida, pois a densidade nesta camada muito pequena. A ionosfera tem pequeno impacto sobre o tempo, mas tem grande influncia sobre a transmisso de ondas de rdio na banda AM. Durante o dia as ondas de rdio tendem a ser absorvidas nas dois camadas mais baixas, especialmente na camada D. A camada F reflete as ondas de rdio durante o dia e a noite. Contudo , mesmo que as ondas consigam atravessar as camadas D e E e ser refletidas na camada F, elas sero absorvidas no seu caminho de volta para a Terra. noite, contudo, a camada absorvedora D desaparece e as ondas podem atingir a camada F mais facilmente e ser refletidas para a superfcie da Terra. Isto explica porque noite os sinais de rdio atingem grandes distncias sobre a Terra (Fig. 1.4).

Fig. 1.4 - Influncia da Ionosfera sobre a transmisso de ondas de rdio.


Prof: Sandro Fernandes

53

Na ionosfera ocorre tambm o fenmeno da aurora boreal (no Hemisfrio Norte) ou austral (no Hemisfrio Sul). As auroras esto relacionadas com o vento solar, um fluxo de partculas carregadas, prtons e eltrons, emanadas do sol com alta energia. Quando estas partculas se aproximam da Terra, elas so capturadas pelo campo magntico da Terra. Sob a ao da fora exercida pelo campo magntico sobre cargas em movimento, elas descrevem trajetrias espiraladas ao longo das linhas de induo do campo magntico terrestre, movendo-se para frente e para trs entre os plos magnticos sul e norte, onde so "refletidas" devido ao aumento do campo magntico. Estes eltrons e prtons aprisionados constituem os chamados "cintures radioativos de Van Allen". Algumas partculas acompanham o campo magntico da Terra em direo aos plos geomagnticos, penetrando na ionosfera, onde colidem com tomos e molculas de oxignio e nitrognio, que so temporariamente energizados. Quando estes tomos e molculas retornam do seu estado energtico excitado, eles emitem energia na forma de luz, o que constitui as auroras. As zonas de maior ocorrncia das auroras situam-se em torno de 20-30 ao redor dos plos geomagnticos (76N, 102W; 68S, 145E). A atividade auroral varia com a atividade do sol. Quando o sol est calmo, a zona auroral diminui; quando o sol est ativo (com exploses solares), intensificando o vento solar, a zona auroral se expande em direo ao equador.
Fonte: http://fisica.ufpr.br

(III) O EFEITO ESTUFA: FATO OU FICO? Os seres vivos necessitam de energia para sobreviver. A energia que mantm a vida sobre a Terra vem do Sol, que irradia energia para o espao por ser muito quente. Uma proporo minscula dessa energia alcana a Terra. A atmosfera terrestre funciona como uma camada protetora sobre a superfcie de nosso planeta, impedindo as variaes de temperatura que existiriam em um mundo sem ar. A maior parte da energia irradiada pelo Sol passa pela atmosfera terrestre. A Terra absorve parte dessa energia e a outra parte refletida pela superfcie terrestre. Parte dessa energia refletida absorvida pela atmosfera. Como resultado disso, a temperatura mdia acima da superfcie da Terra mais alta do que seria se no existisse atmosfera. A atmosfera terrestre funciona como uma estufa, da o termo efeito estufa. O efeito estufa teria tornado-se mais evidente durante o Sculo XX. Os grficos abaixo indicam a relao entre a emisso de gs carbnico e o aumento da temperatura mdia da atmosfera terrestre. Em jornais e revistas, o aumento da emisso do gs carbnico freqentemente apontado como o principal responsvel pela elevao da temperatura no Sculo XX.

Prof: Sandro Fernandes

54

Fonte: Questo do PISA

Algumas Aplicaes... 1) Analisando os dois grficos e texto sobre o efeito estufa, analise as afirmativas abaixo indicando a nica correta. a) A pequena parte da energia irradiada pelo sol reflete na atmosfera aumentando assim a temperatura na camada de oznio e acarretando no aumento da temperatura mdia do planeta. b) De acordo com os grficos desde 1860 j percebemos um aumento progressivo e constante da temperatura na atmosfera terrestre e por esse motivo no podemos relacionar a emisso de gs carbnico com esse aumento de temperatura. c) De acordo com os grficos podemos verificar que ao longo das dcadas analisadas, o aumento da emisso de carbono sempre esteve acompanhado do aumento da temperatura da nossa atmosfera. d) De acordo com o texto: A atmosfera terrestre funciona como uma estufa, da o termo efeito estufa. Logo, se nosso planeta no tivesse atmosfera no teramos este efeito e com isso as temperaturas seriam mais amenas ao longo das vrias dcadas analisadas. e) O ttulo, O efeito estufa: Fato ou Fico? , juntamente com a anlise dos dois grficos durante as primeiras dcadas do sculo XX nos leva a refletir sobre a real responsabilidade do gs carbnico no aumento da temperatura da nossa atmosfera e no efeito estufa.
Prof: Sandro Fernandes

55

2) Na figura a seguir tem-se um dispositivo que nos ajuda a entender as formas pelas quais o calor se propaga.

Observa-se que, em um local livre de correntes de ar, ao ligar a lmpada - transformao de energia eltrica em trmica -, a ventoinha acima da lmpada comea a girar. Isto se deve, principalmente, devido (s): a) irradiao trmica prxima lmpada aquecida b) conveco trmica do ar prximo lmpada aquecida c) conduo trmica que predomina nos metais d) fora de atrao gravitacional entre a ventoinha e a lmpada e) foras de ao e de reao 3) Analise as afirmativas a seguir e em seguida marque a alternativa correta: I) Um corpo pode permanecer com sua temperatura inalterada, mesmo que esteja ganhando ou perdendo energia trmica. II) A quantidade de calor que altera a temperatura de um corpo chama-se calor sensvel. III) A energia do Sol no pode chegar at ns atravs dos processos de conduo e conveco trmica. a) Apenas as afirmativas II e III so corretas b) Apenas as afirmativas I e III so corretas c) Apenas a afirmativa III est correta d) Apenas as afirmativas I e II so corretas e) Todas as afirmativas so corretas 4) A histria da maioria dos municpios gachos coincide com a chegada dos primeiros portugueses, alemes, italianos e de outros povos. No entanto, atravs dos vestgios materiais encontrados nas pesquisas arqueolgicas, sabemos que outros povos, anteriores aos citados, protagonizaram a nossa histria. Diante da relevncia do contexto e da vontade de valorizar o nosso povo nativo, "o ndio", foi selecionada a rea temtica CULTURA e as questes foram construdas com base na obra "Os Primeiros Habitantes do Rio Grande do Sul" (Custdio, L. A. B., organizador. Santa Cruz do Sul: EDUNISC; IPHAN, 2004).

Prof: Sandro Fernandes

56

"Os habitantes dos cerritos, com o tempo, foram aprendendo a plantar e a moldar potes de barro cozido." Os ndios usavam panelas de barro. Modernamente usamos panelas de metais, como alumnio e ao inoxidvel, com cabos de madeira ou baquelite. Os metais so ________________ de energia na forma de calor, pois possuem ___________ condutividade trmica. O material do cabo possui ___________ condutividade trmica. Assinale a alternativa que preenche corretamente as lacunas. a) bons condutores - baixa baixa c) bons condutores - alta alta e) bons condutores - alta - baixa 5) O SI (Sistema Internacional de unidades) adota como unidade de calor o joule, pois calor energia. No entanto, s tem sentido falar em calor como energia em trnsito, ou seja, energia que se transfere de um corpo a outro em decorrncia da diferena de temperatura entre eles. Assinale a afirmao em que o conceito de calor est empregado corretamente. a) A temperatura de um corpo diminui quando ele perde parte do calor que nele estava armazenado. b) A temperatura de um corpo aumenta quando ele acumula calor. c) A temperatura de um corpo diminui quando ele cede calor para o meio ambiente. d) O aumento da temperatura de um corpo um indicador de que esse corpo armazenou calor. e) Um corpo s pode atingir o zero absoluto se for esvaziado de todo o calor nele contido. 6) Estufas rurais so reas limitadas de plantao cobertas por lonas plsticas transparentes que fazem, entre outras coisas, com que a temperatura interna seja superior externa. Isso se d porque: a) o ar aquecido junto lona desce por conveco at as plantas. b) as lonas so mais transparentes s radiaes da luz visvel que s radiaes infravermelhas. c) um fluxo lquido contnuo de energia se estabelece de fora para dentro da estufa. d) a expanso do ar expulsa o ar frio para fora da estufa. e) o ar retido na estufa atua como um bom condutor de calor, aquecendo o solo. 7) Numa rea de praia, a brisa martima uma conseqncia da diferena no tempo de aquecimento do solo e da gua, apesar de ambos estarem submetidos s mesmas condies de irradiao solar. No local (solo) que se aquece mais rapidamente, o ar fica mais quente e sobe, deixando uma rea de baixa presso, provocando o deslocamento do ar da superfcie que est mais fria (mar). b) maus condutores - baixa - alta d) maus condutores - baixa - baixa

Prof: Sandro Fernandes

57

noite, ocorre um processo inverso ao que se verifica durante o dia. Como a gua leva mais tempo para esquentar (de dia), mas tambm leva mais tempo para esfriar ( noite), o fenmeno noturno (brisa terrestre) pode ser explicado da seguinte maneira: a) O ar que est sobre a gua se aquece mais; ao subir, deixa uma rea de baixa presso, causando um deslocamento de ar do continente para o mar. b) O ar mais quente desce e se desloca do continente para a gua, a qual no conseguiu reter calor durante o dia. c) O ar que est sobre o mar se esfria e dissolve-se na gua; forma-se, assim, um centro de baixa presso, que atrai o ar quente do continente. d) O ar que est sobre a gua se esfria, criando um centro de alta presso que atrai massas de ar continental. e) O ar sobre o solo, mais quente, deslocado para o mar, equilibrando a baixa temperatura do ar que est sobre o mar. 8) Analise as afirmaes referentes conduo trmica I - Para que um pedao de carne cozinhe mais rapidamente, pode-se introduzir nele um espeto metlico. Isso se justifica pelo fato de o metal ser um bom condutor de calor. II - Os agasalhos de l dificultam a perda de energia (na forma de calor) do corpo humano para o ambiente, devido ao fato de o ar aprisionado entre suas fibras ser um bom isolante trmico. III - Devido conduo trmica, uma barra de metal mantm-se a uma temperatura inferior de uma barra de madeira colocada no mesmo ambiente. Podemos afirmar que a) I, II e III esto corretas. c) Apenas I est correta. e) Apenas I e II esto corretas. b) I, II e III esto erradas. d) Apenas II est correta.

9) Uma garrafa de vidro e uma lata de alumnio, cada uma contendo 330mL de refrigerante, so mantidas em um refrigerador pelo mesmo longo perodo de tempo. Ao retir-las do refrigerador com as mos desprotegidas, tem-se a sensao de que a lata est mais fria que a garrafa. correto afirmar que:
Prof: Sandro Fernandes

58

a) a lata est realmente mais fria, pois a cidade calorfica da garrafa maior que a da lata. b) a lata est de fato menos fria que a garrafa, pois o vidro possui condutividade menor que o alumnio. c) a garrafa e a lata esto mesma temperatura, possuem a mesma condutividade trmica, e a sensao deve-se diferena nos calores especficos. d) a garrafa e a lata esto mesma temperatura, e a sensao devida ao fato de a condutividade trmica do alumnio ser maior que a do vidro. e) a garrafa e a lata esto mesma temperatura, e a sensao devida ao fato de a condutividade trmica do vidro ser maior que a do alumnio. 10) O resultado da converso direta de energia solar uma das vrias formas de energia alternativa de que se dispe. O aquecimento solar obtido por uma placa escura coberta por vidro, pela qual passa um tubo contendo gua. A gua circula, conforme mostra o esquema abaixo.

Fonte: Adaptado de PALZ, Wolfgang, "Energia solar e fontes alternativas". Hemus, 1981. So feitas as seguintes afirmaes quanto aos materiais utilizados no aquecedor solar: I. o reservatrio de gua quente deve ser metlico para conduzir melhor o calor. II. a cobertura de vidro tem como funo reter melhor o calor, de forma semelhante ao que ocorre em uma estufa. III. a placa utilizada escura para absorver melhor a energia radiante do Sol, aquecendo a gua com maior eficincia. Dentre as afirmaes acima, pode-se dizer que, apenas est(o) correta(s): a) I b) I e II c) II d) I e III e) II e III

11) A gua de um rio encontra-se a uma velocidade inicial V constante, quando despenca de uma altura de 80 m, convertendo toda a sua energia mecnica em calor. Este calor integralmente absorvido pela gua, resultando em um aumento de 1 K de sua temperatura. Considerando 1 cal = 4 J, acelerao da gravidade g = 10 m/s2 e calor especfico da gua c = 1,0 cal/g.C, calcula-se que a velocidade inicial da gua V de a) 10 m/s. b) 20 m/s. c) 50 m/s. d) 70 m/s. e) 80 m/s.

Prof: Sandro Fernandes

59

12) Numa aula de Fsica, o Professor Sandro apresenta a seus alunos esta experincia: dois blocos um de alumnio e outro de ferro -, de mesma massa e, inicialmente, temperatura ambiente, recebem a mesma quantidade de calor, em determinado processo de aquecimento. O calor especfico do alumnio e o do ferro so, respectivamente, 0,90 J/(gC) e 0,46 J/(gC). Questionados quanto ao que ocorreria em seguida, dois dos alunos, Alexandre e Lorena, fazem, cada um deles, um comentrio: - Paulo: "Ao final desse processo de aquecimento, os blocos estaro mesma temperatura." - Priscilla: "Aps esse processo de aquecimento, ao se colocarem os dois blocos em contato, fluir calor do bloco de ferro para o bloco de alumnio." Considerando-se essas informaes, CORRETO afirmar que a) apenas o comentrio de Paulo est certo. b) apenas o comentrio de Priscilla est certo. c) ambos os comentrios esto certos. d) nenhum dos dois comentrios est certo. e) so necessrios mais dados para responder. 13) Uma "bala perdida" disparada com velocidade de 200,0 m/s penetrou na parede ficando nela incrustada. Considere que 50% da energia cintica da bala foi transformada em calor, ficando nela retida. A variao de temperatura da bala, em C, imediatamente ao parar, (Considere: Calor especfico da bala: 250 J / kg C) a) 10 b) 20 c) 40 d) 80 e) 160

14) No banheiro, de sua casa de Petrpolis, Santos Dumont fez um chuveiro de gua quente tendo o lcool por combustvel. O calor da chama faz a gua entrar em ebulio, subindo para um balde que possui duas alavancas: uma para misturar a gua quente com a fria e outra para abrir o fundo de um balde, perfurado como regador de plantas. Assim, ele podia repetir o movimento quantas vezes julgasse necessrio, tornando a ducha aquecida, um luxo indito para a poca. (Adaptado de "http://novaescola.abril.com.br/index.htm")

Prof: Sandro Fernandes

60

Considere que para um banho quente com o chuveiro descrito, um visitante da casa utiliza 4 kg de gua fria a 20 C e 2 kg de gua quente a 80 C. Sendo o sistema termicamente isolado, a temperatura da mistura das guas, em C Dados: Calor Especfico da gua: c = 1 kcal/kg.C a) 20. b) 30. c) 40. d) 50. e) 60.

15) Qualquer dos seus leitores que tenha a ventura de residir em meio ao romntico cenrio do Pas de Gales ou da Esccia poderia, no tenho dvida, confirmar meus experimentos medindo a temperatura no topo e na base de uma cascata. Se minhas observaes esto corretas, uma queda de 817 ps deve gerar um grau de calor, e a temperatura do rio Nigara deve subir cerca de um quinto de grau por causa de sua queda de 160 ps. Esse trecho foi publicado em 1845 por James P. Joule na seo de cartas da revista inglesa "Philosophical Magazine" e ilustra os resultados por ele obtidos em suas experincias para a determinao do equivalente mecnico do calor. Sendo c(gua) = 4 200 J/(kgC) o calor especfico da gua, adotando g = 10 m/s, 817ps = 250 m e 160ps = 50 m, pode-se afirmar que, ao se referir a "um grau de calor" e a "um quinto de grau", Joule est exprimindo valores de temperatura que, em graus Celsius, valem aproximadamente a) 5,0 e 1,0. d) 0,30 e 0, 060. b) 1,0 e 0,20. e) 0,10 e 0, 020. c) 0,60 e 0,12.

16) Na tabela possvel ler os valores do calor especfico de cinco substncias no estado lquido, e no grfico representada a curva de aquecimento de 100 g de uma dessas substncias.

A curva de aquecimento representada a a) da gua. d) da acetona. b) do lcool etlico. e) do benzeno. c) do cido actico.

Prof: Sandro Fernandes

61

17) Considere seus conhecimentos sobre mudanas de fase e analise as afirmaes I, II e III, referentes substncia gua, um recurso natural de alto valor. I. Durante a transio de slido para lquido, a temperatura no muda, embora uma quantidade de calor tenha sido fornecida gua. II. O calor latente de condensao da gua tem um valor diferente do calor latente de vaporizao. III. Em determinadas condies, a gua pode coexistir na fase slida, lquida e gasosa. Pode-se afirmar que a) apenas a afirmao I correta. b) apenas as afirmaes I e II so corretas. c) apenas as afirmaes I e III so corretas. d) apenas as afirmaes II e III so corretas. e) as afirmaes I, II e III so corretas.

18) Em 1883, um vapor ingls de nome Tramandata naufragou no rio Tiet encontrando-se, hoje, a 22 metros de profundidade em relao superfcie. O vapor gerado pela queima de lenha na caldeira fazia girar pesadas rodas laterais, feitas de ferro, que, ao empurrarem a gua do rio, movimentavam o barco. Considere que na caldeira do Tramandata sejam aquecidos 5000 litros de gua inicialmente a 20C. Para que metade dessa gua seja transformada em vapor d'gua, so necessrios, joules, a) 5,4 b) 6,2 c) 7,0 d) 7,5 e) 8,0

19) A Terra cercada pelo vcuo espacial e, assim, ela s perde energia ao irradi-la para o espao. O aquecimento global que se verifica hoje decorre de pequeno desequilbrio energtico, de cerca de 0,3%, entre a energia que a Terra recebe do Sol e a energia irradiada a cada segundo, algo em torno de 1 W/m2. Isso significa que a Terra acumula, anualmente, cerca de 1,6 10 22 J. Considere que a energia necessria para transformar 1 kg de gelo a 0C em gua lquida seja igual a 3,2 105 J. Se toda a energia acumulada anualmente fosse usada para derreter o gelo nos plos (a 0C), a quantidade de gelo derretida anualmente, em trilhes de toneladas, estaria entre a) 20 e 40. b) 40 e 60. c) 60 e 80. d) 80 e 100. e) 100 e 120.

20) Um bloco de gelo com 725 g de massa colocado num calormetro contendo 2,50 kg de gua a uma temperatura de 5,0C, verificando-se um aumento de 64 g na massa desse bloco, uma vez alcanado o equilbrio trmico. Considere o calor especfico da gua (c = 1,0 cal/gC) o dobro do calor especfico do gelo, e o calor latente de fuso do gelo de 80 cal/g. Desconsiderando a capacidade trmica do calormetro e a troca de calor com o exterior, assinale a temperatura inicial do gelo. a) -191,4C b) -48,6C c) -34,5C d) -24,3C e) -14,1C

Prof: Sandro Fernandes

62

De acordo com a Matriz de Referncias para o Enem 2009. Professor: Sandro Fernandes Tema: Energia, Trabalho e Potncia

Aula V de Fsica
(I) Panorama energtico atual e perspectivas futuras
A demanda projetada de energia no mundo aumentar 1,7% ao ano, de 2000 a 2030, quando alcanar 15,3 bilhes de toneladas equivalentes de petrleo (TEP, ou toe, na sigla internacional, em ingls) por ano, de acordo com o cenrio base traado pelo Instituto Internacional de Economia (Mussa, 2003). Em condies ceteris paribus, sem alterao da matriz energtica mundial, os combustveis fsseis responderiam por 90% do aumento projetado na demanda mundial, at 2030. Entretanto, o esgotamento progressivo das reservas mundiais de petrleo uma realidade cada vez menos contestada. A Bristish Petroleum, em seu estudo Reviso Estatstica de Energia Mundial de 2004, afirma que atualmente as reservas mundiais de petrleo durariam em torno de 41 anos, as de gs natural, 67 anos, e as reservas brasileiras de petrleo, 18 anos. A matriz energtica mundial tem participao total de 80% de fontes de carbono fssil, sendo 36% de petrleo, 23% de carvo e 21% de gs natural (Tabela 1, abaixo). O Brasil se destaca entre as economias industrializadas pela elevada participao das fontes renovveis em sua matriz energtica. Isso se explica por alguns privilgios da natureza, como uma bacia hidrogrfica contando com vrios rios de planalto, fundamental a produo de eletricidade (14%), e o fato de ser o maior pas tropical do mundo, um diferencial positivo para a produo de energia de biomassa (23%).

Fonte Petrleo Carvo mineral Gs natural Biomassa tradicional Energia Nuclear Energia Hidroeltrica Biomassa moderna Outras energias renovveis

Mundo 35,3 23.2 21,1 9,5 6,5 2,2 1,7 0,5

Brasil 43,1 6,0 7,5 8,5 1,8 14,0 23,0 0,1

Prof: Sandro Fernandes

63

A Figura 1 apresenta a cotao do barril de petrleo, no perodo 1861 - 2005 (valores de 1/7 de cada ano). Do grfico possvel inferir, com clareza, que o perodo de 100 anos de petrleo barato (cotao entre US$10-20/barril), que vigiu at 1970, est definitivamente superado. Por questes conjunturais, eventualmente o preo spot poder oscilar abaixo de US$60,00/barril, porm a tendncia de mdio prazo de valores crescentes. perfeitamente razovel traar cenrios com o piso da cotao em US$100,00 a partir do incio da prxima dcada.
Figura 1. Preo internacional do barril de petrleo.

Fonte: Elaborao D. L. Gazzoni

Nesse contexto, passa a ser fundamental a relao de preos entre matrias primas (petrleo, etanol na usina, leo vegetal). O break even, em condies ceteris paribus, entre o preo do lcool e da gasolina (tributao exclusa) oscila entre US$30 e US$35,00. Por ser uma tecnologia ainda imatura, a mesma relao estimada em torno de US$60,00 para biocombustveis derivados de leo vegetal (Figura 2). Figura 2. Preo internacional do petrleo e eventos conexos.

Fonte: Elaborao D. L. Gazzoni


Prof: Sandro Fernandes

64

Como a maioria dos cenrios traados para o preo internacional do petrleo prev a continuidade da escalada de preos, consolida-se o programa do etanol combustvel e ficam criadas as condies para alavancar o programa de biodiesel. Entende-se que as condies comerciais esto delineadas, em forma estrutural, para a viabilizao da agroenergia enquanto componente de alta densidade do agronegcio. As presses de cunho social (emprego, renda, fluxos migratrios) e ambiental (mudanas climticas, poluio) apenas reforam e consolidam essa postura, alm de antecipar cronogramas. Nesse particular, o mundo est cada vez mais temeroso dos impactos negativos dos combustveis fsseis sobre o clima. Consolidando de forma reducionista a percepo de autoridades e cientistas, verifica-se que os extremos climticos (secas, cheias, furaces, etc.) tornaram-se mais freqentes e mais severos. Assad et al. (2004) apresentaram modelos matemticos, que projetam alteraes profundas na temperatura do planeta e desastrosas conseqncias para o agronegcio. As alteraes do clima acarretam modificaes na incidncia de pragas agrcolas, com srias conseqncias econmicas, sociais e ambientais. O cenrio fitossanitrio atual seria significativamente alterado, expondo a vulnerabilidade da agropecuria a essas mudanas e a necessidade de desenvolver estratgias adaptativas de longo prazo. Embora no exista um estudo definitivo comparando a gerao de emprego e renda e sua distribuio, cotejando as cadeias de energia de carbono fssil e de bioenergia, a experincia brasileira e o senso comum indicam que possvel gerar 10-20 vezes mais empregos na agricultura de energia, comparativamente cadeia de petrleo com a vantagem de que os empregos seriam gerados internamente, auxiliando na soluo de um dos mais srios desafios brasileiros. A produo agrcola desconcentra renda mais intensamente que a extrao de petrleo ou gs, podendo tornar o Brasil um paradigma mundial de como enfrentar trs grandes desafios do sculo XXI, com uma nica poltica pblica: atravs do incentivo agricultura de energia, possvel enfrentar os desafios da produo de energia sustentvel, da proteo ambiental e da gerao de emprego e renda, com distribuio mais eqitativa. Alm da temtica ambiental, a questo sanitria tambm possui interface com a temtica da agroenergia. O desenvolvimento de tecnologias para o tratamento e utilizao dos resduos o grande desafio para as regies com alta concentrao de sunos e aves. De um lado, existe a presso pelo aumento do nmero de animais em pequenas reas de produo, e pelo aumento da produtividade e, do outro, que esse aumento no provoque a destruio do meio ambiente e esteja de acordo com o MDL (Mecanismo de Desenvolvimento Limpo). Ressalta-se que a recente crise energtica e a alta dos preos do petrleo tm determinado uma procura por alternativas energticas no meio rural (Lucas Junior, 1994). O processo de digesto anaerbica (biometanizao) consiste de um complexo de cultura mista de microorganismos, capazes de metabolizar materiais orgnicos complexos, tais como carboidratos, lipdios e protenas para produzir metano (CH4) e dixido de carbono (CO2) e material celular (Lucas Junior, 1994; Santos, 2001). Projetando o mdio prazo, importante alinhavar os principais aspectos positivos e negativos das principais fontes energticas, para tornar mais transparente a percepo da evoluo futura da matriz energtica e as reais possibilidades de participao de cada fonte no market share da energia (Tabela 2).

Prof: Sandro Fernandes

65

Tabela 2. Anlise das principais fontes da matriz energtica. Combustvel Carvo Aspectos positivos Abundante, economicamente acessvel, uso seguro Fcil de transportar e de armazenar Amplamente distribudo Aspectos negativos Alta emisso de gases de efeito estufa Necessita portentosos investimentos para desenvolvimento de tecnologias que reduzam as emisses de gases de efeito estufa (GEE) a nveis aceitveis Extrao perigosa Conveniente Petrleo Alta densidade energtica Fcil de transportar e de armazenar Co-evoluo da fonte energtica com os equipamentos para seu uso Fortemente poluidor da atmosfera Preos volteis Concentrao geogrfica das jazidas Produto cartelizado e mercado manipulvel Vulnerabilidade de interrupo de oferta e instabilidade geopoltica Riscos de transporte e armazenamento Reservas em esgotamento Eficiente e conveniente Gs Combustvel multiuso Alta densidade energtica Produto emissor de gases de efeito estufa Transporte e armazenamento caro e arriscado Requer infra-estrutura cara, prpria e inflexvel Volatilidade de preos Jazidas concentradas geograficamente Produto cartelizado e mercado manipulvel No h emisses de gases de efeito estufa Energia Nuclear Energia Renovvel Poucas limitaes de recursos Alta densidade energtica Baixas emisses de gases de efeito estufa Sustentabilidade Baixa aceitao da sociedade Sem soluo para eliminao dos resduos Operao arriscada e perigosa Muito intensivo em capital Custos altos Fontes intermitentes Distribuio desigual Estgio tecnolgico inferior s demais fontes em uso
http://www.biodieselbr.com/energia/agro-energia.htm

Prof: Sandro Fernandes

66

(II)

Instalada Primeira Turbina Elica Ocenica Flutuante

Acaba de ser instalada, na costa da Noruega, a primeira turbina elica ocenica de grande porte. Localizada a 12 km a leste da cidade de Karmoy, a turbina tem um rotor com um dimetro de 82 metros e ser capaz de gerar sozinha 2,3 MegaWatts de energia. A turbina elica flutuante, chamada de HyWind, ser conectada rede eltrica do pas e dever servir como um laboratrio de testes em escala real para a tecnologia de turbinas elicas flutuantes. Ela comear a gerar eletricidade em Julho prximo. Sem necessidade de fundaes Construir fundaes para turbinas elicas torna-se muito caro quando a profundidade ocenica supera os 50 metros, o que poderia limitar a explorao ocenica da energia elica. J a HyWind pode flutuar, tendo sido projetada para ser instalada em locais com profundidades entre 120 e 700 metros. O local onde a primeira HyWind foi instalada tem 220 metros de profundidade. O mastro da turbina estende-se por 65 metros acima da linha d'gua. Seu flutuador construdo em ao, indo at 100 metros de profundidade. Trs cabos de ao ancoram a turbina elica flutuante ao fundo do mar, para que sua posio se mantenha constante. Um sistema avanado de controle permite que a turbina anule parcialmente os movimentos induzidos pelas ondas, mantendo-se mais estvel, o que aumenta sua capacidade de gerao de energia. A HyWind um projeto conjunto das empresas StatoilHydro e Siemens.
http://www.inovacaotecnologica.com.br

(III)

Motor Bate Recorde Mundial, Superando 1 Milho de RPM.

Motores construdos industrialmente j alcanam velocidades de 250.000 rpm, depois de progressos recentes bastante acelerados. Agora, contudo, pesquisadores do Instituto Federal de Tecnologia da Sua superaram de longe este recorde, construindo um motor que atinge 1.000.000 de rpm. Para suportar a extrema fora centrfuga gerada durante seu funcionamento, o minsculo motor foi acondicionado no interior de uma carcaa de titnio e usou rolamentos de esferas otimizados para velocidades extremamente altas. Motores de alta rotao No futuro, motores eltricos de alta eficincia sero necessrios para o processamento de materiais e para equiparem inmeros mecanismos, como os compressores de automveis e avies. Essas aplicaes exigiro motores capazes de atingirem altssimas rotaes e fornecer potncias adequadas - o movo motor gera uma potncia na sada de 100 watts. A regra bsica no projeto de motores eltricos que, quanto maior a rotao, maiores so as perdas. Mas os pesquisadores suos resolveram esse problema projetando um estator que apresenta um nvel muito baixo de perdas. As bobinas eltricas foram construdas com fios de cobre ultrafinos e inseridas em um compartimento de ferro especial, cuja composio no foi detalhada pelos pesquisadores, mas que eles afirmam ser de um tipo at hoje no utilizado em motores.

Prof: Sandro Fernandes

67

Mdulo eletrnico de controle do motor Outro elemento-chave para o estabelecimento do novo recorde de velocidade foi a construo do mdulo eletrnico de controle do motor, projetado para lidar com altssimas velocidades. "Nosso objetivo de quebrar a barreira de um milho de rpm estava clara, mas o avano somente foi possvel graas a esta nova tecnologia," diz o engenheiro Christof Zwyssig. O recorde anterior de velocidade de um motor eltrico pertencia mesma equipe, com uma verso inicial do mesmo projeto, capaz de atingir 500.000 rpm.
http://www.inovacaotecnologica.com.br

Algumas Aplicaes...

1) Considere uma sala noite iluminada apenas por uma lmpada fluorescente. Assinale a alternativa correta. a) A iluminao da sala proveniente do campo magntico gerado pela corrente eltrica que passa na lmpada. b) Toda potncia da lmpada convertida em radiao visvel. c) A iluminao da sala um fenmeno relacionado a ondas eletromagnticas originadas da lmpada. d) A energia de radiao que ilumina a sala exatamente igual energia eltrica consumida pela lmpada. e) A iluminao da sala deve-se ao calor dissipado pela lmpada. 2) O Brasil utiliza o represamento das guas dos rios para a construo de usinas hidroeltricas na gerao de energia eltrica. Porm, isso causa danos ao meio ambiente, como por exemplo: - imensa quantidade de madeira nobre submersa nas guas; - alterao do habitat da vida animal; - assoreamento dos leitos dos rios afluentes. Numa usina hidroeltrica existe uma transformao seqencial de energia. Esta seqncia est indicada na alternativa a) cintica - potencial - eltrica; b) qumica - cintica - eltrica; c) cintica - elstica - eltrica; d) potencial - cintica - eltrica; e) potencial - qumica - eltrica.

Prof: Sandro Fernandes

68

3) Neste momento milhares de pessoas esto passando fome no Brasil e no mundo. A fome conseqncia da pobreza e tambm sua causadora. Para romper esse crculo vicioso, fundamental unir toda a sociedade. S dessa forma ser possvel garantir a condio bsica de direito vida: viver sem fome. (ONU - "8 Objetivos do Milnio - 8 Jeitos de mudar o Mundo") A alimentao diria de um jovem deve conter 2400 quilocalorias (kcal) de nutrientes energticos para que os seus rgos possam desenvolver suas funes. A unidade caloria (cal) utilizada no campo da Fsica relacionada com o conceito de trabalho e energia. Outra unidade relacionada com a noo de trabalho e energia conhecida por a) ampere b) joule. c) newton. d) volt. e) watt.

4) O bloco A da figura desliza sobre uma superfcie horizontal sem atrito puxado pelo bloco B. O fio e a polia so ideais. O grfico que representa qualitativamente a energia cintica do sistema em funo do tempo a partir do instante em que o bloco A atinge o ponto P

5) A figura representa um projtil logo aps ter atravessado uma prancha de madeira, na direo x perpendicular prancha. Supondo que a prancha exera uma fora constante de resistncia ao movimento do projtil, o grfico que melhor representa a energia cintica do projtil, em funo de x,

Prof: Sandro Fernandes

69

6) O setor de transporte, que concentra uma grande parcela da demanda de energia no pas, continuamente busca alternativas de combustveis. Investigando alternativas ao leo diesel, alguns especialistas apontam para o uso do leo de girassol, menos poluente e de fonte renovvel, ainda em fase experimental. Foi constatado que um trator pode rodar, NAS MESMAS CONDIES, mais tempo com um litro de leo de girassol, que com um litro de leo diesel. Essa constatao significaria, portanto, que usando leo de girassol, a) o consumo por km seria maior do que com leo diesel. b) as velocidades atingidas seriam maiores do que com leo diesel. c) o combustvel do tanque acabaria em menos tempo do que com leo diesel. d) a potncia desenvolvida, pelo motor, em uma hora, seria menor do que com leo diesel. e) a energia liberada por um litro desse combustvel seria maior do que por um de leo diesel. 7) O carrinho da figura a seguir repousa sobre uma superfcie horizontal lisa e no seu interior h um pndulo simples, situado inicialmente em posio horizontal. O pndulo liberado e sua massa m se move at colidir com a parede do carrinho no ponto P, onde fica colada. A respeito desse fato, considere as seguintes afirmaes.

I. A lei de conservao da quantidade de movimento assegura que, cessada a coliso, o carrinho estar se movendo para a direita com velocidade constante. II. A ausncia de foras externas horizontais atuando sobre o sistema (carrinho+pndulo) assegura que, cessada a coliso, o carrinho estar em repouso esquerda de sua posio inicial. III. A energia mecnica (mgl) quase totalmente transformada em energia trmica. Assinale a alternativa correta. a) Apenas I verdadeira. c) Apenas III verdadeira. e) Apenas II e III so verdadeiras. 8) Durante muito tempo, a partir da Idade Mdia, foram projetadas mquinas, como a da figura a seguir, que seriam capazes de trabalhar perpetuamente. b) Apenas II verdadeira. d) Apenas I e III so verdadeiras.

Prof: Sandro Fernandes

70

(FRISCH, Otto R. "A natureza da matria". Lisboa: Verbo, 1972.) O fracasso desses projetos levou compreenso de que o trabalho no poderia ser criado do nada e contribuiu para a elaborao do conceito fsico de: a) fora d) momento angular b) energia e) momento linear c) velocidade

9) Um pra-quedista est caindo com velocidade constante. Durante essa queda, considerando-se o pra-quedista em relao ao nvel do solo, correto afirmar que a) sua energia potencial gravitacional se mantm constante. b) sua energia potencial gravitacional est aumentando. c) sua energia cintica se mantm constante. d) sua energia cintica est diminuindo. e) a soma da energia cintica e da energia potencial gravitacional constante. 10) Analise as seguintes situaes: 1. Um corpo cai em queda livre. 2. Um corpo desce, com velocidade constante, ao longo de um plano inclinado. 3. Um corpo move-se ao longo de um plano horizontal, at parar. 4. Um corpo mantido em repouso sobre um plano horizontal. 5. Um corpo empurrado ao longo de um plano horizontal sem atrito, aumentando a sua velocidade. Das situaes acima, as nicas nas quais a energia mecnica total do corpo diminui, so: a) 1 e 5 b) 1 e 4 c) 2 e 4 d) 2 e 3 e) 2 e 5

11) Energia um dos conceitos mais importantes de toda a Fsica e, basicamente, significa a capacidade de realizao de alguma forma de trabalho. O conceito de energia est presente em vrios ramos da Fsica, como na Mecnica, na Termodinmica e no Eletromagnetismo. As afirmaes a seguir dizem respeito a diversas aplicaes do conceito de energia na Fsica. Assinale a alternativa que contm uma informao INCORRETA: a) Se duplicarmos a velocidade de um corpo material, sua energia cintica tambm dobrar. b) Numa transformao termodinmica cclica, a variao da energia interna nula. c) Quando uma mola comprimida, o trabalho realizado para tal convertido em energia potencial elstica da mola.

Prof: Sandro Fernandes

71

d) Se fizermos a carga de um capacitor cair metade de seu valor, a energia eltrica armazenada no capacitor diminuir quarta parte. e) Quando um objeto cai de uma determinada altura, sua energia potencial gravitacional convertida gradualmente em energia cintica. 12) Vrios processos fsicos envolvem transformaes entre formas diferentes de energia. Associe a coluna superior com a coluna inferior, e assinale a alternativa que indica corretamente as associaes entre as colunas: Dispositivo mecnico ou gerador: 1. Pilha de rdio 2. Gerador de usina hidreltrica 3. Chuveiro eltrico 4. Alto-falante 5. Mquina a vapor Transformao de tipo de energia: a. Eltrica em Mecnica b. Eltrica em Trmica c. Trmica em Mecnica d. Qumica em Eltrica e. Mecnica em Eltrica a) 1-d, 2-e, 3-b, 4-a, 5-c c) 1-b, 2-e, 3-d, 4-a, 5-c e) 1-b, 2-a, 3-d, 4-e, 5-c 13) Uma partcula est submetida a uma fora com as seguintes caractersticas: seu modulo proporcional ao modulo da velocidade da partcula e atua numa direo perpendicular quela do vetor velocidade. Nestas condies, a energia cintica da partcula deve a) crescer linearmente com o tempo. b) crescer quadrticamente com o tempo. c) diminuir linearmente com o tempo. d) diminuir quadrticamente com o tempo. e) permanecer inalterada. b) 1-d, 2-a, 3-b, 4-e, 5-c d) 1-d, 2-b, 3-c, 4-a, 5-e

Prof: Sandro Fernandes

72

14) A energia trmica liberada em processos de fisso nuclear pode ser utilizada na gerao de vapor para produzir energia mecnica que, por sua vez, ser convertida em energia eltrica. Abaixo est representado um esquema bsico de uma usina de energia nuclear.

A partir do esquema so feitas as seguintes afirmaes: I. a energia liberada na reao usada para ferver a gua que, como vapor a alta presso, aciona a turbina. II. a turbina, que adquire uma energia cintica de rotao, acoplada mecanicamente ao gerador para produo de energia eltrica. III. a gua depois de passar pela turbina pr-aquecida no condensador e bombeada de volta ao reator. Dentre as afirmaes acima, somente est(o) correta(s): a) I. b) II. c) III. d) I e II. e) II e III.

15) No processo de obteno de eletricidade, ocorrem vrias transformaes de energia. Considere duas delas: I. cintica em eltrica II. potencial gravitacional em cintica

Analisando o esquema a seguir, possvel identificar que elas se encontram, respectivamente, entre: a) I - a gua no nvel h e a turbina, II - o gerador e a torre de distribuio. b) I - a gua no nvel h e a turbina, II - a turbina e o gerador. c) I - a turbina e o gerador, II - a turbina e o gerador. d) I - a turbina e o gerador, II - a gua no nvel h e a turbina. e) I - o gerador e a torre de distribuio, II - a gua no nvel h e a turbina.
Prof: Sandro Fernandes

73

16) Na figura a seguir est esquematizado um tipo de usina utilizada na gerao de eletricidade.

Analisando o esquema, possvel identificar que se trata de uma usina: a) hidreltrica, porque a gua corrente baixa a temperatura da turbina. b) hidreltrica, porque a usina faz uso da energia cintica da gua. c) termoeltrica, porque no movimento das turbinas ocorre aquecimento. d) elica, porque a turbina movida pelo movimento da gua. e) nuclear, porque a energia obtida do ncleo das molculas de gua. 17) Jean deixa cair uma bola de tnis de uma altura H medida a partir do solo, considerado como o nvel zero de energia potencial gravitacional. Ao passar pelo ponto de altura h = H/4 sua energia cintica vale 12 J.

Desprezando-se a resistncia do ar, a energia potencial da bola, em joules, no ponto de altura H, vale a) 3,0. b) 12. c) 16. d) 48. e) 56.

18) Um menino desce a rampa de acesso a um terrao dirigindo um carrinho de lomba (carrinho de rolem). A massa do sistema menino-carrinho igual a 80 kg. Utilizando o freio, o menino mantm, enquanto desce, a energia cintica do sistema constante e igual a 160 J. O desnvel entre o incio e o fim da rampa de 8 m. Qual o trabalho que as foras de atrito exercidas sobre o sistema realizam durante a descida da rampa? (Considere a acelerao da gravidade igual a 10 m/s2). a) -6.560 J. b) -6.400 J. c) -5.840 J. d) -800 J. e) -640 J.

Prof: Sandro Fernandes

74

19) Uma bola de massa m = 500 g lanada do solo, com velocidade v 0 e ngulo de lanamento 0, menor que 90. Despreze qualquer movimento de rotao da bola e a influncia do ar. O valor da acelerao da gravidade, no local, g=10 m/s2. O grfico adiante mostra a energia cintica K da bola como funo do seu deslocamento horizontal, x. Analisando o grfico, podemos concluir que a altura mxima atingida pela bola :

a) 60 m

b) 48 m

c) 30 m

d) 18 m

e) 15 m

20) Uma partcula de massa m abandonada do repouso em A e desliza, sem atrito, ao longo de um trilho, conforme a figura. O raio da parte circular, R, equivalente a 1/3 da altura do ponto A. As expresses que determinam a energia cintica nos pontos B, C e D so, respectivamente,

a) 3 mgR; 2 mgR; mgR b) 2 mgR; mgR; 0 c) 3 mgR; mgR; 2 mgR d) mgR; 2 mgR; 3 mgR e) 0; 2 mgR; 3 mgR 21) Um jovem escorrega por um tobog aqutico, com uma rampa retilnea, de comprimento L, como na figura, podendo o atrito ser desprezado. Partindo do alto, sem impulso, ele chega ao final da rampa com uma velocidade de cerca de 6m/s.

Prof: Sandro Fernandes

75

Para que essa velocidade passe a ser de 12 m/s, mantendo-se a inclinao da rampa, ser necessrio que o comprimento dessa rampa passe a ser aproximadamente de a) L/2 b) L c) 1,4 L d) 2 L e) 4 L

22) Um bloco com massa de 0,20kg, inicialmente em repouso, derrubado de uma altura de h=1,20m sobre uma mola cuja constante de fora k=19,6N/m. Desprezando a massa da mola, a distncia mxima que a mola ser comprimida a) 0,24 23) O esquema b) 0,32 c) 0,48 d) 0,54 e) 0,60

abaixo mostra, em termos de potncia (energia/tempo), aproximadamente, o fluxo de

energia, a partir de certa quantidade de combustvel vinda do tanque de gasolina, em um carro viajando com velocidade constante.

1. Evaporao 1kW 2. Energia dos hidrocarbonetos no queimados, energia trmica dos gases de escape e transferida ao ar ambiente 56,8kW 3. Luzes, ventilador, gerador, direo, bomba hidrulica, etc. 2,2kW 4. Energia trmica 3kW O esquema mostra que, na queima da gasolina, no motor de combusto, uma parte considervel de sua energia dissipada. Essa perda da ordem de: a) 80% b) 70% c) 50% d) 30% e) 20%

24) O corpo C, de massa m, abandonado do repouso no ponto A do trilho liso abaixo e, aps realizar o "looping" de raio R, atinge o trecho horizontal. Desprezando qualquer resistncia ao deslocamento e

Prof: Sandro Fernandes

76

sabendo que a acelerao gravitacional local g, o mdulo da quantidade de movimento desse corpo, ao passar pelo ponto B do trilho, :

a) m . (R . g)1/2 d) 5 m . R . g / 2

b) m . R1/2 g e) 2 m. R . g / 5

c) m . g1/2 R

25) A massa m de um pndulo simples, cujo fio tem comprimento L=0,90m, abandonada a partir do repouso quando o fio forma ngulo de 60 com a vertical, como mostra a figura.

g = 10m/s2 Desprezando a resistncia do ar, a velocidade de m, quando o fio fica na posio vertical, , em m/s, a) 1,0 b) 2,0 c) 3,0 d) 4,0 e) 5,0

26) Um objeto de massa 8,0kg e volume 1,0 litro est imerso em um lquido de densidade igual da gua, contido num grande recipiente, como mostra a figura. O objeto se move para baixo com velocidade constante v=0,20m/s, devido ao conjunta da gravidade, do empuxo e da resistncia viscosa do lquido ao movimento. Podemos afirmar que a quantidade de energia transformada em calor, a cada segundo, no sistema "objeto-lquido" de:

a) 0,0 J

b) 0,14 J

c) 0,16 J

d) 14 J

e) 16 J

27) Uma pedra lanada verticalmente para cima com uma energia cintica de 25J, a partir de um ponto A, subindo at um ponto B e retornando ao ponto do lanamento. Em B, a energia potencial da pedra, com relao ao ponto A, de 15J. Entre as afirmativas a seguir, aponte a que est ERRADA, segundo os dados apresentados.
Prof: Sandro Fernandes

77

a) A energia mecnica total da pedra, no ponto A, de 25J. b) No trajeto de ida e volta da pedra, o trabalho total realizado pela fora de resistncia do ar nulo. c) Durante a subida da pedra, o trabalho realizado pela fora de resistncia do ar de -10J. d) Durante a descida da pedra, o trabalho realizado pela fora de resistncia do ar de -10J. e) A energia cintica da pedra, ao retornar ao ponto de lanamento, de 5J. 28) Uma esteira rolante transporta 15 caixas de bebida por minuto, de um depsito no sub-solo at o andar trreo. A esteira tem comprimento de 12m, inclinao de 30 com a horizontal e move-se com velocidade constante. As caixas a serem transportadas j so colocadas com a velocidade da esteira. Se cada caixa pesa 200N, o motor que aciona esse mecanismo deve fornecer a potncia de: a) 20 W b) 40 W c) 300 W d) 600 W e) 1800 W

29) Leia com ateno e analise as afirmativas a seguir: I. O trabalho total realizado sobre um corpo que se desloca entre dois pontos igual variao da energia cintica do corpo entre esses mesmos dois pontos. II. Quando sobre um corpo em movimento atuam somente foras conservativas, sua energia mecnica apresenta o mesmo valor em todos os pontos da trajetria. III. Se a resultante das foras externas que atuam sobre um sistema de partculas for nula, a quantidade de movimento total do sistema se conserva. IV. O trabalho realizado por uma fora conservativa sobre um corpo, que se desloca entre dois pontos, depende da trajetria seguida pelo corpo. Assinale a alternativa CORRETA: a) somente esto corretas as afirmativas I, II e III; b) somente est correta a afirmativa IV; c) somente esto corretas as afirmativas I, III e IV; d) somente esto corretas as afirmativas I, II e IV; e) todas as afirmativas esto corretas. 30) A casa de Dona Maria fica no alto de uma ladeira. O desnvel entre sua casa e a rua que passa no p da ladeira de 20 metros. Dona Maria tem 60kg e sobe a rua com velocidade constante. Quando ela sobe a ladeira trazendo sacolas de compras, sua velocidade menor. E seu corao, quando ela chega casa, est batendo mais rpido. Por esse motivo, quando as sacolas de compras esto pesadas, Dona Maria sobe a ladeira em ziguezague. O fato de Dona Maria subir a ladeira em ziguezague e com velocidade menor est diretamente associado reduo de: a) potncia. d) energia. b) acelerao. e) trabalho. c) deslocamento.

Prof: Sandro Fernandes

78

De acordo com a Matriz de Referncias para o Enem 2009. Professor: Sandro Fernandes Tema: Fenmenos Eltricos e Magnticos

Aula VI de Fsica
(I) O ELETROMAGNETISMO
Parte da fsica que estuda as propriedades eltricas e magnticas da matria, em particular as relaes estabelecidas entre elas. Conta uma lenda grega que o pastor Magnes se surpreendeu ao ver como a bola de ferro de seu basto era atrada por uma pedra misteriosa, o mbar (em grego, elektron). A histria demonstra como antigo o interesse pelos fenmenos eletromagnticos. Denomina-se eletromagnetismo a disciplina cientfica que estuda as propriedades eltricas e magnticas da matria e, em especial, as relaes que se estabelecem entre elas. A existncia de foras naturais de origem eltrica e magntica fora observada em contextos histricos independentes, mas s na primeira metade do sculo XIX um grupo de pesquisadores conseguiu unificar os dois campos de estudo e assentar os alicerces de uma nova concepo da estrutura fsica dos corpos. No final do sculo XVIII Charles-Augustin de Coulomb e Henry Cavendish haviam determinado as leis empricas que regiam o comportamento das substncias eletricamente carregadas e o dos ms. Embora a similaridade entre as caractersticas dos dois fenmenos indicasse uma possvel relao entre eles, s em 1820 se obteve prova experimental dessa relao, quando o dinamarqus Hans Christian Oersted, ao aproximar uma bssola de um fio de arame que unia os dois plos de uma pilha eltrica, descobriu que a agulha imantada da bssola deixava de apontar para o norte, orientando-se para uma direo perpendicular ao arame. Pouco depois, Andr-Marie Ampre demonstrou que duas correntes eltricas exerciam mtua influncia quando circulavam atravs de fios prximos um do outro. Apesar disso, at a publicao, ao longo do sculo XIX, dos trabalhos do ingls Michael Faraday e do escocs James Clerk Maxwell, o eletromagnetismo no foi - nem comeou a ser - considerado um autntico ramo da fsica. Os fenmenos eletromagnticos so produzidos por cargas eltricas em movimento. A carga eltrica, assim como a massa, uma qualidade intrnseca da matria e apresenta a particularidade de existir em duas variedades, convencionalmente denominadas positiva e negativas. A unidade elementar da carga o eltron, partcula atmica de sinal negativo, embora sua magnitude no resulte em entidade suficiente para clculos macroscpicos normais. Como unidade usual de carga usa-se ento o coulomb; o valor da carga de um eltron equivale a 1,60 x 10-19 coulombs. Duas cargas eltricas de mesmo sinal se repelem, e quando de sinais contrrios se atraem. A fora destas interaes diretamente proporcional a sua quantidade de carga e inversamente proporcional ao quadrado da distncia que as separa. Para explicar a existncia dessas foras adotou-se a noo de campo eltrico criado em torno de uma carga, de modo que a fora eltrica que vai atuar sobre outra carga distanciada da primeira corresponde ao produto da quantidade de carga desta primeira por uma grandeza chamada intensidade de campo eltrico. A energia que este campo transmite unidade de carga chama-se potencial eltrico e geralmente se mede em volts.
Prof: Sandro Fernandes

79

Uma das variveis magnticas fundamentais a induo magntica, intimamente relacionada com a intensidade do campo magntico. A induo representa a fora magntica exercida sobre um corpo por unidade de carga eltrica e de velocidade. A unidade de induo magntica o tesla, que equivale a um weber por metro quadrado; o weber uma medida de fluxo magntico (grandeza que reflete a densidade dos campos magnticos). Tanto a intensidade de campo eltrico e magntico quanto a induo magntica apresentam um carter vetorial e, por conseguinte, para descrev-las adequadamente devem-se definir, para cada uma, sua magnitude, direo e sentido. Por correlacionar a eletricidade e o magnetismo, adquiriu funo especial no campo da fsica a noo de corrente eltrica, entendida como a circulao de cargas livres ao longo de um material condutor. Sua magnitude determinada pela intensidade da corrente, que a quantidade de cargas eltricas livres que circulam pelo condutor em um tempo determinado. Chama-se ampre a unidade de intensidade de corrente resultante da passagem em um condutor de um coulomb de carga durante um segundo. Essa unidade tornou-se a mais importante do ponto de vista eletromagntico, levando o sistema internacional de unidades a ter a notao MKSA: metro, quilograma, segundo, ampre. No decorrer do sculo XIX, as experincias de rsted e Ampre demonstraram a influncia que as correntes eltricas exercem sobre os materiais imantados, enquanto Faraday e Joseph Henry determinaram a natureza das correntes eltricas induzidas por campos magnticos variveis no espao. Os resultados de suas pesquisas, fundamento da induo eletromagntica, constituem a base do eletromagnetismo. Outros postulados enunciam a existncia de dois plos eltricos, positivo e negativo, independentes e separados, e de dois plos magnticos inseparveis de nomes diferentes (norte e sul). Ampre, estimulado pelas descobertas de rsted, aprofundou-se na pesquisa das foras magnticas provocadas nas proximidades de uma corrente eltrica e demonstrou que esses impulsos se incrementam na razo direta da corrente e na razo inversa da distncia ao fio pelo qual ela circula. Comprovou, alm disso, que as foras induzidas esto em grande medida condicionadas pela orientao do fio condutor. Ao aproximar-se um m de uma pilha eltrica observa-se uma variao em sua fora eletromotriz, que a medida da energia fornecida a partir de cada unidade de carga eltrica nela contida. Essa alterao interrompida quando se imobiliza o m, e adquire sinal contrrio quando este afastado. Deduz-se da que os campos magnticos produzem correntes eltricas em um circuito e que o sentido de seu fluxo tende a compensar a perturbao exterior, com a induo simultnea de um campo magntico oposto ao inicial. Analogamente, uma corrente eltrica que circula em um condutor gera um campo magntico associado que, como efeito derivado, induz no condutor uma corrente de sentido contrrio ao da inicial. Esse fenmeno conhecido como auto-induo, e a relao entre o campo magntico e a intensidade da corrente induzida por ele fornecida por um coeficiente denominado indutncia, que depende das caractersticas fsicas e geomtricas do material condutor. A unidade de medida de induo o henry, definido como a grandeza gerada entre dois circuitos dispostos de forma tal que quando num deles a intensidade varia em um ampre por segundo seja induzida no outro uma fora eletromotriz de um volt. Desde o advento das idias inovadoras de Isaac Newton, estabeleceu-se uma interpretao causal do universo segundo a qual todo efeito observado obedeceria a foras exercidas por objetos situados a certa distncia. Nesse contexto histrico nasceu a teoria eletromagntica, segundo a qual as atraes e repulses eltricas e magnticas resultavam da ao de corpos distantes. Era preciso, pois, encontrar a verdadeira causa final dessas foras, buscando-se uma analogia com a massa gravitacional de Newton e,
Prof: Sandro Fernandes

80

simultaneamente, explicar de forma rigorosa os mecanismos de interao eletromagntica entre os corpos. Coube a Ampre, a partir de seus trabalhos sobre correntes eltricas, expor a teoria da existncia de partculas eltricas elementares que, ao se deslocar no interior das substncias, causariam tambm os efeitos magnticos. No entanto, em suas experincias, ele no conseguiu encontrar essas partculas. Por outro lado, Faraday introduziu a noo de campo, que teve logo grande aceitao e constituiu um marco no desenvolvimento da fsica moderna. Concebeu o espao como cheio de linhas de fora -correntes invisveis de energia que governavam o movimento dos corpos e eram criadas pela prpria presena dos objetos. Assim, uma carga eltrica mvel produz perturbaes eletromagnticas a seu redor, de modo que qualquer outra carga prxima detecta sua presena por meio das linhas do campo. Esse conceito foi desenvolvido matematicamente pelo britnico James Clerk Maxwell, e a fora de seus argumentos acabou com a da idia de foras que agiam sob controle remoto, vigente em sua poca. Os mltiplos trabalhos tericos sobre o eletromagnetismo culminaram em 1897, quando Sir Joseph John Thomson descobriu o eltron, cuja existncia foi deduzida do desvio dos raios catdicos na presena de um campo eltrico. A natureza do eletromagnetismo foi confirmada ao se determinar a origem das foras magnticas no movimento orbital dos eltrons ao redor dos ncleos dos tomos. O conceito de onda eletromagntica, apresentado por Maxwell em 1864 e confirmado experimentalmente por Heinrich Hertz em 1886, utilizado para demonstrar a natureza eletromagntica da luz. Quando uma carga eltrica se desloca no espao, a ela se associam um campo eltrico e outro magntico, interdependentes e com linhas de fora perpendiculares entre si. O resultado desse conjunto uma onda eletromagntica que emerge da partcula e, em condies ideais - isto , sem a interveno de qualquer fator de perturbao - se move a uma velocidade de 299.793km/s, em forma de radiao luminosa. A energia transportada pela onda proporcional intensidade dos campos eltrico e magntico da partcula emissora e fixa as diferentes freqncias do espectro eletromagntico. A teoria eletromagntica muito usada na construo de geradores de energia eltrica, dentre estes se destacam os alternadores ou geradores de corrente alternada, que propiciam maior rendimento que os de corrente contnua por no sofrerem perdas mediante atrito. A base do alternador o eletrom, ncleo em geral de ferro doce e em torno do qual se enrola um fio condutor revestido de cobertura isolante. O dispositivo gira a grande velocidade, de modo que os plos magnticos mudam de sentido e induzem correntes eltricas que se invertem a cada instante. Com isso, as cargas circulam vrias vezes pela mesma seo do condutor. Os eletroms tambm so utilizados na fabricao de elevadores e instrumentos cirrgicos e teraputicos. Seu uso abrange diversos campos industriais, uma vez que os campos que geram podem mudar de direo e de intensidade
http://www.sbf1.sbfisica.org.br/eventos/ebm

Prof: Sandro Fernandes

81

(II)

CAPACITORES

Um Capacitor ou Condensador constitudo por duas placas metlicas condutoras (as armaduras), dispostas uma paralela outra e separadas por um material isolante (o dieltrico). Utiliza-se como dieltrico o papel, a cermica, a mica, os materiais plsticos, vidro, parafina ou mesmo o ar. O Capacitor dispositivo muito usado em circuitos eltricos. Este aparelho destinado a armazenar cargas eltricas e constitudo por dois condutores separados por um isolante: os condutores so chamados armaduras (ou placas) do capacitor e o isolante o dieltrico do capacitor. A quantidade de carga armazenada na placa de um capacitor diretamente proporcional diferena de potencial entre as placas. O quociente entre carga (Q) e diferena de potencial (U) ento uma constante para um determinado capacitor e recebe o nome de capacitncia (C).

Q U C Capacitncia medida em Farad (F) Q Carga eltrica medida em Coulomb (C) C U Tenso eltrica medida em Volt (U)

Os capacitores so empregados nos mais variados circuitos eltricos e desempenham sempre um papel muito importante, que o de armazenar cargas eltricas para depois descarreg-las em um determinado momento especfico. Eles so utilizados, por exemplo, em circuitos retificadores, circuitos ressonantes e em divisores de freqncias. Em um rdio, a antena capta as ondas que so emitidas pelas estaes transmissoras e cada estao possui uma freqncia determinada. Na antena h um receptor que sintoniza inmeras estaes graas ao circuito ressonante. Esse circuito transforma corrente alternada em corrente contnua e constitudo basicamente por um capacitor varivel que fica em paralelo com uma bobina. Para cada valor de capacitncia do capacitor, o receptor ajusta o aparelho de rdio ao comprimento de onda que transmitido pela emissora de rdio, ou seja, ele sintoniza a estao de rdio que corresponde a uma freqncia de onda especfica. Os capacitores tm uma propriedade que a de bloquear correntes contnuas e alternadas de baixas freqncias e facilitar a passagem de correntes alternadas de altas freqncias. Essa propriedade utilizada para separar sons agudos de uma msica, por exemplo, encaminhando esses sons para os alto-falantes que so adequados para fazer a reproduo desse tipo de som. Esses auto-falantes so chamados de tweeter. Os sons graves so sons de baixas freqncias, e eles so reproduzidos pelos chamados woofers. Um capacitor, com capacitncia e tipo adequado, faz o bloqueio dessas baixas freqncias deixando passar somente os sons de freqncias mais elevadas, que so os sons agudos. Dessa forma, ocorre a separao de sons agudos e graves

Prof: Sandro Fernandes

82

(III)

QUAL A DIFERENA ENTRE CORRENTE CONTNUA E CORRENTE ALTERNADA?

A diferena o sentido da tal corrente. Uma corrente eltrica nada mais que um fluxo de eltrons (partculas que carregam energia) passando por um fio, algo como a gua que circula dentro de uma mangueira. Se os eltrons se movimentam num nico sentido, essa corrente chamada de contnua. Se eles mudam de direo constantemente, estamos falando de uma corrente alternada. Na prtica, a diferena entre elas est na capacidade de transmitir energia para locais distantes. A energia que usamos em casa produzida por alguma usina e precisa percorrer centenas de quilmetros at chegar tomada. Quando essa energia transmitida por uma corrente alternada, no h muita perda no meio caminho. J na contnua o desperdcio muito grande. Isso porque a corrente alternada pode, facilmente, ficar com uma voltagem muito mais alta que a contnua, e quanto maior essa voltagem, mais longe a energia chega sem perder fora no trajeto. Se todos os sistemas de transmisso fossem em corrente contnua, seria preciso uma usina em cada bairro para abastecer as casas com eletricidade. O nico problema da alta voltagem transportada pela corrente alternada que ela poderia provocar choques fatais dentro das residncias. "Por isso, a alta voltagem transformada no final em tenses baixas. As mais comuns so as de 127 ou 220 volts", diz o fsico Cludio Furukawa, da USP. Portanto, a corrente que chega tomada de sua casa continua sendo alternada, mas com uma voltagem bem mais baixa. J a corrente contnua sai, por exemplo, de pilhas e baterias, pois a energia gerada por elas, usada nos prprios aparelhos que as carregam, no precisa ir longe. Tambm h muitos equipamentos eletrnicos que s funcionam com corrente contnua, possuindo transformadores internos, que adaptam a corrente alternada que chega pela tomada. Mo simples e mo dupla Modo como os eltrons se movem determina o tipo de corrente Alternada Nesse tipo de corrente, o fluxo de eltrons que carrega a energia eltrica dentro de um fio no segue um sentido nico. Ora os eltrons vo para frente, ora para trs, mudando de rota 120 vezes por segundo. Essa variao fundamental, pois os transformadores que existem numa linha de transmisso s funcionam recebendo esse fluxo de eltrons alternado. Dentro do transformador, a voltagem da energia transmitida aumentada, permitindo que ela viaje longe, desde uma usina at a sua casa Contnua Aqui o fluxo de eltrons passa pelo fio sempre no mesmo sentido. Como no h alternncia, essa corrente no aceita pelos transformadores e no ganha voltagem maior. Resultado: a energia eltrica no pode seguir muito longe. Por isso, a corrente contnua usada em pilhas e baterias ou para percorrer circuitos internos de aparelhos eltricos, como um chuveiro. Mas ela no serve para transportar energia entre uma usina e uma cidade
Fonte: http://mundoestranho.abril.com.br/ciencia/pergunta_286391.shtml

Prof: Sandro Fernandes

83

Algumas Aplicaes... 1) Este diagrama mostra um pndulo com uma placa de cobre presa em sua extremidade.

Esse pndulo pode oscilar livremente, mas, quando a placa de cobre colocada entre os plos de um im forte, ele pra de oscilar rapidamente. Isso ocorre porque a) a placa de cobre fica ionizada. b) a placa de cobre fica eletricamente carregada. c) correntes eltricas so induzidas na placa de cobre. d) os tomos de cobre ficam eletricamente polarizados. e) os eltrons livres da placa de cobre so atrados eletrostaticamente pelos plos do m 2) Pares de ims em forma de barra so dispostos conforme indicam as figuras a seguir:

A letra N indica o plo Norte e o S o plo Sul de cada uma das barras. Entre os ims de cada um dos pares anteriores (a), (b) e (c) ocorrero, respectivamente, foras de: a) atrao, repulso, repulso; c) atrao, repulso, atrao; e) repulso, atrao, atrao. 3) Considere os experimentos: EXPERIMENTO 1 Um carrinho de material isolante colocado sobre trilhos e preso a duas molas. Sobre ele fixado um m, conforme a figura A. O conjunto deslocado 20 cm direita, em seguida liberado. Ocorre ento, um movimento oscilatrio, diminuindo gradativamente de amplitude at o repouso, devido s foras de atrito.
Prof: Sandro Fernandes

b) atrao, atrao, repulso; d) repulso, repulso, atrao;

84

EXPERIMENTO 2 O experimento repetido fixando-se uma bobina (figura B). Isto faz com que, durante o movimento oscilatrio, o m penetre no interior da bobina, sem no entanto toc-la, no havendo portanto nenhuma fora adicional de atrito. G um galvanmetro ligado bobina.

Analise as proposies: I- O experimento 2 proporcionar o aparecimento de uma corrente induzida na bobina. II- O experimento 2 proporcionar o aparecimento, na bobina, de uma corrente induzida sempre no mesmo sentido. III- A presena da bobina dar origem a foras magnticas, diminuindo o tempo para o sistema entrar em repouso relativamente ao experimento 1. IV- O tempo para o sistema entrar em repouso no primeiro experimento o mesmo que no segundo experimento. Est correta ou esto corretas: a) Somente II. d) II e IV. b) I e IV. e) Somente I. c) I e III.

4) Considere as seguintes afirmativas I. Um prego ser atrado por um m somente se j estiver imantado. II. As linhas de fora de um campo magntico so fechadas. III. Correntes eltricas fluindo por dois condutores paralelos provocam fora magntica entre eles. Pode-se afirmar que SOMENTE a) I correta d) I e II so corretas. b) II correta. e) II e III so corretas. c) III correta.

5) Uma partcula de carga q entra com velocidade V 0 numa regio onde existe um campo magntico uniforme B.

Prof: Sandro Fernandes

85

No caso em que V0 e B possuem a mesma direo, podemos afirmar que a partcula a) sofrer um desvio para sua direita. b) sofrer um desvio para sua esquerda. c) ser acelerada na direo do campo magntico uniforme B. d) no sentir a ao do campo magntico uniforme B. e) ser desacelerada na direo do campo magntico uniforme B. 6) A utilizao de cabos eletrodinmicos no espao tem sido considerada como uma das alternativas para a gerao da energia eltrica necessria em satlites e estaes espaciais. Isso pode ser conseguido com a utilizao de duas massas, separadas por um cabo metlico de alguns quilmetros. Os satlites devem se mover em uma rbita baixa, situada entre 200 e 2000 km da superfcie terrestre. Nessa regio, existe um campo magntico terrestre suficientemente forte para induzir corrente eltrica no cabo, a atmosfera muito rarefeita e perdas por atrito so mnimas. Desconsiderando-se detalhes tcnicos do processo, alguns elementos fundamentais a um projeto deste tipo podem ser visualizados na figura a seguir. Nesta, as duas massas ligadas por um fio vertical descrevem rbitas com mesma velocidade angular no plano equatorial. O vetor campo magntico terrestre e o vetor velocidade do fio, em um de seus pontos, esto representados.

Com base na figura e nos conhecimentos sobre o tema, correto afirmar que estabelecer-se- uma corrente para: a) Cima e o fio condutor sofrer a influncia de uma fora magntica na direo e sentido de sua velocidade. b) Baixo e o fio condutor sofrer a influncia de uma fora magntica na direo e sentido de sua velocidade.

Prof: Sandro Fernandes

86

c) Cima e o fio condutor sofrer a influncia de uma fora magntica na direo de sua velocidade e de sentido contrrio a esta. d) Baixo e o fio condutor sofrer a influncia de uma fora magntica na direo de sua velocidade e de sentido contrrio a esta. e) Baixo e o fio condutor no sofrer a influncia de nenhuma fora magntica na direo de sua velocidade. 7) Um velho caminho a gasolina necessita de alta tenso nas velas, para produzir as fascas que iniciam o processo de queima desse combustvel nas cmaras de combusto, sendo essa tenso obtida por meio de uma bobina de induo. A transformao de baixa tenso para alta tenso est baseada na lei de a) Coulomb. d) Ampre. b) Ohm. e) Faraday. c) Kirchhoff.

8) Uma interessante e histrica experincia foi realizada pelo fsico dinamarqus Hans Christian Oersted, em 1820, ao utilizar uma pilha conectada aos extremos de um condutor metlico nas proximidades de uma bssola, cuja agulha estava orientada inicialmente na direo norte-sul do campo magntico terrestre. Com o estabelecimento da corrente eltrica no condutor, Oersted pde perceber que a agulha da bssola se desviava em relao a sua orientao inicial. Os resultados dessa experincia permitiram concluir corretamente que a) uma mesma teoria passaria a dar conta de fenmenos eltricos e magnticos, at ento considerados independentes um do outro. b) os plos da agulha da bssola so inseparveis. c) as correntes eltricas so estabelecidas apenas em condutores metlicos. d) os plos da pilha so os responsveis pela alterao do alinhamento original da bssola. e) o campo magntico terrestre afeta a corrente eltrica no condutor. 9) Grande parte dos aparelhos eltricos que usamos tm a funo de produzir movimento, a partir da eletricidade. Entre eles, esto: batedeira, liqidificador, ventilador, aspirador de p... alm de inmeros brinquedos movidos a pilha, como robs, carrinhos... Outros so igualmente utilizados para o conforto humano, como os aquecedores de ambiente e de gua. O alto consumo de energia eltrica, porm aliado pouca quantidade de chuvas, levou algumas regies do pas, em 2001, ao famoso "racionamento de energia", que trouxe, como lio, a indispensabilidade do consumo racional e consciente da energia eltrica. GREF. "Eletricidade". vol. 3 [adapt.] Com base no texto, considere que um fio condutor percorrido por uma corrente constante, "i" e que o campo magntico gerado por essa corrente, a uma distncia "r", "B".

Prof: Sandro Fernandes

87

Esse campo magntico, se a distncia for reduzida tera parte (r/3), ter o valor de a) B. b) B/3. c) 6B. d) 3B. e) B/6.

10) Um m, preso a um carrinho, desloca-se com velocidade constante ao longo de um trilho horizontal. Envolvendo o trilho h uma espira metlica, como mostra a figura adiante. Pode-se afirmar que, na espira, a corrente eltrica:

a) sempre nula. b) existe somente quando o m se aproxima da espira. c) existe somente quando o m est dentro da espira. d) existe somente quando o m se afasta da espira. e) existe quando o m se aproxima ou se afasta da espira. 11) Das afirmativas a seguir, a mais correta : a) A f.e.m. induzida num circuito igual ao quociente da variao do fluxo magntico pelo inverso do tempo decorrido nesta variao. b) O sentido da corrente induzida tal que seus efeitos so a favor da variao do fluxo que lhe deu origem. c) O fluxo magntico o n0 de linhas de campo que no passa atravs da superfcie da espira. d) O n0 de linhas de campo que atravessam uma espira s depende do vetor campo magntico. e) Dois fios paralelos, percorridos por correntes eltricas de sentidos contrrios atraem-se. 12) Um fio metlico retilneo de massa 50g e comprimento MN=50 cm, suspenso por um dinammetro D de massa desprezvel e mantido em equilbrio na direo horizontal numa regio onde existe um campo de induo magntica uniforme B de intensidade 0,040T. Se o fio se encontra perpendicularmente s linhas de induo, quando a intensidade da corrente eltrica indicada na figura 20A, o dinammetro assinala:

Prof: Sandro Fernandes

88

Adote g = 10m/s2 a) 0,1 N. b) 0,2 N. c) 0,4 N. d) 0,5 N. e) 0,9 N.

13) Se no laboratrio dispomos somente de capacitores de 2nF, ento o nmero mnimo destes dispositivos que devemos associar para obtermos uma capacitncia equivalente de 9nF : a) 4 b) 3 c) 5 d) 7 e) 6

14) Um capacitor de placas planas e paralelas totalmente carregado utilizando-se uma fonte de 12 volts em trs situaes diferentes. Na situao A, ele permanece vazio. Em B, um dieltrico preenche metade do volume entre as placas e, em C, o mesmo dieltrico preenche todo o volume entre as placas.

Assim, com relao s cargas acumuladas, CORRETO afirmar que: a) as cargas em A, B e C tero o mesmo valor. b) A ter a maior carga e C, a menor. c) A ter a menor carga e C, a maior. d) B ter a maior carga e A, a menor. e) B ter a menor carga e C, a maior. 15) A energia armazenada pela associao de 3 capacitores de mesmo valor nominal, mostrada a seguir, 0,1J. A capacitncia de cada capacitor :

a) 10 F

b) 15 F

c) 20 F

d) 25 F

e) 30 F

Prof: Sandro Fernandes

89

16) Um capacitor ideal de placas planas paralelas carregado mediante a aplicao de uma d.d.p. entre suas placas. A distncia entre as placas ento duplicada, mantendo-se a mesma d.d.p. entre elas. Nessa nova situao, a carga nas placas _________ e a energia eletrosttica armazenada no capacitor __________. Preencher CORRETAMENTE as lacunas, na seqncia em que aparecem na frase acima: a) dobra - reduz-se metade c) reduz-se metade - reduz-se metade e) reduz-se metade - no se altera 17) A foto mostra uma lanterna sem pilhas, recentemente lanada no mercado. Ela funciona transformando em energia eltrica a energia cintica que lhe fornecida pelo usurio - para isso ele deve agit-la fortemente na direo do seu comprimento. Como o interior dessa lanterna visvel, pode-se ver como funciona: ao agit-la, o usurio faz um m cilndrico atravessar uma bobina para frente e para trs. O movimento do m atravs da bobina faz aparecer nela uma corrente induzida que percorre e acende a lmpada. b) no se altera - dobra d) dobra - dobra

O princpio fsico em que se baseia essa lanterna e a corrente induzida na bobina so, respectivamente: a) induo eletromagntica; corrente alternada. b) induo eletromagntica; corrente contnua. c) lei de Coulomb; corrente contnua. d) lei de Coulomb; corrente alternada. e) lei de Ampere; correntes alternada ou contnua podem ser induzidas. 18) ENERGIA A quase totalidade da energia utilizada na Terra tem sua origem nas radiaes que recebemos do Sol. Uma parte aproveitada diretamente dessas radiaes (iluminao, aquecedores e baterias solares, etc.) e outra parte, bem mais ampla, transformada e armazenada sob diversas formas antes de ser usada (carvo, petrleo, energia elica, hidrulica, etc). A energia primitiva, presente na formao do universo e armazenada nos elementos qumicos existentes em nosso planeta, fornece, tambm, uma frao da energia que utilizamos (reaes nucleares nos reatores atmicos, etc). (Antnio Mximo e Beatriz Alvarenga. "Curso de Fsica". v.2. S. Paulo: Scipione, 1997. p. 433)

Prof: Sandro Fernandes

90

A enguia eltrica ou poraqu, peixe de gua doce da regio amaznica chega a ter 2,5 m de comprimento e 25 cm de dimetro. Na cauda, que ocupa cerca de quatro quintos do seu comprimento, est situada a sua fonte de tenso - as eletroplacas. Dependendo do tamanho e da vitalidade do animal, essas eletroplacas podem gerar uma tenso de 600V e uma corrente de 2,0A, em pulsos que duram cerca de 3,0 milsimos de segundo, descarga suficiente para atordoar uma pessoa ou matar pequenos animais. (Adaptado de Alberto Gaspar. "Fsica". v.3. So Paulo: tica, 2000, p. 135) Numa descarga eltrica da enguia sobre um animal, o nmero de cargas eltricas elementares que percorre o corpo do animal, a cada pulso, pode ser estimado em: Dado: carga elementar = 1,6 . 10-19 C a) 5 . 106 b) 1 . 109 c) 2 . 1012 d) 4 . 1016 e) 8 . 1018

19) Quando uma corrente eltrica passa por um condutor ela provoca alguns efeitos muito importantes. Considere os seguintes efeitos da corrente eltrica: I. Efeito Joule ou trmico: um condutor percorrido por corrente eltrica sofre um aquecimento. II. Efeito qumico: uma soluo eletroltica sofre decomposio quando percorrida por corrente eltrica. III. Efeito luminoso: a passagem da corrente eltrica atravs de um gs rarefeito, sob baixa presso. IV. Efeito fisiolgico: a corrente eltrica ao atravessar organismos vivos produz contraes musculares. V. Efeito magntico: um condutor percorrido por corrente eltrica cria, na regio prxima a ele, um campo magntico. Na nossa residncia, os efeitos que sempre acompanham a corrente eltrica so a) I e II b) II e III c) III e IV d) IV e V e) V e I

20) Um eletricista tem uma tarefa para resolver: precisa instalar trs lmpadas, cujas especificaes so 60W e 110V, em uma residncia onde a tenso 220V. A figura a seguir representa os trs esquemas considerados por ele.

Analisando os elementos da figura, correto concluir que, no esquema

Prof: Sandro Fernandes

91

a) 1, todas as lmpadas queimaro. b) 2, duas lmpadas queimaro, e a outra ter seu brilho diminudo. c) 3, todas as lmpadas tero seu brilho diminudo. d) 1, s uma das lmpadas queimar, e as outras no acendero. e) 2, duas lmpadas exibiro brilho normal. 21) No lustre da sala de uma residncia, cuja tenso de entrada de 110 V, esto colocadas duas lmpadas "queimadas" de potncia nominal igual a 200 W cada, fabricadas para funcionarem ligadas rede de 220 V. Para substituir as "queimadas" por uma nica, que ilumine o ambiente da mesma forma que as duas lmpadas anteriores iluminavam, ser preciso que a especificao desta nova lmpada seja de
a) 400 W - 110 V b) 200 W - 110 V c) 200 W - 220 V d) 100 W - 110 V e) 100 W - 220 V

22) Um circuito com 3 resistores iguais submetido a uma diferena de potencial V entre os pontos A e C, conforme mostra a figura.

A diferena de potencial que se estabelece entre os pontos A e B a) V/4 b) V/3 c) V/2 d) 2/3 V e) 3/2 V

23) O consumo total de energia nas residncias brasileiras envolve diversas fontes, como eletricidade, gs de cozinha, lenha, etc. O grfico mostra a evoluo do consumo de energia eltrica residencial, comparada com o consumo total de energia residencial, de 1970 a 1995.

Verifica-se que a participao percentual da energia eltrica no total de energia gasto nas residncias brasileiras cresceu entre 1970 e 1995, passando, aproximadamente, de a) 10% para 40%. e) 40% para 80%. 24) As companhias de eletricidade geralmente usam medidores calibrados em quilowatt-hora (kWh). Um kWh representa o trabalho realizado por uma mquina desenvolvendo potncia igual a 1 kW
Prof: Sandro Fernandes

b) 10% para 60%.

c) 20% para 60%.

d) 25% para 35%.

92

durante 1 hora. Numa conta mensal de energia eltrica de uma residncia com 4 moradores, lem-se, entre outros, os seguintes valores: CONSUMO (kWh) - 300 TOTAL A PAGAR (R$) - 75,00 Cada um dos 4 moradores toma um banho dirio, um de cada vez, num chuveiro eltrico de 3 kW. Se cada banho tem durao de 5 minutos, o custo ao final de um ms (30 dias) da energia consumida pelo chuveiro de a) R$ 4,50. b) R$ 7,50. c) R$ 15,00. d) R$ 22,50. e) R$ 45,00.

25) Seguem a seguir alguns trechos de uma matria da revista Superinteressante, que descreve hbitos de um morador de Barcelona (Espanha), relacionando-os com o consumo de energia e efeitos sobre o ambiente. I. Apenas no banho matinal, por exemplo, um cidado utiliza cerca de 50 litros de gua, que depois ter que ser tratada. Alm disso, a gua aquecida consumindo 1,5 quilowatt-hora (cerca de 1,3 milhes de calorias), e para gerar essa energia foi preciso perturbar o ambiente de alguma maneira... II. Na hora de ir para o trabalho, o percurso mdio dos moradores de Barcelona mostra que o carro libera 90 gramas do venenoso monxido de carbono e 25 gramas de xidos de nitrognio... Ao mesmo tempo, o carro consome combustvel equivalente a 8,9kwh. III. Na hora de recolher o lixo domstico... quase 1kg por dia. Em cada quilo h aproximadamente 240 gramas de papel, papelo e embalagens; 80 gramas de plstico; 55 gramas de metal, 40 gramas de material biodegradvel e 80 gramas de vidro. Com relao ao trecho I, supondo a existncia de um chuveiro eltrico, pode-se afirmar que: a) a energia usada para aquecer o chuveiro de origem qumica, transformando-se em energia eltrica. b) a energia eltrica transformada no chuveiro em energia mecnica e, posteriormente, em energia trmica. c) o aquecimento da gua deve-se resistncia do chuveiro, onde a energia eltrica transformada em energia trmica. d) a energia trmica consumida nesse banho posteriormente transformada em energia eltrica. e) como a gerao da energia perturba o ambiente, pode-se concluir que sua fonte algum derivado do petrleo.

Prof: Sandro Fernandes

93

De acordo com a Matriz de Referncias para o Enem 2009. Professor: Sandro Fernandes Tema: Oscilaes, Ondas, pticas e Radiao.

Aula VII de Fsica


(I) A Luneta, Galileu e o ano de 2009
A Astronomia provavelmente a cincia natural mais antiga, datando a pocas da antiguidade, com suas origens em praticas religiosas pr-histricas: vestgios dessas prticas que ainda so encontrados na astrologia, uma disciplina que por muito tempo foi entrelaada com a astronomia e, no mundo ocidental, no muito diferente da mesma at aproximadamente 1750-1800. A astronomia antiga envolvia-se em observar os padres regulares dos movimentos de objetos celestiais visveis, especialmente o Sol, a Lua, estrelas, e os planetas vistos olho nu. Um exemplo da astronomia antiga poderia envolver o estudo da mudana da posio do Sol ao longo do horizonte ou as mudanas nos aparecimentos de estrelas no curso de um ano, o que poderia ser usado para estabelecer um calendrio ritualstico ou agrcola. Em algumas culturas os dados obtidos eram usados em prognsticos astrolgicos. Astrnomos da antiguidade eram capazes de diferenciar entre uma estrela e uma planeta, j que as estrelas permaneciam relativamente fixas durante os sculos enquanto planetas moviam-se consideravelmente em um tempo comparativamente menor. H apenas uma dcada a Igreja Catlica se posicionou contra um erro do passado: perdoou o italiano Galileu Galilei, que foi excomungado durante a Inquisio por causa de suas descobertas astronmicas. O reconhecimento, mesmo que tardio, s refora a importncia do cientista no campo do conhecimento. Em homenagem aos quatro sculos de observaes telescpicas feitas por Galileu, 2009 foi declarado como o Ano Internacional de Astronomia. Coincidentemente, tambm vai lembrar dos 40 anos da conquista da Lua pelos tripulantes do Apollo 8, em 1969. Galileu um marco da astronomia no s pelos seus feitos, mas tambm porque conseguiu expor suas ideias durante uma poca atribulada. Quase foi queimado em praa pblica, mas se livrou do castigo porque aceitou afirmar, mesmo sabendo no ser verdade, que era a Terra que estava no centro do Universo. Mas teve de ficar em priso domiciliar at morrer. Com a luneta que Galileu conseguiu fabricar a partir da justaposio de duas lentes de culos, ele fez as primeiras observaes das manchas solares o que lhe custou a viso de um dos olhos. Analisou ainda Vnus, os anis de Saturno e as Luas de Jpiter. Para resgatar as descobertas de Galileu, o Ano Internacional da Astronomia est com uma srie de programaes especiais mundo afora. H uma meta para ser atingida: colocar um milho de brasileiros, ao longo do ano, olhando por telescpios os mesmos astros observados por Galileu.

Prof: Sandro Fernandes

94

Segundo o engenheiro Bertoldo Schneider, professor pesquisador da Universidade Tecnolgica Federal do Paran (UTFPR), alm de levar a Astronomia populao, a rea enfrenta outros desafios. Continuamos a buscar vidas em outros planetas, porque so grandes as chances de isso acontecer, diz. H outra questo ainda no respondida, que o grande mistrio do sculo: cientistas descobriram que o universo est crescendo de forma acelerada, que as estrelas esto cada vez mais distantes umas das outras. No se sabe, entretanto, por que isso est acontecendo. O que conhecemos apenas 5% de tudo o que existe. Precisamos desvendar as entidades energia e matria escura, explica Schneider
http://portal.rpc.com.br/gazetadopovo/vidaecidadania

(II)

Funcionamento de um RADAR

Radar, do termo em ingls Radia Detection and Rangig, um aparelho utilizado para localizar objetos a longa distncia. A deteco de objetos feita a partir das ondas eletromagnticas que os objetos emitem, permitindo que os mesmos sejam localizados. Esse equipamento comeou a ser utilizado na dcada de 30 para descobrir e localizar objetos a longa distncia utilizando para isso a reflexo das ondas de rdio, principalmente para fins militares. O princpio de funcionamento do radar bem simples. Eles funcionam atravs das ondas de rdio. Elas se descolam a uma velocidade de 300 000 km/s, uma velocidade muito rpida, e so capazes de cobrir grandes distncias. O radar constitudo de uma antena transmissora receptora de sinais de alta freqncia, a transmisso ocorre atravs de pulsos eletromagnticos de alta potncia, curto perodo e feixe curto. Esse feixe ao ser propagado se alarga, ganhando a forma de um cone at atingir o alvo que est sendo monitorado. Aps atingir o alvo, o sinal ento refletido e a antena passa a ser receptora de sinais. Com a velocidade de propagao do pulso e o tempo gasto para o eco chegar, possvel calcular com exata preciso a localizao do objeto. O radar tem aplicao em muitas reas, principalmente na militar. No exrcito, na aeronutica, na meteorologia e na marinha esse equipamento muito utilizado. Na rea das aplicaes cientficas eles so utilizados para localizar objetos espaciais como satlites ou qualquer outro objeto que esteja na rbita da Terra.

(III)

Radiao Eletromagntica

As ondas de radiao eletromagntica so uma juno de campo magntico com campo eltrico que se propagam no vcuo transportando energia. A luz um exemplo de radiao eletromagntica. Esse conceito foi primeiramente estudado por James Clerk Maxwell e depois afirmado por Heinrich Hertz. Maxwell foi fsico e matemtico escocs que ficou conhecido por dar forma final teoria do eletromagnetismo, teoria essa que une o magnetismo, a eletricidade e a ptica. Dessa teoria surgem as equaes de Maxwell, assim chamadas em sua homenagem e porque ele foi o primeiro a descrev-las, juntando a lei de Ampre, a lei de Gauss e a Lei da induo de Faraday. A radiao eletromagntica se propaga no espao. Ela possui campo magntico e campo eltrico que se geram mutuamente e se propagam perpendicularmente um em relao ao outro e na direo de propagao da energia, transportando assim energia sob a forma de radiao eletromagntica. A
Prof: Sandro Fernandes

95

radiao eletromagntica varia conforme a freqncia da onda. A luz visvel aos olhos humanos uma radiao eletromagntica, assim como os raios x, a nica diferena entre essas duas formas de radiao est na faixa de freqncia que o olho humano consegue visualizar, ou seja, os raios x tm faixa de freqncia que fica fora do alcance da viso humana. As ondas do forno de microondas tambm so ondas eletromagnticas. Os campos magnticos e eltricos obedecem ao princpio da superposio. Os vetores campo magntico e campo eltrico se cruzam e criam o fenmeno da reflexo e refrao. A luz uma onda eletromagntica e em um meio no linear como um cristal, por exemplo, pode sofrer interferncias e causar o efeito Faraday aonde a onda pode ser dividida em duas partes com velocidades diferentes. Na refrao, uma onda ao passar de um meio para outro, com densidade diferente, tem a sua velocidade e direo alterada. Uma fonte de radiao, como o Sol, por exemplo, pode emitir luz dentro de um espectro varivel. A luz solar ao ser decomposta em um prisma possibilita a visualizao de espectros de vrias cores, como no arco-ris.

(IV)

Cientistas Tentam Recriar o Big Bang

Cientistas de mais de 50 pases participaram da construo do maior acelerador de partculas do mundo, uma mquina gigantesca que levou cerca de quatorze anos para ser construda e um gasto de aproximadamente US$ 8 bilhes. Essa parece ser a mais ambiciosa experincia de todos os tempos, experincia que tem como foco principal a recriao do Big Bang, a exploso que teria dado origem ao Universo. Localizada entre a Frana e a Sua, essa mquina est construda em uma profundidade de cem metros e tem extenso de pouco mais de 27 quilmetros. Essa aparelhagem possui quatro pontos principais nos quais foram construdos gigantescos detectores construdos especialmente para poder detectar e visualizar as partculas to pequenas que preciso juntar milhes delas para formar um gro de areia. Essa ambiciosa experincia tem a participao de cientistas de vrios pases, dentre eles o brasileiro Carley Martins. Nenhum pas est disposto a gastar sozinho uma quantia to exorbitante quanto essa que foi gasta na construo desse incrvel e gigantesco equipamento, isso faz ver que o esprito de cooperao e colaborao de grande importncia para que o projeto realmente seja concludo. No dia 10 de setembro de 2008 os tneis do maior acelerador de partculas do mundo, denominado LHC (sigla para Grande Colisor de Hndrons), foram carregados com os primeiros feixes de prtons, que uma das partculas que formam o tomo. A grosso modo, o LHC funciona como um rodoanel para prtons, onde eles podero ser acelerados at 99,99% da velocidade da luz. Esses tneis possuem imensos e poderosssimos ims supercondutores que tm a capacidade de acelerar e fazer o desvio da rota dessas partculas, fazendo com que elas girem em sentidos opostos e se choquem. Com esse choque dos prtons os cientistas esperam recriar o big bang, a exploso que deu origem ao universo. Aps o choque, os prtons despedaados devem liberar milhes de partculas menores que os cientistas teorizam que possam existir. Essas partculas originadas do choque dos prtons foram denominadas pelos cientistas de partculas de Deus ou Bson Higgs. Segundo as teorias dos cientistas elas seriam as responsveis pela criao de todo o universo.

Prof: Sandro Fernandes

96

Apesar do grande gasto nessa mais nova e gigantesca experincia, existem grupos que buscam na justia fazer com que o experimento seja interrompido. Eles alegam que essa experincia pode ocasionar o fim do mundo, pois a coliso dos prtons pode formar um buraco negro com uma concentrao de energia to grande capaz de sugar tudo que est ao seu redor.
www.brasilescola.com

(V)

A Fsica E A Evoluo Dos Meios De Armazenamento De Informaes.

A humanidade, com o passar dos tempos, vem se evoluindo de modo rpido e gradativo, no que diz respeito s formas de armazenagem de informaes, e junto a essa revoluo est inserida a fsica, que busca diferentes maneiras para facilitar e aumentar a capacidade dos meios de armazenamento. Guardar informaes uma tarefa muito antiga realizada pelo ser humano. Seja atravs da escrita, fala ou at mesmo atravs das lendas e histrias passadas de gerao em gerao, as civilizaes mais antigas buscavam sempre um meio de preservar as suas culturas e dar continuidade aos seus costumes. Contudo, mesmo com toda evoluo tecnolgica, as histrias e as ledas ainda so muito utilizadas pelos povos indgenas como forma de armazenar e transmitir informaes de uma gerao para outra. Apesar de toda evoluo tecnolgica, ainda hoje a escrita o principal meio de guardar informaes, seja por meio de livros, jornais e at mesmo revistas. Contudo, com o avano tecnolgico surgiram novos meios e formas de armazenamento de informaes como, por exemplo, vdeos, filmes, DVDs, computadores e mais atualmente os revolucionrios celulares, mp4, HDs portteis, entre tantos outros que so cada vez menores, mas com alta capacidade de armazenamento. A cincia fsica se encontra inserida nesse processo de evoluo tecnolgica. por meio da nanotecnologia e da fsica de partculas, entre vrias outras reas das cincias, que os cientistas e pesquisadores tm conseguido construir equipamentos cada vez mais compactos e de alta capacidade de armazenamento de dados. A fsica de partculas uma rea da fsica que estuda os constituintes da matria. J a rea da nanotecnologia se baseia no princpio bsico da construo de estruturas e materiais a partir dos tomos como, por exemplo, os chips e os semicondutores. Tanto a fsica de partculas quanto a nanotecnologia, so duas reas em grande evoluo que buscam, incessantemente, meios cada vez mais eficazes para o armazenamento das informaes.
www.brasilescola.com

Prof: Sandro Fernandes

97

Algumas Aplicaes... 1) Dizem que o segredo de Kelly Slater colocar o Surf antes de tudo, at mesmo da famlia. O nico hexacampeo do WCT passa meses treinando nas ondas do Hawaii. Inovou o surfe mundial, incorporando ao esporte manobras radicais do skate, como o areo. um surfista que se adapta e se supera em qualquer tipo de mar. Nasceu na Flrida, cresceu na beira da praia, comeou a competir com 8 anos. Aos 14 anos j tinha mais de dez ttulos. Aos 18 anos se profissionalizou, mergulhou de cabea no treinamento e se tornou a maior lenda do Surf mundial. (http://www.sobresites.com/surf/surfistas.htm) Analise as afirmativas abaixo que relacionam alguns conceitos fsicos que podem ser identificados na prtica do surf. (I) (II) (III) A onda do mar que conduzir o surfista no possui nenhuma energia. Tanto a energia cintica como a energia potencial gravitacional so formas relevantes para o fenmeno da prtica do surf numa prancha. Por ser um tipo de onda mecnica, a onda do mar pode ser til para gerar energia para consumo no dia-a-dia. Dentre as alternativas, podemos afirmar que pode (em) estar correta (s): a) II b) I e II c) II e III d) I e II e) I, II e III

2) Logo na primeira jornada dos Discorsi intorno a due nuevo scienze, de Galileu Galilei, temos o seguinte dilogo entre Salviati (isto , Galileu) e seu discpulo Sagredo. Salvati: ... Quanto proporo entre os tempos de oscilao de mveis suspenso por fios de diferentes comprimentos, esses tempos esto entre si na mesma proporo que as razes quadradas dos comprimentos desses fios, o que quer dizer que os comprimentos esto entre si como os quadrados dos tempos...; do que se segue que os comprimentos dos fios esto entre si na proporo inversa dos quadrados os nmeros de oscilaes realizadas no mesmo tempo. Sagredo: Se entendi bem, eu poderia, portanto, conhecer rapidamente o comprimento de uma corda pendente de qualquer altura, ainda que o ponto a que esta atada fosse invisvel e somente se visse sua extremidade inferior. Com efeito, se amarro parte inferior da corda em questo um peso bastante grande, ao qual comunico um movimento de vaivm, e se um amigo conta o nmero de suas oscilaes enquanto ao mesmo tempo conto tambm as oscilaes de outro mvel, atado a uma corda com o comprimento exato de um cvado, a partir dos nmeros de oscilaes desses pndulos, efetuadas ao mesmo tempo, encontro o comprimento da corda. Salviati: E no teria errado nem mesmo de um palmo, especialmente se tomasse um grande nmero de oscilaes.

Prof: Sandro Fernandes

98

Esse texto coloca Galileu como um dos pioneiros no estudo da teoria da semelhana fsica e dos modelos. O pndulo constitudo pela corda comprida com um corpo pesado amarrado sua extremidade inferior o prottipo, e o pequeno pndulo, seu modelo reduzido. Galileu percebeu, no caso das oscilaes de pndulos, que: a) Quanto maior o comprimento do pndulo, menor ser o seu tempo de oscilao. b) O comportamento do prottipo poderia ser descrito a partir de observaes realizadas sobre o modelo. c) No era possvel relacionar o perodo de oscilao do prottipo se o comprimento do fio fosse muito longo. d) Os comprimentos dos fios esto entre si na proporo direta dos quadrados dos nmeros de oscilaes realizadas em um tempo qualquer. e) Pegando um grande nmero de oscilaes o erro das medidas obtidas aumentaria. 3) Observando uma onda unidimensional, que se propaga com velocidade constante e sem perda de energia, produzida pela sucesso de uma srie de abalos de mesma freqncia, tem-se que o afastamento entre duas cristas consecutivas representa a grandeza fsica denominada a) altura. d) comprimento de onda. b) amplitude. c) freqncia. e) velocidade de propagao da onda.

4) Em 2005, Ano Mundial da Fsica, comemora-se o centenrio da Teoria da Relatividade de Albert Einstein. Entre outras conseqncias esta teoria poria fim idia do ter, meio material necessrio, semelhantemente ao som, atravs do qual a luz se propagava. O jargo popular "tudo relativo" certamente no se deve a ele, pois seus postulados esto fundamentados em algo absoluto: a velocidade da luz no vcuo - 300000 km/s. Hoje sabe-se que: I. O som propaga-se no vcuo. II. A luz propaga-se no vcuo. III. A velocidade da luz no vcuo a velocidade limite do universo. (so) verdadeira(s): a) todas b) nenhuma c) somente II d) II e III e) somente III

Prof: Sandro Fernandes

99

5) Analise o quadro a seguir:

A emissora de TV utiliza ondas eletromagnticas para sua transmisso e recepo e possui uma freqncia de vibrao. A freqncia emitida das ondas da emissora de TV : a) inferior da radiodifuso e superior das microondas. b) inferior das microondas e superior dos infravermelhos. c) inferior da luz visvel (faixa preta) e superior da radiodifuso. d) inferior da luz visvel (faixa preta) e superior dos raios X. e) inferior das ultravioletas e superior das microondas. 6) Um trem de ondas senoidais, gerado por um dispositivo mecnico oscilante, propaga-se ao longo de uma corda. A tabela a seguir descreve quatro grandezas que caracterizam essas ondas mecnicas.

As grandezas 1, 2, 3 e 4 so denominadas, respectivamente, a) freqncia, fase, amplitude e comprimento de onda. b) fase, freqncia, comprimento de onda e amplitude. c) perodo, freqncia, velocidade de propagao e amplitude. d) perodo, freqncia, amplitude e comprimento de onda. e) freqncia, perodo, comprimento de onda e amplitude.

Prof: Sandro Fernandes

100

7) O grfico do movimento de subida e descida de uma rolha, na superfcie de um lago ondulado, mostrado na figura a seguir, em que y a altura da rolha em relao ao nvel da gua parada e t o tempo transcorrido.

Se a rolha leva 1,0 s para sair do nvel zero e atingir, pela primeira vez, a altura mxima, a freqncia do movimento igual a a) 0,125 Hz b) 0,25 Hz c) 0,50 Hz d) 1,0 Hz e) 4,0 Hz

8) Ao iluminar a caverna, o espeleologista descobre um lago cristalino e observa que a gua de uma infiltrao atravs das rochas goteja periodicamente sobre o lago, provocando pulsos ondulatrios que se propagam em sua superfcie. Ele capaz de estimar a distncia (d) entre dois pulsos consecutivos, assim como a velocidade (v) de propagao dos mesmos. Com o aumento da infiltrao, o gotejamento aumenta e a quantidade de gotas que cai sobre a superfcie do lago, por minuto, torna-se maior. Comparando essa nova situao com a anterior, o espeleologista observa que: a) v permanece constante e d aumenta; b) v aumenta e d diminui; c) v aumenta e d permanece constante; d) v permanece constante e d diminui; e) v e d diminuem. 9) Cientistas descobriram que a exposio das clulas humanas endoteliais radiao dos telefones celulares pode afetar a rede de proteo do crebro. As microondas emitidas pelos celulares deflagram mudanas na estrutura da protena dessas clulas, permitindo a entrada de toxinas no crebro. ("Folha de S.Paulo", 25.07.2002) As microondas geradas pelos telefones celulares so ondas de mesma natureza que a) o som, mas de menor freqncia. b) a luz, mas de menor freqncia. c) o som, e de mesma freqncia. d) a luz, mas de maior freqncia. e) o som, mas de maior freqncia.

Prof: Sandro Fernandes

101

10) Uma cena comum em filmes de fico cientfica a passagem de uma nave espacial em alta velocidade, no espao vazio, fazendo manobras com a ajuda de foguetes laterais, tudo isso acompanhado e um forte rudo. Assinale a alternativa CORRETA. a) A cena correta, pois no h problema com o fato de uma nave voar no espao vazio. b) A cena correta, porque perfeitamente perceptvel o rudo de uma nave no espao vazio. c) A cena no correta, pois o som no se propaga no vcuo. d) A cena no correta, pois no possvel que uma nave voe no espao vazio. e) A cena no correta, pois no possvel fazer manobras no espao vazio. 11) Os versos a seguir lembram uma poca em que a cidade de So Paulo tinha iluminao a gs: "Lampio de gs! Lampio de gs! Quanta saudade Voc me traz. Da sua luzinha verde azulada Que iluminava a minha janela Do almofadinha, l na calada Palheta branca, cala apertada" (Zica Bergami) Quando uma "luzinha cor verde azulada" incide sobre um carto vermelho, a cor da luz absorvida : a) verde e a refletida azul b) azul e a refletida verde c) verde e a refletida vermelha d) verde azulada e nenhuma refletida e) azul e a refletida vermelha 12) "O mundo permanecia na escurido. Deus disse: 'Faa-se Newton', e tudo foi luz" (traduo livre). Com esse verso, um poeta ingls homenageia Sir Isaac Newton. Newton, o brilhante cientista, formulou leis fsicas em vrios campos, EXCETO no campo da a) ressonncia magntica. b) inrcia dos corpos. c) gravitao. d) dinmica. e) teoria corpuscular da luz.

Prof: Sandro Fernandes

102

13)

Nuvem negra A astcia faz com que os polvos no percam tempo diante de um inimigo. Apesar de serem surdos, como todos os membros da famlia cefalpode, eles enxergam com impressionante nitidez. Seus olhos possuem 50 000 receptores de luz por milmetro quadrado, o que lhes d uma viso melhor do que a humana. Os adversrios tambm so reconhecidos pelo olfato. As pontas dos oito tentculos funcionam como narizes, com clulas especializadas em captar odores. Provavelmente, o bicho percebe pelo cheiro que o outro animal est liberando hormnios relacionados ao comportamento agressivo. Ou seja, pretende atac-lo. Ento lana uma tinta escura e viscosa para despistar o agressor. E escapa numa velocidade impressionante para um animal aqutico. "SUPER INTERESSANTE". Ano 10, n. 2. fevereiro 1996. p. 62. Esse procedimento usado pelos polvos tem por objetivo dificultar a viso de seus inimigos. No entanto esse recurso das cores pode ser usado tambm com a finalidade de comunicao. Para haver essa comunicao, necessrio, porm, que ocorra o fenmeno fsico da a) refrao da luz. d) induo da luz. b) absoro da luz. e) disperso da luz. c) reflexo da luz.

14) Num dia sem nuvens, ao meio-dia, a sombra projeta no cho por uma esfera de 1,0cm de dimetro bem ntida se ela estiver a 10cm do cho. Entretanto, se a esfera estiver a 200cm do cho, sua sombra muito pouco ntida. Pode-se afirmar que a principal causa do efeito observado que: a) o Sol uma fonte extensa de luz. b) o ndice de refrao do ar depende da temperatura. c) a luz um fenmeno ondulatrio. d) a luz do Sol contm diferentes cores. e) a difuso da luz no ar "borra" a sombra. 15) Admita que o sol subitamente "morresse", ou seja, sua luz deixasse de ser emitida. 24 horas aps este evento, um eventual sobrevivente, olhando para o cu, sem nuvens, veria: a) a Lua e estrelas. d) uma completa escurido. b) somente a Lua. c) somente estrelas. e) somente os planetas do sistema solar.

16) s 18h, uma pessoa olha para o cu e observa que metade da Lua est iluminada pelo Sol. No se tratando de um eclipse da Lua, ento correto afirmar que a fase da Lua, nesse momento: a) s pode ser quarto crescente b) s pode ser quarto minguante c) s pode ser lua cheia. d) s pode ser lua nova. e) pode ser quarto crescente ou quarto minguante.

Prof: Sandro Fernandes

103

17) Quando o Sol est a pino, uma menina coloca um lpis de 7,0x10-3 m de dimetro, paralelamente ao solo, e observa a sombra por ele formada pela luz do Sol. Ela nota que a sombra do lpis bem ntida quando ele est prximo ao solo mas, medida que vai levantando o lpis, a sombra perde a nitidez at desaparecer, restando apenas a penumbra. Sabendo-se que o dimetro do Sol de 14x108 m e a distncia do Sol Terra de 15x1010 m, pode-se afirmar que a sombra desaparece quando a altura do lpis em relao ao solo de: a) 1,5 m. b) 1,4 m. c) 0,75 m. d) 0,30 m. e) 0,15 m.

18) O esquema a seguir mostra a formao da imagem em uma luneta astronmica.

Numa certa luneta as distncias focais da objetiva e da ocular so de 60cm e 30cm, respectivamente, e a distncia entre elas de 80cm. Nessa luneta a imagem final de um astro distante se formar a a) 30cm da objetiva. c) 40cm da objetiva. e) 60cm da ocular. 19) A utilizao da luneta astronmica de Galileu auxiliou a construo de uma nova viso do Universo. Esse instrumento ptico, composto por duas lentes - objetiva e ocular - est representado no esquema a seguir. b) 30cm da ocular. d) 60cm da objetiva.

Considere a observao de um objeto no infinito por meio da luneta astronmica de Galileu. Nesse caso, as imagens do objeto formado pelas lentes objetiva e ocular so, respectivamente: a) real e direita; virtual e direita c) virtual e invertida; real e invertida e) real e invertida; virtual e direita b) real e invertida; virtual e invertida d) virtual e direita; real e direita

Prof: Sandro Fernandes

104

20) O tipo de lente da histria do Bidu usado para corrigir:

a) miopia e astigmatismo. c) presbiopia e hipermetropia. e) astigmatismo e estrabismo.

b) hipermetropia e miopia. d) presbiopia e miopia.

21) A figura a seguir mostra esquematicamente dois defeitos de viso, que podem ser corrigidos pelo uso das seguintes lentes:

a) convergentes para os casos A e B. b) divergentes para os casos A e B. c) convergente para o caso A e divergente para o B. d) divergente para o caso A e convergente para o B. e) um dos defeitos mostrados no pode ser corrigido com o uso de lentes. 22) A figura a seguir mostra um objeto A colocado a 5m de um espelho plano, e um observador O, colocando a 7m deste mesmo espelho. Um raio de luz que parte de A e atinge o observador O por reflexo no espelho percorrer, neste trajeto de A para O.

a) 9m
Prof: Sandro Fernandes

b) 12m

c) 15m

d) 18m

e) 21m

105

23) Um excitador pulsado que gera fascas a uma freqncia de 10 6 Hz est localizado no centro de curvatura C de um espelho cncavo de 1m de raio de curvatura. Considere que o tempo de durao de cada fasca seja desprezvel em relao ao intervalo de tempo entre duas fascas consecutivas. A 2m do centro de curvatura do espelho est situado um anteparo normal aos raios refletidos. O espelho gira em torno de C com uma freqncia de 500 rotaes por segundo, formando faixas luminosas eqidistantes no anteparo. O comprimento do intervalo entre duas faixas luminosas formadas pelos raios refletidos no anteparo de, aproximadamente: a) 3,1 mm. b) 6,3 mm. c) 12,6 mm. d) 1,0 mm. e) 9,4 mm.

24) Isaac Newton foi o criador do telescpio refletor. O mais caro desses instrumentos at hoje fabricado pelo homem, o telescpio espacial Hubble (1,6 bilho de dlares), colocado em rbita terrestre em 1990, apresentou em seu espelho cncavo, dentre outros, um defeito de fabricao que impede a obteno de imagens bem definidas das estrelas distantes (O Estado de So Paulo, 01/08/91, p.14). Qual das figuras a seguir representaria o funcionamento perfeito do espelho do telescpio?

25) Um objeto linear de altura h est assentado perpendicularmente no eixo principal de um espelho esfrico, a 15 cm de seu vrtice. A imagem produzida direita e tem altura de h/5. Este espelho a) cncavo, de raio 15 cm. c) convexo, de raio 7,5 cm. e) convexo, de raio 10 cm. 26) O esquema a seguir representa a direo de um feixe luminoso monocromtico incidente e as direes dos respectivos feixes refletido e refratado. b) cncavo, de raio 7,5 cm. d) convexo, de raio 15 cm.

Sabendo-se que o ngulo de reflexo vale 60, que o ndice de refrao do meio A vale 1 e que o do meio B vale 31/2, correto afirmar que o ngulo de refrao vale: a) 15 b) 30 c) 45 d) 60 e) 90

Prof: Sandro Fernandes

106

27) De uma lanterna colocada no ar sai um estreito feixe de luz que incide na superfcie de separao entre o ar e um lquido transparente, refratando-se conforme mostra a figura abaixo.

O ndice de refrao do lquido a) 1,28 b) 1,33 c) 1,39 d) 1,46 e) 1,51

28) A figura mostra uma placa de vidro com ndice de refrao n(v) = 21/2 mergulhada no ar, cujo ndice de refrao igual a 1,0. Para que um feixe de luz monocromtica se propague pelo interior do vidro atravs de sucessivas reflexes totais, o seno do ngulo de entrada, sen (e), dever ser menor ou igual a

a) 0,18

b) 0,37

c) 0,50

d) 0,71

e) 0,87

29) A lente da historinha do Bidu pode ser representada por quais das lentes cujos perfis so mostrados a seguir?

a) 1 ou 3

b) 2 ou 4

c) 1 ou 2

d) 3 ou 4

e) 2 ou 3

Prof: Sandro Fernandes

107

Gabarito
Aula I 1 2 3 4 5 6 7 8 9 10 11 12 13 14 15 16 17 18 19 20 21 22 23 24 25 26 27 28 29 30 A C C C B A C E B A A D A C B B C C E C B B D D E E C C B D Aula II X X X X X X X E D C C A A B B B D B A C B C C C C E E B A C Aula III A C B C E B B B B A C B A D C B A E B B A C B B A X X X X X Aula IV E B E E C B A E D E E B C C C E C C B B X X X X X X X X X X Aula V C D B A B E E B C D A A E D D B C B D A E E A A C D B C A A Aula VI C A B E D D E A D E A A E C E C A D E E D D B B C X X X X X Aula VII C B D D C E B D B C D A C A C A C E B C C C B D C B B B B X

Prof: Sandro Fernandes

108

Você também pode gostar